Phương pháp thế và sử dụng tính chất ánh xạ giải toán phương trình hàm trên R

Tài liệu gồm 59 trang, hướng dẫn áp dụng phương pháp thế và phương pháp sử dụng tính chất ánh xạ trong việc giải bài toán phương trình hàm trên R. Trong chương trình chuyên Toán ở các trường THPT chuyên, phương trình hàm là một chuyên đề quan trọng. Hiện nay tài liệu về phương trình khá phong phú. 

Chủ đề:
Môn:

Toán 1.9 K tài liệu

Thông tin:
59 trang 1 năm trước

Bình luận

Vui lòng đăng nhập hoặc đăng ký để gửi bình luận.

Phương pháp thế và sử dụng tính chất ánh xạ giải toán phương trình hàm trên R

Tài liệu gồm 59 trang, hướng dẫn áp dụng phương pháp thế và phương pháp sử dụng tính chất ánh xạ trong việc giải bài toán phương trình hàm trên R. Trong chương trình chuyên Toán ở các trường THPT chuyên, phương trình hàm là một chuyên đề quan trọng. Hiện nay tài liệu về phương trình khá phong phú. 

81 41 lượt tải Tải xuống
Tài liu chuyên đề bi ng hc sinh gii
CHUYÊN ĐÊ 1: ĐẠI S VÀ GII TÍCH 1 |
ĐẠI SỐ VÀ GIẢI TÍCH
PHƯƠNG PHÁP THẾ VÀ PHƯƠNG PHÁP SỬ DỤNG TÍNH CHẤT ÁNH XẠ
TRONG VIỆC GIẢII TOÁN PHƯƠNG TRÌNH HÀM TRÊN
Trong chương trình chuyên toán ở các trường THPT chuyên Phương trình hàm là một chuyên đề quan
trọng. Hiện nay tài liệu về phương trình khá phong phú. Tuy vậy, việc giải được phương trình hàm vẫn là
vấn đề khó đối với nhiều học sinh. Trong chuyên đề nhỏ y, chúng tôi sẽ trình bày hai phương pháp
thông dụng và quan trọng để giải phương trình m trên tập
.
Đó là Phương pháp thếPhương pháp
sử dụng tính chất ánh xạ.
I. Phương pháp thế trong giải phương trình hàm
1. Một số lưu ý khi sử dụng phương pháp thế
Quan sát cấu trúc của phương trình xem giữa các biến có tính đối xứng không? Nếu trong phương
trình có tính đối xứng giữa
x
y
thì ta thường hoán vị các biến này cho nhau.
Thay các biến bởi các gtrị thích hợp thoả mãn các điều kiện bài toán. Điều cần chú ý c giá tr của
biến phải thuộc tập xác định của hàm số. Ta tờng chọn giá trị cho biến sao cho sau khi thay ta thu
được một phương trình hàm đơn giản hơn hoặc có thnhận được dạng của hàm số (tuyến tính, bậc hai,
mũ, logarit,…). Trong trưng hợp
f
cộng tính ta thể thay
x
bởi
1x +
và quy giá trị hàm số về
( )
1 .f
Mt vài phép thế đặc bit:
0, 1; 0, 0, 1, 1, , ( ),...x y x y x y x y x y x f y= = = = = = = = = =
Tính các giá tr đặc bit ca hàm s
, chng hn:
( )
.0 , ( ..1)f f
2. Các ví d:
Ví d 1: Cho hàm s
f
tho mãn điều kin:
( )
0 2f =
( ) ( )
,f x y f y x+ = +
, .x y
Tính
( )
1998 .f
Li gii:
Giả sử tồn tại hàm
f
thoả mãn điều kiện đề bài. Cho
0y =
, ta
( ) (0) 2 ,f x f x x= + = +
.x
Thử lại thấy thoả mãn. Do đó:
(1998) 2 1998 2000f = + =
Ví dụ 2: (Australia 1995) Tìm tất cả các hàm
( )
: 0;f +
thoả mãn các điều kiện
( )
1
1
2
f =
( )
( )
( )
3 3
f xy f x f f y f
y x
= +
,
, 0.x y
(1)
PHƯƠNG TNH HÀM TRÊN
1
Chuyên đề
Tài liu chuyên đề bi ng hc sinh gii
2 | C CHUYÊN ĐÊ: PHƯƠNG TRÌNH HÀM
Li gii:
Cho
1, 3x y= =
thay vào (1) ta được:
( ) ( ) ( ) ( )
2
2
1 1 1
3 3 3 0 3
4 2 2
f f f f
= + = =
,x y
+
Thay
1x =
vào (1) ta được:
( ) ( ) ( )
1 3 1 3
2 2
f y f f y f f y
y y
= + =
y
+
Khi đó (1) trở thành:
( ) ( ) ( )
2f xy f x f y=
,x y
+
(2)
Thay
y
bởi
3
x
vào (2) ta có:
2
1
( )
3 1
2
(3) 2 ( ) 2 ( )
1
2
( )
2
f x
f f x f f x
x
f x
=
= =
=
Thay
y x=
vào (2) có:
( )
( ) ( )
2 2
1 1 1
2 2. , 0
4 2 2
f x f x f x x= = = =
Thử lại thấy thoả mãn.
Ví dụ 3: (Nhật Bản 2012)Tìm tất cả các hàm số
:f
sao cho:
2
( ( ) ( )) (y)f f x y f x y x yf+ =
,x y
(1)
Li gii:
T (1) cho
0x y= =
ta thu được
2
( (0)) 0.f f =
T (1) cho
( )
2
0, 0 ,x y f= =
kết hp vi kết qu
( )
( )
2
0 0f f =
ta thu được
( )
0 0.f =
T (1) cho
có:
2
( ) 0 ( )x xf x f x x = =
,
0.x
Hơn nữa
( )
0 0f =
, suy ra:
( )
,f x x=
.x
Th li ta thy tho mãn.
Ví dụ 4: Tìm tất cả các hàm số
:f
thoả mãn:
( ( ) ) ( )f xf y x xy f x+ = +
,x y
(1)
Li gii:
Trong (1) thay
1x =
, ta được
( ( ) 1) (1)f f y y f+ = +
,
y
(2)
Trong (2) thay
y
bởi:
( )
1 1f
ta có:
( )
1 1( ( ) 1)1f f f + =
Đặt:
( ) ( )
1 1 1( ) ; 0f f a f b+ = =
Ta có:
( )
( )
( )
0 .f xf a x f b+ = =
Hay:
( )
( )
( ) ( )
b f xf a x ax f x f x ax b= + = + = +
Thay biu thc ca
vào (1) ta được:
( ( ) ) ,a xf y x b xy ax b + + = +
,x y
( ) ,a x ay b x b xy ax b + + + = +
,x y
2
,a xy abx ax b xy ax b + = +
,x y
Bằng cách đồng nht các h s ta được:
2
1
0
1
1
0
a
b
a
ab a a
a
b
=
=
=
+ =
=
=
Vy
( )
,f x x=
x
( )
,f x x=
.x
Th li thy tho mãn.
Ví d 5: (Serbia 2014, Đồng Nai 2018) Xác định tt c các hàm s
:f
tha mãn
Tài liu chuyên đề bi ng hc sinh gii
CHUYÊN ĐÊ 1: ĐẠI S VÀ GII TÍCH 3 |
( ) ( )
( )
( )
f xf y yf x f xy xy =
vi mi
, .x y
(1)
Li gii:
Cho
0y =
được
( )
( )
( )
0 0f xf f=
. Nếu
( ) ( )
0 0f f x
là hàm hng. Th li thy không tha mãn.
Do đó
( )
0 0.f =
Cho
được
( )
( ) ( )
2 2
0 0 , 0f x x f f x x x = = =
Gi s
( )
: 0a f a =
. Cho
, 1x a y= =
được
( )
( )
( ) ( )
1 0 0f af f a a a f a a a a= = = = =
Vi
, 0x y
ta có
( ) ( )
( )
( ) ( ) ( )
( )
( ) ( ) ( )
0
0 , 0.
f xf y yf x f xy xy f xf y yf x xy xy
xf y yf x f x cx x
= = =
= =
Xét
0x y
thay vào (1) có:
( )
( )
( ) ( ) ( )
1 1 . 2f c xy f xy xy c xy = =
Nếu
( )
1 , .c f x x x= ⎯⎯ =
Nếu
1c
, do
( )
( )
( ) ( )
1 1 ,c 1f c xy c xy c xy
nên t (2) suy ra
( )
1 , .c f x x x= ⎯⎯ =
Vy có 2 hàm tha mãn bài toán
( )
, .f x x x=
hoc
( )
, .f x x x=
Ví d 6: Tìm hàm
:f
tha mãn:
( ) ( ) ( )
( )
( )
2 2
( ) 4 , 1 .x y f x y x y f x y xy x y x y + + = +
Li gii:
Ta có:
( ) ( ) ( ) ( ) ( )
( ) ( ) ( ) ( ) ( ) ( ) ( ) ( )
2 2
1
1 1
4 4
x y f x y x y f x y
x y x y x y x y x y x y x y x y
+ + =
= + + + + + + +
Đặt
u x y
v x y
=
= +
ta được:
( ) ( ) ( )( ) ( ) ( )
( )
( ) ( ) ( )
( )
( )
( )
2 2
3 3 3 3
1
4
vf u uf v u v u v u v u v
vf u uf v u v v u v f u u u f u v
= + +
= =
+ Vi
0uv
ta có:
( ) ( ) ( )
( )
3 3 3
* 3
, , 0
f u u f v v f u u
u v a f u au u u
u v u
= = = +
Tài liu chuyên đề bi ng hc sinh gii
4 | C CHUYÊN ĐÊ: PHƯƠNG TRÌNH HÀM
+ Vi
0; 0u v=
suy ra:
( ) ( ) ( )
3 3
0 0 0.f u u f u u f = = =
Hàm
( )
3
f u au u= +
tha mãn
( )
0 0f =
. Vy
( )
3
,f u au u u= +
Hàm s cn tìm là:
( ) ( )
3
f u ax x a= +
. Th li thấy đúng.
Ví d 7: (Áo 1996) Tìm tt c các hàm s
:f
thỏa mãn điều kin
( ) ( )
2 4
1 2 ,x f x f x x x x+ =
Li gii:
Thay
x
bi
1 x
ta được
( ) ( ) ( ) ( ) ( )
2 4
1 1 2 1 1x f x f x x x + =
Như vậy ta có h
( ) ( )
( ) ( ) ( ) ( ) ( )
2 4
2 4
1 2 2
1 1 2 1 1
x f x f x x
x f x f x x x
+ =
+ =
Ta có
( )( )
2 2
1 1D x x x x= +
( )( )( )
2 2 2
1 1 1
x
D x x x x x= +
. Vy
( )
. ,
x
D f x D x=
. T đó ta
có nghim ca bài toán là
( )
2
4 2
1 : ,
:
2 :2
x x a x b
f x c x a
a a a b
= =
=
(
c
là hng s tùy ý),
Vi
,a b
là nghim của phương trình
2
1 0x x =
Nhn xét: bài toán trên được dùng mt ln na trong kì thi VMO 2000, bng B.
Ví d 8: (VMO 2005) Hãy xác định tt c các hàm s
:f
thỏa mãn điều kin
( )
( )
( ) ( ) ( ) ( )
, ,f f x y f x f y f x f y xy x y = +
(8)
Li gii:
Thế
0x y= =
vào (8) ta được
( )
( )
( )
( )
2
0 0f f f=
Thế
x y=
vào (8) và s dng kết qu trên thì
( )
( )
( )
( )
2 2
2
0 ,f x f x x= +
Suy ra
( )
( )
( )
( )
( ) ( )
2 2
,f x f x f x f x x= =
.
Tài liu chuyên đề bi ng hc sinh gii
CHUYÊN ĐÊ 1: ĐẠI S VÀ GII TÍCH 5 |
Thế
0y =
vào (8) được
( )
( )
( ) ( ) ( ) ( )
0 0 ,f f x f f x f x f x= +
(*)
Thế
0,x y x= =
vào (8) được
( )
( )
( ) ( ) ( )
0 ,f f x f f x f x a x= +
.
T hai đằng thc trên ta có
( ) ( ) ( )
( )
( ) ( ) ( )
0 2 0 ,f f x f x f x f x f x + + =
. (9)
Gi s tn ti
0
0x
sao cho
( ) ( )
0 0
f x f x=
, thì thế
0
x x=
vào (9) ta
( ) ( )
( )
( )
( )
( )
( )
( )
( )
( )
0
2 2
0
2 2
2 2
0
0
0
0
0 0 0
0
f x f
f x f
f x f
x
=
=
+ = +
=
Suy ra mâu thun
Vy
( ) ( )
,f x f x x=
, t điều này kiết hp vi (9) ta có
( ) ( )
( )
0 1 0,f f x x =
T đây suy ra
( )
0 0f =
, vì nếu ngược li thì
( )
1, 0f x x=
, trái với điều kin
f
là hàm l. T đây ta nhận
được quan h quen thuc
( ) ( )
( )
( )
0 0 0 0 0
x f x f f x f x x= = = =
Vô lý. Vy chng t
( )
,f x x x=
. Th li thy hàm này tha mãn bài toán.
Nhn xét: Bài toán trên cho kết qu là hàm chn
( )
f x x=
. Nếu vn gia nguyên vế phải và để nhận được
hàm l
( )
f x x=
, ta sa li d kin trong vế trái như trong ví d sau
Ví d 9: Tìm tt c các hàm s
:f
thỏa mãn điều kin
( )
( )
( ) ( ) ( ) ( )
, ,f f x y f x f y f x f y xy x y = +
Li gii:
Thế
0y =
ta được
( )
( )
( ) ( ) ( ) ( )
0 0 ,f f x f x f f f x x= +
(10)
Thế
( )
y f x=
và s dng kết qu trên, ta được
Tài liu chuyên đề bi ng hc sinh gii
6 | C CHUYÊN ĐÊ: PHƯƠNG TRÌNH HÀM
( ) ( ) ( )
( )
( ) ( )
( )
( ) ( )
( ) ( ) ( ) ( )
( )
( ) ( )
( )
( )
2 2
0 . *
0 0 0 . ,
f f x f f x f x f x xf x
f xf f x f x f f x xf x
= +
= + +
Hay
( ) ( ) ( )
( )
( ) ( )
( )
( )
2 2
2 0 . 0 . 0,f f x f x f f x xf x x + + =
.
Thế
0x =
vào đẳng thức trên ta được
( )
( )
( )
( )
( )
2 2
0 0 0 0 0f f f = =
hoc
( )
0 1f =
.
Nếu
( )
0 0f =
thì thay vào (10) ta có
( )
( )
( )
,f f x f x x=
, thay kết qu này vào trong (*) ta có
( )
f x x=
.
Nếu
( )
0 1f =
thay vào (10) ta có
( )
( )
( )
2 1f f x f x=
, thay vào trong (*) ta có
( )
1
1
2
f x x= +
.
Kết lun: thay vào ta thy ch có hàm s
( )
,f x x x=
là tha mãn yêu cu.
Ví d 10: Tìm tt c các hàm s
:f
tha mãn
( )
( )
( )
( )
( )
2
2
2
2 , ,f x y f x xf y y x y = +
.
Li gii:
Thay
0x y= =
thì
( )
( )
( )
( )
( )
2
0 0 0 0f f f= =
hoc
( )
0 1f =
Trường hp 1: Nếu
( )
0 0f =
, thì thay
x y=
vào điều kiện ban đầu ta đưc
( ) ( )
( )
( ) ( )
( )
( )
2 2
2
0 2 ,f f x xf x x f x x f x x x= + = =
.
Nhn thy hàm s này tha mãn.
Trường hp 2: Nếu
( )
0 1f =
thì li vn thay
0x y= =
ta nhận được, vi mi
x
thì hoc là
( )
1f x x= +
hoc là
( )
1f x x=
. Gi s tn ti giá tr
a
sao cho
( )
1f a a=
. Khi đó thay
, 0x a y= =
ta được
( )
2 2
4 1f a a a= +
Nhưng ta lại hoc là
( )
2 2
1f a a= +
hoc
( )
2 2
1f a a=
. Do đó ta phải có hoc là
2 2
4 1 1a a a + = +
hoc
2 2
4 1 1a a a + =
, tc
0a =
hoc
1
2
a =
. Tuy nhiên kiểm tra đều không tha.
Vy hàm s tha mãn yêu cu
( )
,f x x x=
hoc là
( )
1,f x x x= +
.
Ví d 11: Tìm tt c các hàm s
:f
thỏa mãn điều kin
( )
( )
( )
(
)
( )
( )
2
3 3
2 3 , ,f x y y f x y f x f y x y + + = +
.
Li gii:
Thay
3
y x=
ta có
Tài liu chuyên đề bi ng hc sinh gii
CHUYÊN ĐÊ 1: ĐẠI S VÀ GII TÍCH 7 |
( ) ( )
( )
(
)
( )
( )
2
3 6 3
0 2 3 ,f x f x x f x f x x+ + = +
Thay
( )
y f x=
ta được
( )
( )
( ) ( )
( )
( )
( )
(
)
( )
2 2
3
2 3 0 ,f x f x f x f x f x f x+ + =
.
T hai đẳng thức trên ta được
( )
( )
(
)
( )
( )
2 3
3 6
2 3 8 , .x f x x f x x+ =
Do đó
( )
( )
( )
( )
(
)
( )
( )
( )
( )
(
)
( )
( )
(
)
( )
( )
( )
( )
( )
( )
(
)
( )
( )
( )
2 2
3 6
3 2 2
3 3 9
3
2
3 3 3 3 6
0 4 3
4 4 . .
15
4 2 .
4 16
f x x f x x
f x f x x f x x x
x
f x x f x x f x x f x x f x x
= +
= +
= + + = + +
Chú ý rng
( )
2
3
6
15
2 0
4 16
x
f x x
+ + =
thì
( )
0, 0 0x f= =
. Bi vy trong mọi trường hợp ta đều có
( )
3
f x x=
. Th li thy hàm s này tha mãn bài toán.
3. Bài tập vận dụng
Bài tập 1: (Slovenia 1999)Tìm tất cả các hàm
:f
thỏa mãn
( )
( )
( )
1 , , 1f x f y x y x y =
Li gii:
Thay
x
bởi
( )
f y
trong
( )
1
được
( ) ( ) ( ) ( ) ( )
0 1 0 1 2f f y y f y y f= = +
Cho
0y =
thay vào
( )
2
được
( )
1
0
2
f =
. Suy ra
( )
1
2
f y y y= +
Bài 2: (Khánh Hòa 2017) Tìm tt c các hàm
:f
tha mãn:
( ) ( ) ( )
1 2 1 , .f xy f x y f x y xy x x y+ + + + = + +
Li gii:
Đặt
( ) ( )
g x x f x+ =
thì phương trình hàm đã cho trở thành:
( ) ( ) ( ) ( )
1 0 , . 1g xy g x y g x y x y+ + + + =
Trong (1) lần lượt cho
( )
0, 0; 0, 1; 0, 2; 1, 1; 0 0.x y x y x y x y g= = = = = = = = =
Trong (1) cho
( ) ( ) ( )
0 1 0 . 2y g x g x x= + + =
Tài liu chuyên đề bi ng hc sinh gii
8 | C CHUYÊN ĐÊ: PHƯƠNG TRÌNH HÀM
Trong (1) cho
( ) ( ) ( ) ( )
1 1 0 . 3y g x g x g x x= + + + =
T (2) và (3) suy ra
( ) ( ) ( )
0 0g x x g x x f x x x = = =
. Th li thy tha mãn.
Bài 3: Tìm tt c các hàm s
:f
, biết rng
f
là hàm s chn và tha mãn:
( ) ( ) ( ) ( )
( )
2014 2 1f xy f x f y f x y xy = +
vi mi
, .x y
.
Li gii:
T (1) cho
0y =
, ta có:
( ) ( ) ( ) ( )
( )
( )
( )
( )
( )
0 . 0 2014 1 1 0 2014 0f f x f f x f x f = + =
vi mi
.x
Nếu
( )
0 2014f
thì
( )
1f x =
vi mi
x R
. Khi đó
f
không tha mãn (1)
Do đó
( )
0 2014.f =
T (1) thay
x
bi
x
y
bi
x
, ta được:
( )
( )
( )
( )
( )
2
2 2
2014 2 2 1f x f x f x x =
. (2)
T (1) thay
x
bi
x
y
bi
x
, ta được:
( )
( ) ( ) ( )
( )
2 2
2014 0 2 1 .f x f x f x f x = +
(3)
f
là hàm s chn nên viết (3) lại như sau:
( )
( )
( )
( )
( )
2
2 2
2014 0 2 1f x f x f x = +
. (4)
Ly (4) tr (2) vế theo vế ta được:
( ) ( )
2
2 4 0f x x f= +
vi mi
.x
Suy ra:
( ) ( )
2 2
0 2014f x x f x= + =
vi mi
.x
.
Bài 4: (Thanh Hóa 2017) Tìm tt c các hàm
:f
tha mãn:
( ) ( )
( )
( )
( ) ( )
( )
( )
2
2 2
2 , . 1f f x f y f x x f y f y x y+ = + +
Li gii:
Trong (1) lần lượt cho:
( ) ( )
( )
( ) ( ) ( ) ( )
2
1, 0 1 0 1 2 0 0 . 2x y f f f f f f= = + = + +
( ) ( )
( )
( ) ( ) ( )
2
0, 1 0 1 0 1 . 3x y f f f f f= = + = +
Tài liu chuyên đề bi ng hc sinh gii
CHUYÊN ĐÊ 1: ĐẠI S VÀ GII TÍCH 9 |
T (2) và (3) suy ra
( ) ( )
1 0f f=
hoc
( ) ( )
1 0 1.f f= +
Nếu
( ) ( )
1 0f f=
ta có
( ) ( ) ( ) ( ) ( ) ( )
2
0 1 2 0 0 0 1 0.f f f f f f= + + = =
Trong (1) cho
0x =
có:
( )
( )
( )
2
.f f y f y y=
Trong (1) cho
1x =
có:
( )
( )
( ) ( )
2
2 .f f y f y f y y= +
T đó suy ra
( )
.f y y y=
Nếu
( ) ( )
1 0 1f f= +
, trong (1) thay
x
bởi
y
,
y
bởi
x
được
( )
( ) ( )
( )
( )
( ) ( )
( )
( )
2 2
2 2 2 2
2 2 , . 4f x x f y f y f y y f x f x x y+ + = + +
Trong (4) lần lượt cho
0, 1x x= =
ta được:
( ) ( )
( )
( )
( ) ( )
( )
2 2
2 2
0 2 0 0f f y f y y f f y+ = + +
( ) ( ) ( )
( )
( )
( ) ( )
( )
2 2
2 2
1 2 2 1 1 .f f y f y f y y f f y+ + = + +
T hai phương trình trên kết hp
( ) ( )
1 0 1f f= +
ta được
( ) ( )
2
0 .f x x f x= +
Th li thy
( )
0 0.f =
Hàm
( )
2
f x x x=
tha mãn. Vy có hai hàm tha mãn
( )
0f x x=
hoc
( )
2
f x x x=
.
Bài 5: (Chuyên ĐH Vinh 2017) Tìm tt c các hàm
:f
tha mãn:
1.
( )
1 0f
2.
( ) ( ) ( ) ( )
1 2 3 1 , . 1f xy f x f y xy x y + =
Lời giải:
Trong (1) cho
1y =
có:
( ) ( ) ( )
1 2 0 1 .f f f x x + =
T đây suy ra
( )
0 0f =
nếu ngược li ta có
( )
f x c x=
(c hng s), không thỏa mãn đề bài.
Trong (1) cho
1, 1x y= =
ta có
( ) ( ) ( )
2
0 2 1 2 1 1f f f+ = =
(do
( )
1 0f
)
Trong (1) thay
x
bi
1x +
, cho
1y =
( ) ( ) ( )
2 1 3 2 . 2f x f x x x+ + = +
Trong (2) cho
1x =
( )
1 1f =
.
Trong (1) thay
x
bi
1x
, cho
1y =
( ) ( ) ( )
2 1 3 2 . 3f x f x x x = +
T (2),(3) suy ra
( ) ( )
.f x f x x=
Trong (1) cho
1y =
( ) ( ) ( ) ( ) ( )
1 2 3 1 1 2 3 1 . 4f x f x x f x f x x x + = + + = +
Tài liu chuyên đề bi ng hc sinh gii
10 | C CHUYÊN ĐÊ: PHƯƠNG TRÌNH HÀM
T (3),(4) suy ra
( )
.f x x x=
Th li thy tha mãn.
Bài 6: (Chuyên Sư phạm HN 2018) Tìm tt c các hàm
:f
tha mãn:
( ) ( ) ( )
( )
( ) ( ) ( )
1 0 , 1f x y f x f y x f y x x x y + + + =
Li gii:
Trong (1) cho
0x y= =
ta có:
( ) ( )
( )
( )
2
0 0
0 0
0 1
f
f f
f
=
=
=
Trường hp 1:
( )
0 0f =
.
Trong (1) cho
0x =
ta có:
( )
0 .f y y=
Th li thy không tha mãn.
Trường hp 2:
( )
0 1f =
.
Trong (1) cho
2y x=
ta có:
( ) ( ) ( )
1 .f x f x xf x x x = +
Trong (1) thay
x
bi
x
,thay
y
bi
2x
ta có:
( ) ( ) ( ) ( )
1 . 2f x f x xf x x x = +
T đây suy ra
( ) ( )
2 0f x f x x =
. Mà
( ) ( ) ( )
0 1 0 2 0f f f= =
nên
( ) ( ) ( )
2 . 3f x f x x =
T (2) và (3) được
( )
( )
( )
( )
1 1 0 .f x f x x x + =
Ga s tn ti
, 0a b
tha
( )
1f a =
( )
1f b b=
. Mt khác trong (1) cho
,x b y a b= =
( )
( )
1 1
0
1
1 1
0
b
b
f a b b
a b b
a
=
=
+ =
+ =
=
, vô lý.
Do đó
( )
1f x x=
hoc
( )
1 .f x x x= +
Th li thy tha mãn.
Bài 7: (Bc Ninh 2018) Tìm tt c các hàm
:f
tha mãn:
( ) ( )
( )
( ) ( )
2
, , 1xf x xy xf x f x f y x y+ = +
Li gii:
Trong (1) cho
0x y= =
ta có:
( )
0 0f =
Trong (1) cho
1x y= =
ta có:
( )
( )
1 1
1 0
f
f
=
=
Trường hp 1:
( )
1 0f =
.
Tài liu chuyên đề bi ng hc sinh gii
CHUYÊN ĐÊ 1: ĐẠI S VÀ GII TÍCH 11 |
Trong (1) cho
1y =
ta có:
( )
0 .xf x x=
Kết hp
( )
0 0f =
được
( )
0 .f x x=
Th li thy
tha mãn.
Trường hp 2:
( )
1 1f =
.
Trong (1) cho
1, 1x y= =
ta có:
( )
1 1f =
Trong (1) cho
1y =
ta có:
( )
( )
2
.f x xf x x=
Kết hp
( )
0 0f =
được
( ) ( )
.f x f x x=
Trong (1) cho
1x =
ta có:
( ) ( )
1 1 .f y f y y+ = +
Do
( )
( )
2
.f x xf x x=
nên (1) tr thành
( ) ( ) ( ) ( ) ( ) ( )
( )
( ) ( )
( ) ( ) ( ) ( )
1 1 ,
, 2
f x xy f x f x f y f x f y f x f y x y
f x f y f xy x y
+ = + = + = +
=
Suy ra
( ) ( ) ( ) ( )
, . 3f x xy f x f xy x y+ = +
Vi
0x
trong (3) thay
y
bi
y
x
được
( ) ( ) ( )
0, .f x y f x f y x y+ = +
Kết hp
( )
0 0f =
được
( ) ( ) ( ) ( )
, . 4f x y f x f y x y+ = +
T (2) và (4) kết hp
( )
1 1f =
được
( )
.f x x x=
Th li thy tha mãn. Vy bài toán có 2 nghim
hàm
( )
0,f x x=
hoc
( )
, .f x x x=
Bài 8: (VMO 2002). Hãy tìm tt cc các hàm s
( )
f x
xác định trên tp s thc và tha mãn h thc
( )
( )
( )
( )
2002
2001. . , ,f y f x f x y y f x x y =
(1)
Li gii:
Thế
( )
y f x=
vào (1) ta được
( ) ( )
( )
( )
( )
2
2002
0 202. ,f f x f x f x x=
(2)
Li thay
2002
y x=
vào (1) thì
( )
( )
( ) ( )
2002 2002
0 2001. . ,f x f x f x f x x =
(3)
Ly (2) cng với (3) ta được
( ) ( )
( )
2002
0,f x f x x x+ =
T đây suy ra với mi giá tr
x
thì ta có hoc là
( )
0f x =
hoc là
( )
2002
f x x=
. Ta s ch ra rằng để
tha mãn yêu cu bài toán thì bt buc phải có đồng nht
Tài liu chuyên đề bi ng hc sinh gii
12 | C CHUYÊN ĐÊ: PHƯƠNG TRÌNH HÀM
( )
0,f x x
hoc
( )
2002
,f x x x
.
Tht vy, vì
( )
0 0f =
trong c hai hàm s trên, nên không mt tính tng quát ta có th gi s tn ti
0a
sao
cho
( )
0f a =
và tn ti
0b
sao cho
( )
2002
f b b=
(vì ch cn thay
0x =
vào quan h (1) ta nhận được hàm
f
là hàm chẵn). Khi đó thế
x a=
y b=
vào (1) ta được
( )
( )
2002
f b f a b = +
Vy ta nhận được dãy quan h sau
( )
( )
( )
( )
( ) ( )
(
)
2002
2002
002 2002
2002 2002 2002
0
0 0 0
b
f b
f b
f a b
a b a b b
=
=
= +
=
+ +
Bng cách th li quan h hàm ban đầu ta kết lun ch có hàm s
( )
0,f x x
tha mãn yêu cu bài toán.
Bài 9: (Iran 1999, Trường hè toán hc M 2002) Xác định các hàm s
:f
tha mãn
( )
( )
( )
( )
2
4 , ,f f x y f x y yf x x y+ = +
Li gii:
Thế
2
y x=
ta được
( )
( )
( ) ( )
2 2
0 4 ,f f x x f x f x x+ = +
Thế
( )
y f x=
ta được
( ) ( )
( )
( )
( )
2
2
0 4 ,f f f x x f x x= +
Cộng hai phương trình trên ta được
( ) ( )
( )
2
4 0,f x f x x x =
.
T đây ta thấy vi mi
x
thì hoc là
( )
0f x
hoc là
( )
2
f x x=
. Ta chng minh nếu
f
tha mãn yêu
cu bài toán thì
f
phải đồng nht vi hai hàm s trên. Nhn thy
( )
0 0f =
, t đó thay
0x =
ta được
( ) ( )
,f y f y y=
, hay
f
là hàm chn. Gi s tn ti
0, 0a b
sao cho
( ) ( )
2
0,f a f b b= =
, khi đó
thay
,x a y b= =
ta được
( )
( )
( )
( )
2 2
.f b f a b f b f a b = + = +
T đó ta có quan hệ sau
Tài liu chuyên đề bi ng hc sinh gii
CHUYÊN ĐÊ 1: ĐẠI S VÀ GII TÍCH 13 |
( )
( )
( )
( ) ( )
(
)
2
2
2 2
2 2 2
0
0 (0 0)
b
f b
f b
f a b
a b a a b b
=
=
= +
=
+ +
Do đó xảy ra điu mâu thun. Th li thy hàm s
( )
0f x
,
( )
2
f x x=
tha mãn yêu cu.
Nhn xét:
1. Rõ ràng bài toán VMO 2002 có ý tưởng ging bài toán này.
2. Ngoài phép thế như trên thì bài toán này ta cũng có thể thc hin nhng phép thế khác nhau như:
a. Thế
( )
( )
2
1
.
2
y x f x=
b. Thế
0y =
để
( )
( )
( )
2
f f x f x=
, sau đó thế
( )
2
y x f x=
.
c. Thế
( )
y x f x=
và sau đó là
2
y x x=
.
Bài 10: Tìm tt c các hàm s
: \ 0f
sao cho vi mi
,x y
khác
0
ta có
( ) ( ) ( ).
x
f y f x f y f
x y
=
.
Li gii:
Đặt
1
( )g x f
x
=
ta được:
( ) ( ) ( ) 1
x
g y g x g y g
y
=
(1)
+ Cho
1y =
:
( )
(1) ( ) (1) 1g g x g g x =
. Suy ra
(1) (1) 1
x x
g g g g
y y
=
(2)
+ T (1) (2) suy ra
(1)
( ) ( ) ( ). ( ) ( ). (1)
(1)
x
g g
y
x
g y g x g y g y g g x g
g y
= =
(3), vi mi
, 0;x y x y
.
+ Trong (3) thay
x
bi
y x
, ta được:
( ) 1 ( ). (1)
x
g y g g y x g
y
=
(4).
+ T (1), (4) suy ra
( ) ( ) ( ). (1)g y g x g y x g =
. T đây suy ra
( ) ( ) ( ). (1)g u v g u g v g+ = +
(5), vi mi
, 0; 0u v u v +
.
+ T (3) suy ra
( ) (1) ( ). ( )g xy g g x g y=
vi mi
, 0x y
(6).
+ Hoán đổi vai trò ca u,v trong (5) suy ra nếu
(1) 1g
thì
( ) 0g x
(mâu thuẫn). Do đó
(1) 1g =
và ta
được:
( ) ( ) ( ); ( ) ( ). ( )g u v g u g v g uv g u g v+ = + =
vi mi
, 0u v
.
Tài liu chuyên đề bi ng hc sinh gii
14 | C CHUYÊN ĐÊ: PHƯƠNG TRÌNH HÀM
Theo kết qu bản ta được
( )g x x=
. Vy
1
( )f x
x
=
là hàm duy nht cn tìm.
4. Bài tập củng cố
Bài tập 1: Tìm tất cả các hàm số
:f
thoả mãn:
( )
( )
( )
,f f x y x f y+ = +
,x y
(1)
Li gii:
Giả sử hàm số
f
thoả mãn đề bài. Trong (1) thay
x
bởi
y
và thay
y
bởi
x
ta được:
( )
( )
( )
,f f x y y f x+ = +
,x y
(2)
T (1) và (2) suy ra:
( ) ( )
,x f y y f x+ = +
,x y
(3)
T (3) cho
0y =
ta được
,( )f x x c= +
x
( )
( )
0c f=
Thay (1) vào ta được
0c =
. Vy hàm s cn tìm là:
( )
,f x x=
x
Bài tp 2: Tìm tt c các hàm s
( ) ( )
: 0, 0,f + +
tho mãn:
( )
( )
2 2
y x
f x f y xf yf
= +
,
( )
, 0,x y +
(1)
Li gii:
Giả sử
f
là hàm số thoả mãn yêu cầu của đề bài.
Trong (1) cho
x y=
, ta được:
( )
2
2
2
x
f x xf
=
,
( )
0,x +
Suy ra:
( )
2
2 2
x f x
f
x
=
,
(0, ).x +
Thay vào (1) ta có:
( )
( )
( )
( )
2
2
2 2
f y
f x
f x f y x y
y x
= +
,
( )
, 0,x y +
( )
( )
2
0
2 2
x f y
y f x
y x
=
,
( )
, 0,x y +
( )
( )
2 2
x f y
y f x
y x
=
,
( )
,y 0,x +
( )
( )
;
f y
f x
x y
=
( )
( )
, 0,
f x
x y a
x
+ =
( )
0,x +
(
a
là hng s) thay
( )
,f x ax=
( )
0, .x +
Thay vào (1), đng nhất ta được:
0a =
hoc
1.a =
Vy có 2 hàm s tho mãn đề bài:
( )
0,f x =
( )
0,x +
( )
f x x=
,
( )
0, .x +
Bài tp 3: Tìm tt c các hàm s
:f
tho mãn điều kin:
( ) ( )
2 2
2 ( ),f x y f x y y x y+ = +
, .x y
Hướng dn:
Đặt
, .u x y v x y= + =
Đáp số:
( )
3
f x x c= +
,
.x
Bài tp 4: Tìm hàm
:f
tha mãn:
( ) ( ) ( )
1, 1f x xf x x x+ = +
.
Tài liu chuyên đề bi ng hc sinh gii
CHUYÊN ĐÊ 1: ĐẠI S VÀ GII TÍCH 15 |
Li gii:
Đặt
t x=
, ta được:
( ) ( ) ( )
1, 1f t tf t t t = +
. Ta có h:
( ) ( )
( ) ( )
( )
1
1
1
f x xf x x
f x
xf x f x x
+ = +
=
+ = +
. Th li hàm s cn tìm là:
( )
1f x =
.
Bài tp 5: Tìm hàm s
: \ 0,1f
Tha mãn:
( ) ( )
*
1
1 , 2
x
f x f x x
x
+ = +
.
Li gii:
Đặt
( ) ( ) ( )
1 1
1
, 2 1 .
x
x f x f x x
x
= + = +
Đặt
( ) ( ) ( )
1
2 1 2 1
1
1
1
, 2 1 .
1
x
x f x f x x
x x
= = + = +
Đặt
( ) ( ) ( )
2
3 2 2
2
1
, 2 1 .
x
x x f x f x x
x
= = + = +
Ta có h
( ) ( )
( ) ( )
( ) ( )
( )
1
1 2
2 1 1
3 2 2
1
1
1 1 1
1
2 2 1
1
f x f x x
x x x
f x f x x f x x
x x
f x f x x
+ = +
+ +
+ = + = = + +
+ = +
. Th li thấy đúng. Vậy
hàm s cn tìm có dng:
( )
1 1 1
2 1
f x x
x x
= + +
.
Bài tp 6: Tìm hàm s
: \ 1;0;1f
tha mãn:
( ) ( )
1
1, 1 3 .
1
x
xf x xf x
x
+ =
+
Li gii:
Đặt
( ) ( ) ( )
1 1
1
, 3 2 1.
1
x
x xf x f x
x
= + =
+
Đặt
( ) ( ) ( )
1
2 1 1 2
1
1
1
, 3 2 1.
1
x
x x f x f x
x x
= = + =
+
Đặt
( ) ( ) ( )
2
3 2 2 3
2
1
1
, 3 2 1.
1 1
x
x
x x f x f x
x x
+
= = + =
+
Đặt
( ) ( ) ( )
3
4 3 3
3
1
, 3 2 1.
1
x
x x x f x f x
x
= = + =
+
Tài liu chuyên đề bi ng hc sinh gii
16 | C CHUYÊN ĐÊ: PHƯƠNG TRÌNH HÀM
Ta có h
( ) ( )
( ) ( )
( ) ( )
( ) ( )
( )
( )
1
2
1 1 2
2 2 3
3 3
2 1
2 1
4 1
.
5 1
2 1
2 1
xf x f x
x f x f x
x x
f x
x x
x f x f x
x f x f x
+ =
+ =
+
=
+ =
+ =
Th li thấy đúng.
Vy hàm s cn tìm là:
( )
( )
2
4 1
.
5 1
x x
f x
x x
+
=
Bài tp 7: Tìm tt các các hàm s
:f
thỏa mãn điều kin
( ) ( ) ( )
2 cos , ,f x y f x y f x y x y+ + =
Hướng dn:
1. Thế
2
y
2. Thế
2
y y
+
3. Thế
0x
Đáp số:
( ) ( )
cos sin , , .f x a x b x a b x= +
Bài tp 8: Cho hàm s
:f
thỏa mãn điều kin
( ) ( ) ( ) ( )
, ,f xy x y f xy f x f y x y+ + = + +
. Chng
minh rng:
( ) ( ) ( )
, ,f x y f x f y x y+ = +
Hướng dn:
1. Tính
2. Thế
1y =
. Chng minh
f
là hàm s
3. Thế
( ) ( )
1 2 1 2 1y f x f x= + = +
4. Tính
( )
( )
2 1f u v uv+ + +
theo (3) và theo gi thiết để suy ra
( ) ( ) ( )
2 2f uv u f uv f u+ = +
5. Cho
1
,
2 2
y
v x=
,2u y uv x
để suy ra điều phi chng minh
Bài tp 9: Tìm tt c các hàm s
:f
đồng thi thỏa mãn các điều kin sau:
( ) ( )
( ) ( ) ( ) ( ) ( )
1 , 0
1 , , , , 0,0 ; 0
f x xf x x
f x f y f x y x y x y x y
=
+ = + + +
Hướng dn:
1. Tính
( ) ( )
0 , 1f f
Tài liu chuyên đề bi ng hc sinh gii
CHUYÊN ĐÊ 1: ĐẠI S VÀ GII TÍCH 17 |
2. Tính
1a +
vi
( )
1 1
1 1
1 1
x
a f f f x
x x
+
= = = +
+ +
theo c hai điều kin.
Đáp số:
( )
1, .f x x x= +
Bài tp 10: Tìm tt các các hàm s
*
:f
thỏa mãn điều kin:
( )
*
1
2 3 ,f z f x x
y
+ =
Hướng dn:
Thế
1
x
x
Đáp số:
( )
*
2
, .f x x x
x
=
Bài tp 11: Tìm tt c các hàm s
: \ 0,1f
thỏa mãn điều kin:
( )
1
2 , \ 0,1
x
f x f x x
x
+ =
Hướng dn:
Thế
1 1
,
1
x
x x
x x
Đáp số:
( )
1 1
, \ 0;1
1
x
f x x x
x x
= +
Bài tp 12: (Belarus 1995) Tìm tt c các hàm s
:f
tha mãn:
( )
( )
( ) ( ) ( )
f f x y f x y f x f y+ = + +
Li gii:
Rõ ràng
f
khác hng s.
0y =
vào điều kiện bài toán ta đưc
( )
( )
( )
( )
( )
1 0 ,f f x f f x x= +
Trong đẳng thc trên thay
x
bi
x y+
thì
( )
( )
( ) ( )
( )
( ) ( ) ( )
1 0 ,f f x y f f x y f x y f x f y xy+ + = + = + +
Đơn giản ta được
( ) ( ) ( ) ( )
0 .f f x y f x f y xy+ =
(7)
Thay
1y =
vào (7) thì
Tài liu chuyên đề bi ng hc sinh gii
18 | C CHUYÊN ĐÊ: PHƯƠNG TRÌNH HÀM
( )( ) ( ) ( )
0 1 1f x f x f x+ =
.
Li thay
1y =
vào
x
bi
1x +
vào (7) ta có
( ) ( ) ( ) ( )
0 . 1 . 1 1f f x f x f x= + + +
.
Kết hp hai đẳng thức trên ta được
( )
( )
( ) ( ) ( ) ( ) ( )
( )
( )
2
0 1 1 0 1 0f f f f x f f f x f = +
.
Nếu
( )
( )
( ) ( )
2
0 1 1 0f f f =
, thì thay
0x =
vào phương trình cuối cùng ta được
( )
0 0f =
, nên theo (7) thì
( ) ( )
f x f y xy=
. Khi đó
( ) ( )
1 ,f x f x x=
, điu này dẫn đến
( )
( )
( ) ( )
2
0 1 1 1f f f =
, mâu thun.
Vy
( )
( )
( ) ( )
2
0 1 1 0f f f
, suy ra
( )
f x
là một đa thức bc nht nên có dng
( )
f x ax b= +
. Thay vào
quan h hàm ban đầu suy ra
1, 0a b= =
. Vy hàm s tha mãn yêu cu bài toán là
( )
,f x x x=
.
Nhn xét: Nếu chu khó tính ta s tính được
( )
0 0f =
bng cách thế các biến
,x y
bi hai s 0 và 1.
Bài tp 13: Cho hàm s
:f
thỏa mãn điều kin:
2 2
( ) ( ) ( ) ( ) ( )x y f x y x y f x y xy x y + + =
vi
,x y
.
i) Tính
(0)f
và chứng minh
f
là hàm số lẻ.
ii) Tìm tất cả các hàm số
f
.
Li gii:
i) Tính
(0)f
và chứng minh
f
là hàm số lẻ.
Vi
1x y= =
thì
2 (0) 0f =
hay
(0) 0f =
. (1)
y y
Vi
0y =
thì
(0) ( 0) 0f f= =
(do (1))
Vi
0
0
x
y
=
thì
( ) ( ) 0yf y yf y =
hay
( ) ( )f y f y =
Vy
f
là hàm s l.
ii) Tìm tất cả các hàm số
f
.
Đặt
u x y
v x y
= +
=
ta suy ra
2
2
u v
x
u v
y
+
=
=
Khi đó
2 2
( ) ( )
4
u v
v f u u f v uv
=
(2)
Tài liu chuyên đề bi ng hc sinh gii
CHUYÊN ĐÊ 1: ĐẠI S VÀ GII TÍCH 19 |
Vi
0
0
u
v
T (2) ta được
2 2
( ) ( )
4
f u f v u v
u v
=
hay
2 2
( ) ( )
4 4
f u u f v v
u v
=
.
Chn
1v =
, ta có
2
( ) 1
(1)
4 4
f u u
f
u
=
.
Đặt
1
(1)
4
a f=
Ta có
2
( )
, 0
4
f u u
a u
u
=
. (3)
Suy ra
3
( ) , 0
4
x
f x ax x= +
T (1), (3) ta được
3
( ) ,
4
x
f x ax x= +
.
Th li
Vi
3
( )
4
x
f x ax= +
, ta có:
1 1
(0) 0; (1) (1)
4 4
f f a a f= = + =
2 2
( ) ( ) ( ) ( ) ( )x y f x y x y f x y xy x y + + =
Vy
3
( ) ,
4
x
f x ax x= +
.
Bài tp 14: Tìm tt c các hàm s
:f
tho mãn:
( ) ( ) ( ) ( )
1 1 1
, , ,
3 3 9
f xy f xz f x f yz x y z+
.
Li gii:
Cho
0x y z= = =
thì
( ) ( ) ( ) ( ) ( )
2
2
1 1 1 1 1
0 0 0 0 0 0
3 3 9 3 3
f f f f f
+ =
Cho
1x y z= = =
thì
( ) ( ) ( ) ( ) ( )
2
2
1 1 1 1 1
1 1 1 1 0 1
3 3 9 3 3
f f f f f
+ =
Cho
0y z= =
thì
( ) ( ) ( )
2 1
0 0
3 9
f f x f
.
Do
( )
1
0
3
f =
nên
( )
1
,
3
f x x
. (1)
Tài liu chuyên đề bi ng hc sinh gii
20 | C CHUYÊN ĐÊ: PHƯƠNG TRÌNH HÀM
Cho
1y z= =
, ta có
( ) ( ) ( ) ( )
1 1 1
1
3 3 9
f x f x f x f+
.
Do
( )
1
1
3
f =
nên
( )
1
,
3
f x x
. (2)
T (1) và (2) ta đưc
( )
1
,
3
f x x=
.
Bài tp 15: Tìm tt c các hàm s
:f
tho mãn
( )
( ) ( )
2 2
, , (1).f x y xf x yf y x y+ = +
Li gii:
Cho
0x =
, t
( )
1
suy ra
( )
( )
2
,f y yf y y=
Cho
0y =
, t
( )
1
suy ra
( )
( )
2
,f x xf x x=
.
Do đó (1) trở thành:
( ) ( ) ( )
( ) ( ) ( ) ( )
2 2 2 2
, , , , 0 *f x y f x f y x y f x y f x f y x y+ = + + = +
thay
y
bi
y
t
( )
1
ta được:
( )
( ) ( ) ( ) ( ) ( ) ( )
2 2
, ,f x y xf x yf y yf y yf y y f x f x x+ = = =
( ) ( ) ( ) ( )
, ,yf y yf y y f x f x x = =
, chng t
f
là hàm s l.
Do đó với mi
0, 0x y
ta có
( ) ( )
( )
( ) ( ) ( ) ( )
( ) ( ) ( )
( )
( )
( ) ( )
( ) ( ) ( ) ( )
, 0, 0 **
f x y f x y f x f y f x f y
f x f x y f y
f x y y f x y f y
f x y f x f y x y
= + = + =
= +
+ = +
+ = +
Vi mi
0, 0x y
ta có
( ) ( ) ( ) ( )
( )
( ) ( )
( )
( ) ( ) ( )
***f x y f x y f x f y f x f y f x f y+ = = + = = +
Kết hp
( ) ( )
* , ** ,(***)
và ta được
( ) ( ) ( )
, ,f x y f x f y x y+ = +
.
tính
( )
( )
2
1f x +
theo hai cách. Ta có
Tài liu chuyên đề bi ng hc sinh gii
CHUYÊN ĐÊ 1: ĐẠI S VÀ GII TÍCH 21 |
( )
( )
( )
( ) ( )
( )
( ) ( )
( ) ( ) ( )
( )
( ) ( ) ( )
( ) ( )
( )
2
2
2
1 2 1
1 1 2 1
1 1 2 1
1 ,
, ,
f x f x x
x f x f x f x f
x f x f xf x f x f
f x xf x
f x ax x a
+ = + +
+ + = + +
+ + = + +
=
=
Bài tp 16: (Đồng Nai 2015) Tìm tt c các hàm
:g
tha mãn:
( ) ( ) ( ) ( ) ( )
( ) ( )
2 2 2 2 2 2
2 4
2 2
2 , , , .
g g x x yz g x g x x yz z y g y y z g z
x yz x g y g z x x y z
+ = + + +
+ + +
Li gii:
Đặt
( ) ( )
2
f x g x x=
ta được:
( )
( )
( ) ( ) ( )
, , , . 1f f x yz x f y f z x y z+ = +
Trong (1) cho
0x y= =
được:
( )
( )
( ) ( )
0 0 , .f f f f z z=
T đây ta có
( )
0 0f =
nếu ngược li
( )
0 0f
thì
f
là hàm hng không tha mãn.
Trong (1) cho
0y =
ta được
( )
( )
( )
, . 2f f x x x=
Suy ra
f
là toàn ánh.
Trong (1) cho
0x =
ta được
( ) ( ) ( ) ( )
, , . 3f yz f y f z y z=
T (2) và (3) phương trình đã cho đươc viết li
( )
( )
( )
( )
( ) ( )
, , , . 4f f x yz f f x f yz x y z+ = +
Do
f
toàn ánh nên t (4) suy ra
( ) ( ) ( )
, , .f x y f x f y x y+ = +
Vì f va cng tính va nhân tính nên
( )
, .f x ax x=
Th li (1) có
( ) ( )
2
1 , ,a f x x x g x x x x= ⎯⎯ = = +
.
Bài tp 17: Tìm tt c các hàm s
:f
tha mãn:
1 1
( ( )) ( ) ( ) , ,
2 2
f x xy f y f x f y x y
+ + = + +
.
Li gii:
D thy hàm
f
hng không tha mãn. Ta xét
f
không hng.
1 1
( ( )) ( ) ( ) , , (1)
2 2
f x xy f y f x f y x y
+ + = + +
Trong (1) cho
1y =
ta được:
1 1
( ( 1)) ( ) ( 1) , (2)
2 2
f f f x f x
= + +
Tài liu chuyên đề bi ng hc sinh gii
22 | C CHUYÊN ĐÊ: PHƯƠNG TRÌNH HÀM
Rõ ràng nếu
1
( 1) 0
2
f +
thì
f
là hàm hằng. Do đó:
1 1
( 1) 0 ( 1)
2 2
f f + = =
Ta s chng minh:
( )
1
0 1
2
f x x+ = =
.
Tht vy, gi s tn ti
1a
sao cho
1
( )
2
f a
=
.
Trong (1) chn
y a=
ta có:
1
0,
2
f ax x x
+ =
.
Mâu thun vì
f
không là hàm hằng. Do đó ta có:
1a =
.
Chú ý là
( )
1
1
2
f =
nên t (2) ta:
1
0
2
f
=
.
Trong (1) chn
1
( )
2
, ( 1)
1
f y
x y
y
=
+
ta được:
( ) ( )
( )
( )
( )
( )
( )
1 1 1
1 1
2 2 2
.
1 1 1 2 2
1
1 1 1
2
0, 1
1 2 2 2
f y f y f y
f y f y f f y
y y y
f y
f f y f y
y
+ + = + +
+ + +
+ + = =
+
( ) ( )
1 1
1
2 2
, 1 1, 1
1 2 1
f y f y
f y y
y y
= =
+ +
Suy ra
( )
1
, 1
2
f y y y= +
Do
( )
1
1
2
f =
nên
( )
1
,
2
f x x x= +
.
Th li ta có hàm s cn tìm là
( )
1
,
2
f x x x= +
.
Bài tp 18: Tìm tt c các hàm s
:f
tho mãn
( ( ) ) ( ) ( ) 2013f xf y y f xy x f x y xy+ + + = + +
(1).
Li gii:
Tài liu chuyên đề bi ng hc sinh gii
CHUYÊN ĐÊ 1: ĐẠI S VÀ GII TÍCH 23 |
Thay
1
1,
2013
x y= =
vào (1) ta được:
( ) 1f a =
Trong đó
1 1
2013 2013
a f
= +
Tiếp tc thay
vào (1), ta thu được:
( ) ( ) ( ) 2013 , .f x a f xa x f x a ax x+ + + = + +
hay
( ) 2013 , .f ax x ax x+ =
(2)
T (2) suy ra
1a
. Thay
1
t
x
a
=
+
vào (2), ta được
2013
( ) , .
1
at
f t t
a
=
+
hay
( ) ,( )f t ct c R=
Tiếp theo, thay biu thc ca
( )
f t
vào (1), ta thu được đẳng thc
2
2013 ; , .c xy cy cxy cx cx cy xy x y+ + + = + +
2
2
( 2013) 0, ,
1 8053
2
2013 0
1 8053
2
c c xy x y R
c
c c
c
+ =
=
+ =
+
=
Vy ta nhận được hai hàm s tho mãn đề bài là
1 8053
( )
2
f x x
+
=
1 8053
( )
2
f x x
=
Bài tp 19: Tìm tt c các hàm
:f
tha mãn:
( ) 2015 2015 ( )f xf y x xy f x+ = +
vi mi x, y.
Li gii:
Cho
1x =
thì
( ) 2015 2015 (1)f f y y f+ = +
Chn
y
tha mãn
2015 (1) 2015y f+ =
, và đặt
(y) 2015t f= +
thì
(t) 2015f =
.
Chn
y t=
, và thay vào gi thiết thì:
(t) 2015x 2015 ( )f xf xt f x+ = +
Hay:
(0) 2015 ( ) ( ) 2015 (0), .f xt f x f x tx f x= + = +
Vy
( )f x
là hàm bc nht.
Gi s
( )f x mx n= +
. Thay vào gi thiết ta có:
( )
2
( ) 2015 2015 ( ) 2015 2015
2015 2015
m xf y x n xy mx n mx my n mx n xy mx n
m xy mn m x xy mx
+ + = + + + + + = + +
+ + = +
Tài liu chuyên đề bi ng hc sinh gii
24 | C CHUYÊN ĐÊ: PHƯƠNG TRÌNH HÀM
Đẳng thức trên đúng với mi x, y nên:
2
2015
2015
2015
2014
m
m
mn m m
n
=
=
+ =
=
Vy có 2 hàm tha mãn yêu cu,
( ) 2015 2014f x x=
.
Bài tp 20: Tìm hàm s
:f
tha mãn
( )
( )
( ) ( )
( )
2
2
2
2 ; , .f x y x yf x f y x y = +
Li gii:
Cho
( ) ( )
( )
( )
( )
2
0 0
0 0 0
0 1
f
x y f f
f
=
= = =
=
+) Nếu
( )
0 0f =
. Cho
0,y x=
ta được:
( )
( )
2 2
, 0f x x f t t t= =
Cho
x y=
ta được
( ) ( ) ( )
( )
( )
( )
( )
2 2
2
0 2 0f x xf x f x f x x f x x= + = =
. Thử lại thấy đúng
+) Nếu
( )
0 1f =
cho
0,y x=
ta được
( )
( )
2 2
1 1, 0f x x f t t t= + = +
.
Cho
0,x y=
ta được
( ) ( )
( )
( )
( )
( )
( )
( )
( )
2
2
2 2 2 2
1
2 2 2 1 1
1
f y y
f y y f y f y f y y y y y
f y y
= +
= + = + = + + = +
=
Giả sử tồn tại
0
y
sao cho
( )
0 0
1f y y=
Chọn
0
x y y= =
ta được:
( ) ( )
( )
( )
( )
2
0 0
2
0 0 0 0
0 0
1
1 2
1
f y y
y y f y f y
f y y
= +
= +
=
Nếu
( )
( )
0
0 0 0 0
0
1 1 1
0 1
y
f y y y y
f
=
= =
=
(loại)
Nếu
( ) ( )
0 0 0 0 0
1 1 1 1 1 0f y y y y y f= + = + = =
Vậy
( )
1, .f y y y= +
.
Bài tp 21: Tìm tt c các hàm s
:f
+ +
tha mãn:
( ( ) 2 ) 2 (2 ( ))f x f x y x f f y+ + = +
,
; .x y
+
Li gii:
Gi s tn ti hàm s
( )
f x
tha mãn yêu cu bài ra.
Đặt
(0)f a=
vi
a
+
Chn
0x =
;
y x=
, thay vào (14) ta được
( (0) 2 ) (2 ( (2 ))) (2 ( ))f f x f f x f x f xx a f x
+ +
+ = + =
,
Nên
( ( ) 2 ) 2 (2 )f x f x y x f y a+ + = + +
,
;x y
+
(i)
Tài liu chuyên đề bi ng hc sinh gii
CHUYÊN ĐÊ 1: ĐẠI S VÀ GII TÍCH 25 |
Thay
2y
bi
y
ta được
( ( ) ) 2 ( )f x f x y x f y a+ + = + +
,
;x y
+
(ii)
Vi
;x y a
tha mãn
( ) ( )f x f y t= =
Thay
y
bi
y a
vào (ii) ta được:
( ) 2f x t y a x t+ + = +
,
;x y
+
Thay
x
bi
,y y
bi
x a
vào (ii) ta được:
( ) 2f y t x a y t+ + = +
,
;x y
+
Do đó
Chn
0; 0x y= =
thay vào (i) ta có:
( ) (2 )f a f a=
Theo kết qu phn trên suy ra
2a a=
Suy ra
0a =
Chn
0;x y x= =
, thay vào (i) ta được
(2 ) (2 ( ))f x f f x=
,
x
+
Suy ra:
2 ( ) 2f x x=
,
x
+
( )f x x =
,
x
+
Th li thy hàm s va tìm tha mãn yêu cu bài toán.
Vy
( )f x x=
,
x
+
là hàm s cn tìm.
Bài tp 22: Tìm tt c các hàm s
:f
thỏa mãn điều kin
2 2
( ) ( ). ( ) , , .f x f x y f x y y x y= + +
Li gii:
Cho
0x y= =
ta được
2
(0) 0
(0) (0)
(0) 1
f
f f
f
=
=
=
Cho
2x y= =
ta được
(4) (4). (0) 4 (0) 1f f f f= +
Vy
( )
0 0.f =
Cho
ta được
2 2 2
( ) (2 ). (0) ( ) , 0.f x f x f x x f t t t= + = =
Cho
0, 0x y t= =
, ta được
2 2
(0) ( ) ( ) 0 ( ) ( ) , 0f f t f t t tf t t f t t t= + = + =
Vy
( )
, .f x x x=
Th li ta thy hàm s
( )
, .f x x x=
tha mãn bài toán.
Bài tp 23: Tìm
( )
f x
xác định
x
tha mãn
( )
( ) ( ) ( ) ( )
0 2013; 2014
2
2 cos , . 1
f f
f x y f x y f x y x y
= =
+ + =
Tài liu chuyên đề bi ng hc sinh gii
26 | C CHUYÊN ĐÊ: PHƯƠNG TRÌNH HÀM
Li gii:
Trong (1) cho
0; 0 2014
2 2 2 2 2
x y f f f f
= = + = = =
Trong (1) cho
0
2 2 2 2 2
y f x f x f x f x
= + + = = +
Trong (1) cho
2 cos 4028cos
2 2 2 2
x f y f y f y y
= + = =
Trong (1) cho
( )
4028cos . 2
2 2
y x f x f x x
= + =
Trong (1) cho
( ) ( ) ( )
0 2 0 cos 4026cosx f y f y f y y= + = =
Trong (1) cho
( )
4026cos 4026sin . 3
2 2 2 2
y x f x f x x x
= + + + = + =
Tr tng vế hai phương trình (2) và (3) được:
2 4028cos 4026sin 2014cos 2013sin
2 2
f x x x f x x x
= =
Trong (1) thay
x
bi
2
x
+
có:
( )
2014cos 2013sin 2014sin 2013cos
2 2
f x x x x x
= + + =
Bài tp 24: Tìm các hàm s
( )
: 1;f +
tho mãn điều kin:
( ) ( ) ( ) ( )
f x f y y x f xy =
vi mi
, 1x y
Li gii:
Vi mi
1t
, thay
( ) ( ) ( )
; ;2 , ;4x y t t=
( )
2 ;2t
vào (1) ta được:
( ) ( ) ( ) ( )
( ) ( ) ( ) ( )
( ) ( ) ( ) ( )
2 2 2
4 4 4
2 2 2 2 4
f t f t f t
f t f t f t
f t f t f t
=
=
=
Tài liu chuyên đề bi ng hc sinh gii
CHUYÊN ĐÊ 1: ĐẠI S VÀ GII TÍCH 27 |
( ) ( ) ( ) ( ) ( )
4 3 2 2 5 4f t f t t f t + =
(2), vi mi
1.t
Lấy
( ) ( )
5 1
4 2
2 2
t f f= =
Thay vào (2) ta được:
( ) ( ) ( )
5
2 2 5 4
2
t f t t f t
=
Do đó với mi
( )
( )
2
5
1, 4
2 2
f
t t f t
t
=
T (1) ta có:
( ) ( ) ( ) ( )
( )
2 2
4 4 4
f
f t f t f t
t
= + =
vơi
5
1, .
2
t t
Vi
5
2
t =
, t (1) thay
5
, 2
2
x y= =
ta có:
( ) ( )
( ) ( )
( )
( )
4 2 2 2 2 2
5 1
2 5 , 1.
5
2 2 5
2
f f f
f f f f t t
t
= = = ⎯⎯ =
Đặt
( ) ( )
2 2
c
c f f x
x
= =
vi
1x
.
Th li thỏa mãn điều kin (1).
Vậy hàm số cần tìm là:
( )
, 1.
c
f x x
x
=
.
Bài tp 25: Tìm tt c các hàm s
:f
tho mãn
( )
( ) ( )
2 2
, , (1).f x y xf x yf y x y+ = +
Li gii:
Cho
0x =
, t
( )
1
suy ra
( )
( )
2
,f y yf y y=
Cho
0y =
, t
( )
1
suy ra
( )
( )
2
,f x xf x x=
.
Do đó (1) trở thành:
( ) ( ) ( )
( ) ( ) ( ) ( )
2 2 2 2
, , , , 0 *f x y f x f y x y f x y f x f y x y+ = + + = +
thay
y
bi
y
t
( )
1
ta được:
( )
( ) ( ) ( ) ( ) ( ) ( )
2 2
, ,f x y xf x yf y yf y yf y y f x f x x+ = = =
( ) ( ) ( ) ( )
, ,yf y yf y y f x f x x = =
, chng t
f
là hàm s l.
Do đó với mi
0, 0x y
ta có
Tài liu chuyên đề bi ng hc sinh gii
28 | C CHUYÊN ĐÊ: PHƯƠNG TRÌNH HÀM
( ) ( )
( )
( ) ( ) ( ) ( )
( ) ( ) ( )
( )
( )
( ) ( )
( ) ( ) ( ) ( )
, 0, 0 **
f x y f x y f x f y f x f y
f x f x y f y
f x y y f x y f y
f x y f x f y x y
= + = + =
= +
+ = +
+ = +
Vi mi
0, 0x y
ta có
( ) ( ) ( ) ( )
( )
( ) ( )
( )
( ) ( ) ( )
***f x y f x y f x f y f x f y f x f y+ = = + = = +
Kết hp
( ) ( )
* , ** ,(***)
và ta được
( ) ( ) ( )
, ,f x y f x f y x y+ = +
.
tính
( )
( )
2
1f x +
theo hai cách. Ta có
( )
( )
( )
( ) ( )
( )
( ) ( )
( ) ( ) ( )
( )
( ) ( ) ( )
( ) ( )
( )
2
2
2
1 2 1
1 1 2 1
1 1 2 1
1 ,
, ,
f x f x x
x f x f x f x f
x f x f xf x f x f
f x xf x
f x ax x a
+ = + +
+ + = + +
+ + = + +
=
=
Bài tp 26: Tìm tt c các hàm s
:f
sao cho:
,x y
,
2015 2015
( 2014 ) (2 ) (3 2013 ) 5 2015.f x y f x y f x y x x+ = + + + +
Li gii:
Đặt
( ) ( ),f x x g x x =
.Thay vào gi thiết ta có
2015
( 2014 ) (2 ) (3 2013 ) 2015 , (1).g x y g x y g x y x y+ = + + + +
Thay
2015
3y x x=
vào (1) ta có
2015
(4 ), (2).g x x x
Xét hàm s
2015
(x) 3h x x x=
ta có
( )
h x
liên tc trên
lim ( ) ; lim ( )
x x
h x h x
→+ →−
= − = +
, suy ra
tp giá tr ca
( )
h x
. T (2) suy ra
( ) 2015 .g x x=
Do đó
( ) 2015, .f x x x=
.
Thay vào th li ta thy
( ) 2015 .f x x x=
tha mãn.
Vy
( ) 2015 .f x x x=
.
Bài tp 27: Xác định hàm s
:f
thỏa mãn đồng thời 3 điều kin:
1.
( ) ( )
f x f x =
;
2.
( ) ( )
1 1 ,f x f x x+ = +
3.
( )
2
1
, 0
f x
f x
x x
=
.
Tài liu chuyên đề bi ng hc sinh gii
CHUYÊN ĐÊ 1: ĐẠI S VÀ GII TÍCH 29 |
Li gii:
Vi mi
x
ta
:
( )
( )
2
1 1 1
1 1 1 1
f x
x
f f f
x x x x
+
= + = + = +
Mt khác, vi mi
x
khác
0; 1
ta có:
( )
( )
( ) ( )
( )
( ) ( ) ( ) ( )
2 2
2
2
2 2
2 2
2
2
2 2
1 1 1 1 1
1
. 1
1 1
1
1
1 1 1
1 1
. 1 .
1 1
1 1
1 1 2 2
x
f
x x x x
x
f f f f
x
x x x x x
x
x
x
f x x f x
x x
x x
x x
x f x x x f x
x x
+ + +
+
= = = =
+ +
+
+
+ + +
+ +
= =
+ +
= + = +
T
( )
1
( )
2
ta có:
( )
( ) ( )
2
2 2
1
1 2 , \ 0; 1 .
f x
x x f x f x x x
x x
+ = + ⎯⎯ =
T (1) có
( )
0 (0)f f=
suy ra
( )
0 0f =
Ta có
( ) ( ) ( )
1 1 1 0 1f f f
= = + =
. Vy
( ) , .f x x x=
.
Bài tp 28: Tìm tt c các hàm s
:f
tha mãn:
( ) ( )
( )
( ) ( )
2
, , . 1xf x xy xf x f x f y x y+ = +
.
Li gii:
Trong (1) lấy
0x y= =
được
( )
0 0.f =
Trong (1) lấy
1y =
ta có
( )
( )
( ) ( ) ( )
2
1 0 0, 2xf x f x f xf x+ = =
Trong (2) lấy
1x =
ta được:
( ) ( ) ( )
( )
( )
1 0
1 1 1 0
1 1
f
f f f
f
=
=
=
+ Nếu
( )
1 0f =
thì từ (2) suy ra
f
đồng nhất
0
và hàm này thỏa mãn bài toán.
+ Nếu
( )
1 1f =
thì trong (2) lại lấy
1x =
ta thu được
( )
1 1f =
.
Từ đó (2) trở thành:
( )
( ) ( )
2
, 3f x xf x x=
Trong (1) ta cho
1:y =
( ) ( )
( )
( ) ( ) ( )
2
2 1 , 2 2 , 0xf x xf x f x f x f x f x x= + =
Kết hợp (1) và (3) ta được:
( ) ( ) ( ) ( ) ( )
, , 0 4f x xy f x f x f y y x+ = +
Từ (4) lần lượt lấy
1, 1x x= =
ta có:
( ) ( )
( ) ( )
1 1 ,
1 1 ,
f y f y y
f y f y y
+ = +
=
Tài liu chuyên đề bi ng hc sinh gii
30 | C CHUYÊN ĐÊ: PHƯƠNG TRÌNH HÀM
Như vậy hàm
f
là một hàm số lẻ.
Trong (4) thay
y
bởi
y
và sử dụng tính lẻ của hàm
:f
( ) ( ) ( ) ( ) ( ) ( ) ( ) ( )
, , 0 5f x xy f x f x f y f x f x f y y x = + =
Cộng vế theo vế (4) và (5) :
( ) ( ) ( ) ( )
2 2 , , 0f x xy f x xy f x f x y x+ + = =
( )
0 0f =
nên ta
( ) ( ) ( ) ( )
2 2 , ,f x xy f x xy f x f x x y+ + = =
( ) ( ) ( )
, ,f x y f x f y x y+ = +
Và bây giờ ta sẽ tính biểu thức
( )
( )
2
1f x +
theo hai cách:
( )
( )
( ) ( )
( ) ( ) ( ) ( )
2
2 2
1 2 1 2 1 2 1,f x f x x f x f x f xf x f x x+ = + + = + + = + +
( )
( )
( ) ( ) ( ) ( )
( )
2
1 1 1 1 1 ,f x x f x x f x x+ = + + = + +
Từ hai điều trên thu được:
( ) ( ) ( ) ( )
( )
( )
2 1 1 1 , ,xf x f x x f x x f x x x+ + = + + =
Th li tha. Kết lun ca bài toán là:
( ) ( )
0, ; ,f x x f x x x= =
.
II. Sử dụng tính chất ánh xạ để giải phương trình hàm
1. Nhc li mt s khái nim và tính cht ca ánh x
1.1. Ánh x
Định nghĩa 1. Mt ánh x
f
t tp
X
đến tp
Y
là mt quy tắc đặt tương ứng mi phn t
x
ca
X
vi mt và ch mt phn t ca
Y
. Phn t này được gi nh ca
x
qua ánh x
f
và được ký hiu là
( )
f x
.
Tp
X
gi là tập xác định ca
f
. Tp
Y
gi là tp giá tr ca
f
Ánh x t
X
đến
Y
được ký hiu
( )
:f X Y
x y f x
=
Khi
X
Y
là các tp s thc, ánh x
f
được gi là mt hàm s xác định trên
X
Cho
,a X y Y
. Nếu
( )
f a y=
thì ta nói
y
nh ca
a
a
là nghch nh ca
y
qua ánh x
f
.
Tp hp
( )
,Y y Y x X y f x= =
gi là tp nh ca
f
. Nói cách khác, tp nh
( )
f X
tp
hp tt c các phn t ca
Y
mà có nghch nh.
1.2. Đơn ánh, toàn ánh, song ánh
Định nghĩa 2. Ánh x :
:f X Y
được gọi là đơn ánh nếu vi,
,a X b Y
a b
thì
( ) ( )
f a f b
, tc là hai phn t phân bit s có hai nh phân bit.
Tài liu chuyên đề bi ng hc sinh gii
CHUYÊN ĐÊ 1: ĐẠI S VÀ GII TÍCH 31 |
T định nghĩa ta suy ra ánh x
f
là đơn ánh khi và ch khi vi ,
,a X b Y
( ) ( )
f a f b=
, ta phi
a b=
.
Định nghĩa 3. Ánh x
:f X Y
được gi là toàn ánh nếu vi mi phn t
y Y
đu tn ti mt phn
t
x X
sao cho
( )
y f x=
. Như vậy
f
là toàn ánh nếu và ch nếu
( )
Y f X=
.
Định nghĩa 4. Ánh x
:f X Y
được gi là song ánh nếu nó vừa là đơn ánh vừa là toàn ánh. Như vậy
ánh x
:f X Y
là song ánh nếu và ch nếu vi mi
y Y
, tn ti và duy nht mt phn t
x X
để
( )
y f x=
.
1.3. Ánh x ngược ca mt song ánh
Định nghĩa 4. Ánh x ngược ca
f
, được kí hiu bi
1
f
, là ánh x t
Y
đến
X
gán cho mi phn t
y Y
phn t duy nht
x X
sao cho
( )
y f x=
. Như vậy
( ) ( )
1
f x y f x y
= =
Chú ý. Nếu
f
không phi là song ánh thì ta không th định nghĩa được ánh x ngược ca
f
. Do đó chỉ
nói đến ánh x ngược khi
f
là song ánh.
1.4. Ánh x hp
Định nghĩa 5. Nếu
:g A B
:f B C
( )
g A B
thì ánh x hp
:f g A C
được xác định
bi
( )( ) ( )
( )
f g a f g a=
Kí hiu
...
n
n
p p p p=
Để có th khai thác tính cht ca ánh x cn nm vng các khái nim v đơn ánh, toàn ánh, song ánh.
2. Các ví d:
2.1. Sử dụng tính đơn ánh giải phương trình hàm
Trong phương trình một vế có cha
( )
f x
, vế còn li cha biến
x
bên ngoài thông thường
f
đơn ánh.
Nếu trong phương trình chứa
( ) ( )
;( )f f x x y
+
hoc
( ) ( )
;( )f f y x y
+
, trong đó
( )
;x y
mt biu thức đối xng ca
x
y
thì ta thường chng minh
f
là đơn ánh.
f
đơn ánh trên
D
,a b D
sao cho
( ) ( )
f a f b=
thì
.a b=
Nếu hàm
f
đơn điệu thc s trên
D
thì
f
đơn ánh trên
.D
Ví dụ 1: Tìm hàm số
xác định trên thoả mãn:
( ( ) 2 ) 4 4 3,f f x y x y+ = + +
, .x y
Li gii:
Nhận xét
( )
f x
là đơn ánh.
Thật vậy, giả sử
1 2
( ) ( )f x f x=
thì:
1 2
( ( ) 2 ) ( ( ) 2 )f f x y f f x y+ = +
1 2 1 2
4 4 3 4 4 3x y x y x x + + = + + =
. Vậy
f
là đơn ánh.
Ta có:
( ( ) 2 ) 4 4 3 ( ( ) 2 ).f f x y x y f f y x+ = + + = +
Tài liu chuyên đề bi ng hc sinh gii
32 | C CHUYÊN ĐÊ: PHƯƠNG TRÌNH HÀM
Vì f là đơn ánh nên:
( ) 2 ( ) 2f x x f y x+ = +
hay
( ) 2 ( ) 2f x x f y y =
,
, .x y
Do đó:
( )
2f x x c =
,
.c
Thay
( )
2f x x c= +
vào điều kiện ta có
1c =
.
Vậy hàm số cần tìm là:
( )
2 1f x x= +
,
.x
Ví dụ 2: Tìm tất cả các hàm số
:f
+ +
thoả mãn điều kiện:
( )
( )
2
3 4
9 8
y
f x f y
xy
+ =
+
,
,x y
+
(1)
Li gii:
Ta sẽ chứng minh
f
là đơn ánh. Giả sử:
( ) ( )
,f x f y=
,x y
+
, khi đó:
( )
( )
( )
( )
2
2 2
3 4 3 4
9 8 9 8
y x
f x f y f x f x
xy x
= + = + =
+ +
Vậy
f
là đơn ánh.
( )
, 5 ,x y a +
Với
0a
xét
2 2 8
9 8 9
y y a
a x
xy ay
= =
+
. Do đó, từ (1) suy ra:
( )
( )
2 8 2 8
4 4
3 3
y a x a
f f y a f f x
ay ax
+ = = +
,
( )
, 5 ,x y a +
(2)
Do
f
đơn ánh nên từ (2) ta có:
( )
( )
2 8 2 8
4 4
3 3
y a x a
f y f x
ay ax
+ = +
,
( )
( )
2 2
3 3
f x f y
x y
=
,
( )
, 5 ,x y a +
( )
2
3
f x c
x
= +
,
( )
, 5 ,x y a +
(
c
là hng s) (3)
Với
0x
, luôn tồn tại
0a
sao cho
5x a
theo (3) ta có:
( )
2
.
3
f x c
x
= +
Vậy
( )
2
;
3
f x c
x
= +
(0, ).x +
Thử lại vào (1) dễ dàng suy ra:
0c =
hoặc
265
204
c =
.
Dễ thấy:
( )
2
3
f x
x
=
thoả mãn yêu cầu đề bài. Còn
( )
2 265
3 204
f x
x
=
không thế lớn hơn 0 với mọi
0x
.
Vy có duy nht 1 hàm s cn tìm
( )
2
, 0.
3
f x x
x
=
Ví d 3: (Sóc Trăng 2018) Tìm tt c các hàm
:f
tha mãn:
( )
( )
( )
( )
( )
2 2 8 , 1f x y f x f f y x x y+ + = +
Li gii:
Ta chng minh
f
là đơn ánh. T (1) hoán v
,x y
được
( )
( )
( )
( )
( )
2 2 8 , . 2f x y f y f f x y x y+ + = +
Ga s tn ti
1 2
,x x
tha mãn
( ) ( )
1 2
f x f x=
. Khi đó từ (1) và (2) được
1 2 1 2
8 8x x x x= =
nên
f
đơn ánh. Trong (2) cho
0y =
được
( )
( )
( )
( )
2 0 .f x f f f x x+ =
f
đơn ánh nên
( ) ( )
2 0f x x f x= +
hay
( )
2f x x a x= +
. Th li thy tha mãn.
Tài liu chuyên đề bi ng hc sinh gii
CHUYÊN ĐÊ 1: ĐẠI S VÀ GII TÍCH 33 |
Ví d 4: (Italy TST 2007) Tìm tt c các hàm
:f
tha mãn:
( )
( )
( ) ( ) ( )
, 1f xy f x xf y f x x y+ = +
Li gii:
Nhn thy
0f
là mt nghim hàm. Xét
0f
. Ta chng minh
f
là “tựa đơn ánh”, tức là nếu tn ti
,a b
tha
( ) ( )
0f a f b=
thì
.a b=
Trong (1) lần lượt cho
, ; ,x a y b x b y a= = = =
được:
( )
( )
( ) ( )
( )
( )
( ) ( )
f ab f a af b f a
f ab f b bf a f b
+ = +
+ = +
T 2 điều trên, do
( ) ( )
0f a f b=
suy ra
a b f= ⎯⎯
“tựa đơn ánh”.
Trong (1) cho
0x y= =
được:
( )
( )
( ) ( )
0 0 0 0f f f f= ⎯⎯ =
(vì nếu ngược li,
( )
0 0f
do
f
“tựa
đơn ánh” nên
( )
0 0f
, mâu thun).
Trong (1) cho
0y =
có:
( )
( )
( ) ( )
, . 2f f x f x x=
. Do
f
“tựa đơn ánh” nên t (2) suy ra
( ) ( )
0; , . 3f x x x
D thy các hàm:
( )
0,f x x=
hoc
( )
,f x x x=
tha mãn (1). Ta s chng minh ngoài 2 hàm
này không còn hàm nào khác. Ga s tn ti hàm
f
tha (1) và
( ) ( )
( )
( )
( )
3
: 0, :
0
f a a
a f a b f b b
f b
=
⎯⎯
=
Trong (1) cho
,x a y b= =
được:
( ) ( )
f ab a f a+ =
. Vì
( )
0f a
f
“tựa đơn ánh” nên
0ab a a ab+ = =
, mâu thun. Vy có 2 hàm tha mãn bài toán
( )
0,f x x=
hoc
( )
,f x x x=
.
2.2. Sử dụng tính toàn ánh giải phương trình hàm
Nếu
f
là toàn ánh thì ta hay dùng tồn tại một số
b
sao cho
( ) ( ) ( )
( )
0 1, 1,...f b f b f b= = =
sau
đó tìm
.b
Nếu
:f
toàn ánh thì với mọi
y
luôn tồn tại
x
sao cho
( )
.f x y=
Khi giải phương trình hàm dựa vào giá trị của đối số và giá trị của hàm số ta cũng có thể vận dụng
tính toàn ánh của
f
.
Mọi đa thức bậc lẻ có tập giá trị là . Tức là các hàm đa thức bậc lẻ đều là toàn ánh trên
.
Do đó
ta thường biến đổi để xuất hiện một đa thức bậc lẻ để có thể sử dụng tính toàn ánh.
Đối với phương trình hàm có chứa
( )
( )
( ) ( )
, , ,...f x f y f x f y
thì thường lấy
0x =
thường
thay
x
bởi
( ) ( )
, ,...f x f y
Ví dụ 1: Tìm tất cả các hàm số
:f
thỏa mãn
Tài liu chuyên đề bi ng hc sinh gii
34 | C CHUYÊN ĐÊ: PHƯƠNG TRÌNH HÀM
( ) ( )
( ( )) , , .f x f y x f y xf y x y+ = + +
Lời giải:
Giả sử tồn tại hàm số
thỏa yêu cầu bài ra.
Ta có thể viết lại quan hệ hàm dưới dạng :
( )
( )
( ) ( )
( ) 1 f x f y x f y f y+ = + +
(2)
Trường hợp 1:
( ) 1,f y y=
, thử lại thỏa yêu cầu bài toán.
Trường hợp 2:
( )f y
không đồng nhất với
1
, khi đó vế phải của (2) là hàm số bậc nhất theo
x
nên tập
giái trị của
( )
( )
f x f y+
. Suy ra
f
là toàn ánh.
Thay
x
bởi
0
vào (1) ta được:
( ( )) ( )f f y f y=
(3)
f
là toán ánh nên
,x R y
thỏa
( )f y x=
nên từ (3) ta được, thử lại ta thấy
( )f x x=
không
thỏa mãn. Vậy
( ) 1f x =
là hàm số thỏa mãn yêu cầu bài toán.
Nhận xét:
Trong các bài toán này từ điều kiện
( ( )) ( )f f y f y=
giúp ta tìm ra nghim của bài toán nhưng với điều kin f phi là toán ánh.
Như vậy, tính toán ánh của f trong bài toán này tỏ ra khá hiệu quả giúp bài toán trở nên dễ dàng hơn
trong quá trình tìm bời giải.
Ví d 2: Tìm tt c các hàm
:f
tha mãn:
( )
( )
( ) ( )
3 2
2 , , . 1f x y f y y x f x x y+ + = +
Li gii:
Cho
1x =
ta có
( )
( )
( )
1 2 1 ,f y f y y f y f+ + = +
là toàn ánh. Đặt
( )
0f a=
. Trong (1) cho
0x y= =
được
( )
0f a =
. Trong (1) cho
0,x y a= =
được
( )
2f a a=
. Do đó
( )
0 0 0.a f= ⎯⎯ =
Trong (1) cho
0x =
được
( )
( )
2 , .f y f y y y+ =
Trong (1) cho
0y =
được
( )
( )
3 2
, .f x x f x x=
T đó ta viết (1) dưới dng
( )
( )
( )
( )
( )
3 3
, , .f x y f y f y f y f x x y+ + = + +
f
toàn ánh nên t đây có
( ) ( ) ( ) ( ) ( )
, , , , .f x y f x f y x y f nx nf x x n+ = + =
Li có:
( ) ( )
( )
( ) ( )
( ) ( ) ( ) ( )
3 3
2 3 2
1 1 2 6 2 6 2 6 , . 2f x x f x x f x f x x f x f x x+ + = + = + = +
Tài liu chuyên đề bi ng hc sinh gii
CHUYÊN ĐÊ 1: ĐẠI S VÀ GII TÍCH 35 |
( ) ( )
( )
( )
( )
( )
( )
( ) ( )
( )
( ) ( )
( )
( )
( ) ( )
( )
( )
( ) ( )
( )
( )
( ) ( ) ( )
3 3 3 3 2 2
2 2
2
1 1 1 1 1 1 1 1
2 1 1 2 1 1
2 2 4 1 , . 3
f x x f x f x x f x x f x
x x f x f x x f x f
x f x xf x
+ + = + + = + + +
= + + + + +
= + +
T (2) và (3) suy ra
( ) ( )
1 , .f x xf x=
Th li thy
( ) ( )
1 1 , .f f x x x= ⎯⎯ =
Ví d 3: (Chuyên KHTN 2018) Tìm tt c các hàm
:f
tha mãn:
( ) ( ) ( )
( )
( ) ( ) ( )
1 0 , 1f x y f x f y x f y x x x y + + + =
Li gii:
Trong (1) cho
x y=
ta có:
( )
( )
( )
( )
( ) ( )
0 1 0 . 2f f x f x f x = +
Trường hp 1:
( )
0 1f =
Trong (2) cho
0x =
ta có:
( )
( )
( ) ( )
0 0 1 1 1f f f f = = =
T đó trong (1) cho
1x y= =
ta có:
( )
1 1f =
, cho
1, 0x y= =
được
( )
0 1f =
, mâu thun.
Trường hp 2:
( )
0 1f
. T (2) suy ra
f
là toàn ánh trên nên tn ti
c
sao cho
( )
0f c =
.
Trong (1) cho
0,x y c= =
ta có:
( )
( )
0 0f cf =
Trong (1) cho
x y c= =
ta có:
( ) ( ) ( )
0 1 0 0f c f c+ + + =
Trong (1) cho
( )
0x y cf= =
ta có:
( ) ( )
( )
( ) ( )
0 0 1 0 0 0f cf f cf+ + =
T đó
( )
( )
2
0 1 0 0f c c+ = =
hay
( )
0 0f =
. Do đó từ (2) suy ra
( )
( )
( )
. 3f f x x x =
Trong (1) cho
0x =
ta có:
( )
( )
( ) ( )
( )
( ) ( )
. 4f f y f y y f f x f x x = =
T (3) và (4) suy ra
( )
.f x x x=
Th li thy tha mãn.
Ví d 4: (Shortlist IMO 2012) Tìm tt c các hàm
:f
tha mãn:
( )
( )
( )
( )
( )
2 , , . 1f f x y x f f y x x y+ = +
Li gii:
Trong (1) thay
y
bi
( )
f x
ta có
( ) ( )
( )
( )
0 2 , .f x f f f x x x f= +
là toàn ánh.
Suy ra
a
sao cho
( )
0f a =
.
Trong (1) thay
x a=
được
( ) ( )
( )
( )
( )
( ) ( )
2 . 2f y a f f y a f f y a f y a a= + =
Tài liu chuyên đề bi ng hc sinh gii
36 | C CHUYÊN ĐÊ: PHƯƠNG TRÌNH HÀM
Do
f
toàn ánh nên vi mi
x
đều tn ti
y
sao cho
( )
x f y a=
. T (2) suy ra
( )
, .f x x a x=
Th li thy tha mãn.
Ví d 5: Tìm tt c
:f
, tha mãn
( )
( )
( ) ( ) ( )
( )
2f x f y f x xf y f f y = +
,x y
(1)
Li gii:
Xét
( ) 0f x =
.x
tha mãn.
Gi s tn ti
a
sao cho
( ) 0f a
Thay
y
bi a vào (1) ta có
2 . ( ) ( ( ) ( ) ( ( ))x f a f f a f x f x f a =
Khi đó ta có
x
thì tn ti a,b sao cho
( ) ( )x f a f b=
Do đó
( ) ( )f x f y
toàn ánh trên
Thay
x
bi
( )f y
vào (1) ta được
2
(0) 2 ( ( )) 2 ( )f f f y f y=
Thay
x
bi
( )f x
vào (1) :
2 2
(0)
( ( ) ( )) ( ( )) 2 ( ). ( ) ( ( )) ( ) 2 ( ). ( ) ( )
2
f
f f x f y f f x f x f y f f y f x f x f y f y = + = + +
2
(0)
( ( ) ( )) ,
2
f
f x f y x y= +
2
( ) .f x x b x⎯⎯ = +
Th li thy
0b =
Vy có 2 hàm tha mãn
( ) 0,f x x=
2
( ) ,f x x x=
2.3. Sử dụng tính song ánh giải phương trình hàm
Khi f là một song ánh ta có thể chú ý đến tính chất đơn ánh và toán ánh mà ta đã vận dụng trong phần 1
và phần 2. Ngoài ra ta có thể chú ý thêm.
Nếu
:f
tho mãn:
( )
( )
, , 0( )f f x ax b x a= +
thì
f
là song ánh.
f
song ánh liên tục (đơn điệu) và cộng tính trên tập khi đó
( )
f x ax=
(
f
song ánh và cộng tính trên tập rời rạc khi đó
( )
f x ax=
)
Ví d 1: (Olympic 30.4.2011)
Tìm tt c các hàm s
:f
tho mãn điều kin:
( )
( )
( )
( )
1 ( ) ,f f x f y y xf y+ = +
,x y
(1)
Tài liu chuyên đề bi ng hc sinh gii
CHUYÊN ĐÊ 1: ĐẠI S VÀ GII TÍCH 37 |
Li gii:
Từ (1) cho
0x =
ta được
( )
( )
( )
1 (0)f f f y y+ =
,
y
(1)
Giả sử
( ) ( )
1 2
f y f y=
, từ (2) dễ dàng suy ra
1 2
y y=
. Do vậy
f
là đơn ánh. Với
y
khi đó tồn tại
( )
( )
( )
1 0x f f y= +
sao cho
( )
,f x y=
suy ra
f
là toàn ánh, dẫn tới
f
là song ánh.
Vì thế tồn tại
c
sao cho
( )
0.f c =
Từ (1) cho
y c=
được
( )
0.f c =
Từ (1) cho
0x y= =
ta được
( )
( )
1 0.f c c+ =
Vậy
( )
( )
( )
1f c c f c+ =
, mà
f
là đơn ánh nên:
( ) ( )
1 0 0 0.c c c c f+ = = =
Từ (1) cho
0x =
ta được
( )
( )
f f y y=
,
y
(3)
Từ (1) thay
x
bởi
( )
f x
, thay
y
bởi
( )
f y
và sử dụng (3) ta có:
( )
( )
( )
( )
1f y x f y yf x+ = +
,
,x y
(4)
Từ (4), cho
1x =
, sử dụng
( )
0 0f =
và đặt
( )
1a f=
ta được:
( )
f y ay=
,
y
Thay vào (3) đồng nhất được
1:1a
Vậy:
( ) ( )
, .f x x f x x
Thử lại ta thấy cả 2 hàm đều thoả mãn.
Ví dụ 2: Tìm tt c các hàm s
:f
tha mãn:
2 2
( 2 ( )) 2 ( ); ,f x f y y f x x y+ = +
Li gii:
Gi s tn ti hàm s
( )f x
tha mãn
2 2
( 2 ( )) 2 ( ); , (1)f x f y y f x x y+ = +
Trong (1), thay
0x =
suy ra
2
(2 ( )) 2 (0)f f y y f= +
(2)
Suy ra
f
là song ánh
Cho
0y =
, thay vào (1) ta được
2 2
( 2 (0)) ( ); , (3)f x f f x x y+ =
Thay
x
bi
x
vào (3) ta được
2 2
( 2 (0)) ( )f x f f x+ =
Suy ra
2 2
( ) ( );f x f x x =
Nếu
( ) ( ) 0f x f x x x x= = =
Do đó
( ) ( ), 0f x f x x =
Vì
f
là song ánh nên tn ti duy nht
: ( ) 0.b f b =
Nếu
0b
thì
( ) ( )
0f b f b = =
, hay
( ) ( )
f b f b =
hay
b b=
ta được
0.b =
Suy ra
( )
0 0f =
hay
( ) ( )
f x f x =
x
hay
f
là hàm l trên
( ) 0 0f x x= =
Tài liu chuyên đề bi ng hc sinh gii
38 | C CHUYÊN ĐÊ: PHƯƠNG TRÌNH HÀM
Khi đó (2) thành
( )
( )
2 2 ,f f y y=
y
(3) thành
2 2
( ) ( ) 0,f x f x x=
, hay
( ) 0, 0f x x
Khi đó (1) thành
2 2
( 2 ( )) (2 ( )) ( ) (4)f x f y f f y f x+ = +
Hay
( ) ( ) ( ) 0,f x f y f x y x y+ = +
Mt khác nếu
0,x y
:
( ) ( ) ( ( ) ( )) ( ) ( )f x f y f x f y f x y f x y+ = + = = +
Hay
( ) ( ) ( ) ,f x f y f x y x y+ = +
Vi
x y
ta có:
( ) ( ) ( ) ( ) ( )
f x f x y y f x y f y f y= + = +
hay
f
đồng biến trên
Ta đi chứng minh
( ) ,f x x x=
Tht vy nếu
o
x
:
( )
o o
f x x
hay 2
( ) 2 ( ( )) (2 )
o o o o
f x x f f x f x
hay
2 2 ( ) ( )
o o o o
x f x x f x
( vô lý)
Tương tự nếu
o
x
:
0
( )
o
f x x
, ta cũng dn ti vô lý.
hay
( )f x x=
x
Th li ta thy
( )f x x=
x
tha mãn phương trình ( 1)
Vy
( )f x x=
x
Ví d 3: (VMO 2017) Tìm tt c các hàm
:f
tha mãn:
( ) ( )
( )
( ) ( )
2 , 1f xf y f x f x xy x y = +
Li gii:
Trong (1) cho
1x =
đươc:
( ) ( )
( )
( ) ( )
1 2 1 2f f y f f y y = +
T đây dễ thy
f
là song ánh. Do đó tồn ti duy nht s thc
a
để
( )
0f a =
. Trong (1) cho
x a=
được
( )
( )
( )
. 3f af y ay y=
Trong (3) cho
0y =
được
( )
( )
( )
0 0 .f af f a= =
T đó vì
f
đơn ánh nên
0a =
hoc
( )
0 1.f =
Nếu
( )
0 0 0a f= =
. Thay
0y =
vào (1) được
( )
( )
( )
2f f x f x =
. Do
f
là toàn ánh nên
( )
2 , .f x x x=
Th li thy không tha mãn. Vy
( )
0 0 1.a f =
Trong (1) thay
0x =
ta được
( )
1 2f =
. Thay
y a=
vào (3) được
( ) ( )
( )
2
0 1 1 1 2a f a do f= = = =
tc
( )
1 0.f =
Tài liu chuyên đề bi ng hc sinh gii
CHUYÊN ĐÊ 1: ĐẠI S VÀ GII TÍCH 39 |
Do
( )
( )
( )
( )
( )
2
1 1 . 2f f f y y y
= ⎯⎯ =
Thay
y
bi
( )
f y
vào (1) và s dụng (2’) được
( )
( )
( ) ( ) ( )
2 , . 4f xy f x f x xf y x y = +
Trong (4), xét
0x
và thay
( )
f x
y
x
=
được
( )
( ) ( ) ( )
1 2
1 2 , 0.
f x f x f x
f x xf f x
x x x
= + ⎯⎯ =
Thay
( )
f x
y
x
=
vào (1) và s dng kết qu trên được
( )
( )
( )
1 3 3 , 0.f f x f x x =
Do
f
là song ánh và
( )
0 1f =
nên vi
0x
thì
( )
1 3 f x
có th nhn mi giá tr thc khác
2
. Do đó
t kết qu trên suy ra
( )
1, 2.f x x x= +
Nói riêng
( )
3 2f =
. Thay
3y =
vào (2’) ta được
( )
2 3f =
. Vy
( )
1, .f x x x= +
Th li tha
mãn.
3. Bài tập vận dụng
Bài 1: (Olympic 30.4.2006)
Tìm tt c các hàm s
:f
tho mãn điều kin:
( ) ( )
( )
f x f y xf y x xy y+ + = + +
,
,x y
(1)
Li gii:
Ta s chng minh
f
là đơn ánh. Giả s
( ) ( )
1 2
f y f y=
( ) ( )
( )
1 1 2 2
( ( ) ( ))f x f y xf y f x f y xf y + + = + +
1 1 2 2 1 2
x xy y x xy y y y f + + = + + = ⎯⎯
là đơn ánh.
Cho
0x =
, thay vào (1) ta có:
( ( )) ,f f y y=
.y
Cho
0y =
, thay vào (1) ta có:
( ) ( )
( )
( )
( )
0 0f x f xf x f f x+ + = =
(0) (0) ( ).x f xf f x + + =
Đặt
( )
0 ,f a=
t đó ta có:
( ) ( 1) ,f x a x a= + +
x
(2)
Thay (2) vào (1), d dàng suy ra
0a =
hoc
2.a =
Khi
( )
0, .a f x x= =
Khi
2,a =
( ) 2.f x x=
Th li ta thy tho mãn.
Vy có 2 hàm s to mãn đề bài:
( ) ,f x x=
x
( ) 2,f x x=
.x
Bài 2: Tìm tt c các hàm s
f
xác định và đồng biến trên tha mãn:
( ) ( )
1
2 4 1, , . 1
4
f f y x x y x y
+ = + +
Tài liu chuyên đề bi ng hc sinh gii
40 | C CHUYÊN ĐÊ: PHƯƠNG TRÌNH HÀM
Li gii:
Thay
1
4
x y=
vào (1) được:
( ) ( )
1 1
1, . 2
4 2
f f y y y
=
Trong (1) cho
0x y= =
được
( )
1
0 1.
4
f f
=
Kết hp (2) suy ra
( ) ( ) ( )
1 1 1
0 . 3
4 2 4
f f y y f f y
=
Do
f
đồng biến nên
f
là đơn ánh trên , t (3) suy ra
( ) ( ) ( )
1 1 1
0 , 2 , .
4 2 4
f y y f y f x x a x = = +
Thay vào (1) được
( )
2 2
2 , .
3 3
a f x x x= ⎯⎯ = +
Bài 3: (VMO 2016) Tìm tt c các s thc
a
để tn ti hàm
:f
tha mãn:
i)
( )
1 2016f =
ii)
( )
( )
( )
, , .f x y f y f x ay x y+ + = +
Li gii:
Vi
0a =
có th tìm được 1 hàm tha mãn bài toán
( )
2016.f x =
Do đó chỉ xét trường hp
0a
đủ. Thay
( )
x f y=
vào ii) ta được:
( ) ( )
( )
, .f y f f y ay y= +
T đó dễ thy
f
là đơn ánh.
Thay
0y =
vào ii) được
( )
( )
( ) ( )
0 , 0 0.f x f f x x f+ = =
Thay
( )
f x
y
a
=
vào ii) và kết hợp tính đơn ánh của
f
được
( ) ( )
, .
f x f x
f x x
a a
+ =
Thay
y
bi
( )
f y
a
vào ii) và s dng kết qu trên được
( ) ( ) ( )
, , .f x y f x f y x y =
Suy ra
f
cng tính. T đó dễ tính được
( ) ( )
2
2016 2016 1 2016 .f f= =
Do
f
cng tính nên t ii) rút ra
( ) ( )
( )
.f y f f y ay y+ =
Cho
1 2016.2017.y a= ⎯⎯ =
Th li vi giá tr trên ca
a
ta tìm được
( )
2016f x x=
tha mãn bài toán.
Vy có 2 giá tr cn tìm ca
a
0a =
hoc
2016.2017.a =
Bài 4: ( Ra Vũng Tàu 2014) Tìm tt c các hàm
:f
tha mãn:
( )
( )
( )
( )
( )
2 , 1f xy f x f x yf x x x y+ + =
Tài liu chuyên đề bi ng hc sinh gii
CHUYÊN ĐÊ 1: ĐẠI S VÀ GII TÍCH 41 |
Li gii:
Trong (1) cho
0y =
được
( )
( )
( )
2 ,f f x f x x x f+ =
là đơn ánh.
Trong (1) lần lượt cho
1, 1y y= =
được
( )
( )
( )
( )
( )
2 , . 2f x f x f x f x x x+ + =
( )
( )
( )
( )
( )
2 , . 3f x f x f x f x x x + + + =
T (2) và (3) suy ra
( )
( )
( )
( )
( )
, . 4f x f x f x f x x = +
. Vì
f
đơn ánh nên từ
( ) ( ) ( ) ( )
4 , ,x f x x f x x f x x x⎯⎯ = + =
. Th li thy tha mãn.
Bài 5: (Quãng Ninh 2018) Tìm tt c các hàm
:f
tha mãn:
( )
( )
(
)
( ) ( )
2
2 2
, 1f x f y xf x y x y = +
Li gii:
Đặt
( )
2
0a f=
. Trong (1) cho:
0x y= =
được
( )
0f a =
0,x y a= =
được
( )
2
0f a=
Do đó
4
0
1
a
a a
a
=
=
=
Nếu
( )
1 0 1a f= =
. Trong (1) cho
( ) ( ) ( )
1, 0 1 1 1 1 0x y f f f a= = = = = =
, vô lý.
Suy ra
( )
0 0 0a f= =
. Trong (1) cho
( )
( )
2
0 .y f x xf x x= =
Nếu
( )
( )
( )
(1)
2 2
0 0.f y f x xf x y y= ⎯⎯ = =
Trong (1) cho thay
0,x y x= =
được
( )
( )
( )
2 2
. 2f f x x x =
Trong (1) cho thay
x
bi
x
,
0y =
được
( )
( )
2
f x xf x x=
Suy ra
( ) ( ) ( ) ( )
0.xf x xf x x f x f x x= =
Kết hp
( ) ( ) ( ) ( )
0 0 . 3f f x f x x= =
Ta s chng minh
f
là toàn ánh trên
.
T (2) ta có vi mi
0y
luôn tn ti
x
sao cho
( )
.f x y=
Còn vi
0y
theo (2), (3) thì tn ti
x
sao cho
( )
( )
( )
( )
2 2 2
y x f f x f f x= = =
.
Tài liu chuyên đề bi ng hc sinh gii
42 | C CHUYÊN ĐÊ: PHƯƠNG TRÌNH HÀM
Vy
f
là toàn ánh. Tiếp theo chng minh
( )
, .f x x x=
T (1) s dng các kết qu trên ta có
( )
( ) ( )
( )
( )
( )
2 2 2 2
, . 4f x f y f x f f y x y = +
T (4) do
f
toàn ánh nên
( ) ( ) ( )
, 0, 0.f x y f x f y x y+ = +
Kết hp (3) suy ra
( ) ( ) ( )
, 0, 0.f x y f x f y x y+ = +
Mt khác
( ) ( ) ( ) ( ) ( ) ( )
, 0, 0.f x f x y y f x y f y f x y f y x y= + = + + = +
Kết hp (3) suy ra
( ) ( ) ( )
, 0, 0.f x f x y f y x y= +
Vy
( ) ( ) ( ) ( )
, , . 5f x y f x f y x y+ = +
S dng
( )
( )
2
.f x xf x x=
và (5) ta được:
( )
( )
( ) ( ) ( )
( )
( )
( )
( )
( ) ( ) ( )
( ) ( )
2
2
2
1 1 1
1 2 1 2 1
1 .
f x x f x f
f x f x x xf x f x f
f x xf cx x
+ = + +
+ = + + = + +
= =
Thay vào (1) được
( )
1 , .c f x x x= =
Th li tha mãn
Bài 6: (Hà Nam 2018) Cho s thc
a
khác
0
và khác
1
. Tìm tt c các hàm
:f
tha mãn:
( )
( )
( )
( )
( )
( )
2
, 1f f x ay a a x f f y x x y+ = + +
Li gii:
Trong (1) thay
y
bi
( )
f x
a
ta có:
( )
( )
( )
2
0 .
f x
f f x a a x f x
a
= + +
Suy ra
f
là toàn ánh trên .
Bây gi gi s tn ti
1 2
,x x
sao cho
( ) ( )
1 2
f x f x=
, do
f
là toàn ánh nên tn ti
t
sao cho
( )
1 2
f t x x= +
. Khi đó, trong (1) cho
1
,x x y t= =
có:
( )
( )
( )
( )
2
1 1 2
f f x at a a x f x+ = + +
Trong (1) cho
2
,x x y t= =
có:
( )
( )
( )
( )
2
2 2 1
f f x at a a x f x+ = + +
Do đó
1 2
x x=
, tc
f
là đơn ánh trên
.
Mt khác trong (1) cho
0x =
có:
( )
( )
( ) ( ) ( )
0 0 .f ay f f y y f y y f y+ = = +
Th li thy tha mãn.
Bài 7: ( Ra Vũng Tàu 2017) Tìm tt c các hàm
:f
tha mãn:
( )
( )
( ) ( ) ( )
, . 1f x yf x xf y f x x y+ = +
Tài liu chuyên đề bi ng hc sinh gii
CHUYÊN ĐÊ 1: ĐẠI S VÀ GII TÍCH 43 |
Li gii:
Trong (1) cho
0y =
có:
( ) ( ) ( ) ( )
0 0 0.f x xf f x x f= + =
Ta nhận thấy
( )
0f x
là một nghiệm hàm.
Xét
( )
0f x
. Khi đó tồn tại
0a
sao cho
( )
0f a
Trong (1) cho
x a=
ta có
( ) ( )
0 0af y f y y= =
vô lý.Vy
( )
0 0.f x x= =
Trong (1) cho
( )
1, 1 1 1.x y f= = =
Trong (1) cho
( )
( )
( )
1, 1 1 1 1 1.x y f f f= = =
Trong (1) cho
( ) ( )
( )
( )
2
1 1 , 1 1 1 0 1 1.x f y f f= = = =
Trong (1) cho
( ) ( )
1 1 1 .x f y f y y= + = +
Trong (1) thay
y
bi
( ) ( )
( )
( ) ( ) ( )
( )
1 , .y f x yf x f x xf y f x x f x yf x x x y+ + + = + + = + +
Mt khác do
( )
x yf x+
toàn ánh nên thay
( )
x yf x+
bi
y
được
( )
( )
( ) ( )
, . 2f y f x f y x x y+ = +
Trong (2) cho
0y =
được
( )
( )
.f f x x x=
Trong (2) thay
x
bi
( )
f x
được
( ) ( ) ( ) ( )
, 3f x y f x f y x y+ = +
Trong (1) thay
x
bi
( )
f x
và kết hợp (3) được
( ) ( ) ( ) ( )
, . 4f xy f x f y x y=
T (3), (4) ta nghim hàm là
( )
f x x
hoc
( )
0.f x
Bài 8: (Chọn đổi tuyển Indonesia 2010)
Xác định tất cả các số thực
a
sao cho có một hàm số thoả mãn:
( ) ( ( ))x f y af y f x+ = +
,
,x y
(1)
Li gii:
Dễ thấy
0a =
không thoả mãn. Giả sử:
0.a
Thay
0y =
vào (1) ta được:
(0)
( ( )) ;
x f
f f x
a
+
=
x
(2)
Từ (2) suy ra
f
là toàn ánh nên tồn tại
sao cho f() = 0.
Khi đó từ (1) lấy
x
=
ta được:
( ) ( ),f y af y
+ =
y
hay:
( 1) ( ),a f y
=
y
(3)
Từ (3) thì sẽ xảy ra hoặc
1a =
hoặc
f
là hàm hằng.
+) Nếu
f
là hàm hng thì không tho mãn (1).
+) Nếu
1a =
chn
( )
f x x=
tho mãn (1).
Tài liu chuyên đề bi ng hc sinh gii
44 | C CHUYÊN ĐÊ: PHƯƠNG TRÌNH HÀM
Vậy
4. Bài tập củng cố
Bài tập 1: Tìm tất cả các hàm
:f
thỏa mãn
( ) ( )
( )
( ) ( ) ( )
2 1f x f x f y x f x y f y+ + = + + +
với
, .x y
Li gii:
Giả sử tồn tại hàm
thỏa mãn.
Ta chứng minh
f
là đơn ánh. Thật vậy, giả sử tồn tại
1 2
,y y
thỏa mãn
( ) ( )
1 2
f y f y=
. Cố định
x
, lần
lượt cho
1 2
,y y y y= =
từ
( )
1
ta có:
( ) ( )
( )
( ) ( )
( )
( ) ( ) ( ) ( )
1 1 1 1 2 2
2 2f x f x f y f x f x f y x f x y f y x f x y f y+ + = + + + + + = + + +
Suy ra
1 2
y y=
. Vậy
f
là đơn ánh.
Bài tp 2: (Thụy Sĩ 2010) Tìm tt c các hàm
:f
tha mãn:
( ) ( )
( )
( ) ( )
2 , . 1f f x f y y f x y x y+ = +
Li gii:
Trong (1) cho
x y=
được:
( )
( )
( )
2 2 0 .f f x x f x= +
T đây dễ dàng suy
f
đơn ánh.
Trong (1) cho
0x y= =
được:
( )
( )
( ) ( )
2 0 0 0 0.f f f f= ⎯⎯ =
Trong (1) cho
0y =
được:
( ) ( )
( )
( ) ( ) ( ) ( )
0 0 , , .f f x f f x x f x f x x f x x x+ = + = =
Th li thy tha mãn.
Bài tp 3: (Tây Ban Nha 2012) Tìm tt c các hàm
:f
tha mãn:
( ) ( ) ( )
( )
( )
( )
( )
2 2 , , . 1x f y f y f x f x yf x x y + + = +
Li gii:
Nếu
( )
0 0f =
trong (1) cho
0x =
được
( )
0, .f y y=
Nếu
( )
0 0f
. Ta chng minh
f
đơn ánh. Thật vy trong (1) cho
0y =
được:
( ) ( ) ( )
( )
( )
2 0 2 , .x f f f x f x x + =
T đây dễ dàng suy ra
f
đơn ánh.
Trong (1) cho
2, 1x y= =
được:
( )
( )
( )
( )
( ) ( ) ( )
1 2 2 2 2 1 2 2 2 2 2 1f f f f f f f+ = + ⎯⎯ + = + ⎯⎯ =
Do
f
đơn ánh nên
( )
1, 2.f x x
Tài liu chuyên đề bi ng hc sinh gii
CHUYÊN ĐÊ 1: ĐẠI S VÀ GII TÍCH 45 |
Trong (1) thay
( )
( )
2
1
f x x
y
f x
=
ta có:
( )
( )
( )
( )
( )
( )
( )
( )
( )
( )
( )
( )
( )
( )
( )
2 2 2
2 2 , 2
1 1 1
2
2 0, 2
1
2
0, , 2
1
f x x f x x f x x
x f f f x f x f x x
f x f x f x
f x x
x f x
f x
f x x
f a a x
f x
+ + = +
=
= =
Trong (1) thay
, 2x a y= =
ta có:
( ) ( ) ( )
( )
( )
( )
( )
( )
( )
( )
2 2 2 2 2 0 2 1
2
1, 2 1, 1.
1
a f f f a f a f a a f
f x x
x f x x x
f x
+ + = + = = =
⎯⎯ = ⎯⎯ =
Ta thy vi
2x =
cũng thỏa mãn hàm trên. Vy bài toán có 2 nghim hàm
( )
0,f x x=
hoc
( )
1, .f x x x=
Bài tp 4: (Th Nhĩ Kỳ 2012) Tìm tt c các hàm
:f
tha mãn:
i)
( )
( )
( )
( )
2 2
2 .f f x y f y x f y+ + = +
ii)
x y
thì
( ) ( )
.f x f y
Li gii:
Ta chứng minh
f
đơn ánh.
Cho
0y =
thay vào i) đưc
( )
( )
( )
( ) ( ) ( )
( )
( ) ( )
2 2
0 2 0 , x 0 2 0 , x 0. 1f f x f x f f f x f x f+ = + + = +
Do đó
f
đơn ánh trên
)
0;+
Gi s tn ti
1 2
,y y
tha mãn
( ) ( )
( )
( )
( )
( )
( )
( )
)
2 2
1 2 1 1 2 2
i
f y f y f f x y f y f f x y f y= ⎯⎯ + + = + +
. Vì
f
không b chn nên cho
x
đủ ln thì
( )
( )
( )
( )
2 2
1 1 2 2
,f x y f y f x y f y+ + + +
đều dương nên từ (1) kết
hp
f
đơn ánh trên
)
0;+
ta có
1 2
y y f=
đơn ánh trên
.
Ta chứng minh
( )
0 0.f =
+ Nếu
( )
0 0f
cho
( )
2 0x f=
thay vào (1) được
( )
( )
( )
( )
2 0 0 0f f f f + =
hay tn ti
c
tha
( )
0.f c =
Cho
0,x y c= =
thay vào i) đưc
( )
( )
0 0.f f c+ =
f
đơn ánh nên
( ) ( )
0 0 0.f c c f+ = =
Tài liu chuyên đề bi ng hc sinh gii
46 | C CHUYÊN ĐÊ: PHƯƠNG TRÌNH HÀM
Nếu
( )
0f x
cho
0x y= =
thay vào i) được
( )
( )
( )
2 0 2 0 .f f f=
Trong (1) cho
( ) ( ) ( )
( )
0
3 0 0 2 0x x f f f f= = = +
có:
( ) ( ) ( )
( )
( )
( ) ( ) ( )
0
0 2 0 2 0 2 0 4 0 .f x f f f f f f f= + = + =
Ta có
( ) ( )
( )
( )
( )
( ) ( ) ( ) ( )
0
0 5 0 3 0 2 0 5 0 . 2f f x f f f f f f+ = = + =
Cho
( )
0, 2 0x y f= =
thay vào (1) được
( )
( )
( ) ( )
5 0 4 0 . 3f f f=
T (2) và (3) suy ra
( )
0 0.f =
Khi đó (1) viết lại
( )
( )
, x 0. (2)f f x x=
Thay
0x =
vào i) ta đưc
( )
( )
( )
2 , y .f y f y f y+ =
Cho
( )
y f x=
ta được
( )
( )
2 , .f f x x x x+ =
T (2) và (3) suy ra
( ) ( )
, 0. 3f x x x=
Với mỗi
0
y
luôn tồn tại
0
x
sao cho
( )
2
0 0 0
0x y f y+ +
. Do đó từ i) và (3) ta có
( )
( )
( ) ( ) ( ) ( )
2 2 2
0 0 0 0 0 0 0 0 0 0
2 , .f x y f y x y f y x f y f y y f x x x+ + = + + = + ⎯⎯ = ⎯⎯ =
Th li thy tha mãn.
Bài tp 5: (Olympic Duyên hi Bc b 2016) Tìm tất cả các hàm số
:f
sao cho:
( )
( )
( )
( )
2
, ,f xf x y f yf x x x y+ = +
(1)
Li gii:
Cho
0x =
vào (1) ta
( ) ( )
( )
0 0 ,f f yf y=
, suy ra
( )
0 0f =
nếu ngược
lại cho
(0)
t
y
f
=
thì
( ) ( )
0 ,f f t t=
ta thu được
f
là hàm hằng, thay vào (1) thấy vô lý.
Cho
0y =
y x=
vào (1) thì có
( )
( )
2
f xf x x=
( )
( )
( )
( )
( )
( )
2 2
0 f xf x x f xf x x x x f xf x= = + =
.
Nếu tn ti
0
0t
sao cho
( )
0
0f t =
thì
( )
( )
( )
2
0 0 0
0 f t f t t= =
vô lý.
Chng minh hàm s cần tìm là đơn ánh.
Gi s
( ) ( )
f x f y=
ta có:
Tài liu chuyên đề bi ng hc sinh gii
CHUYÊN ĐÊ 1: ĐẠI S VÀ GII TÍCH 47 |
( )
( )
( )
( )
( ) ( )
( )
( ) ( )
( )
( )
2 2
0
0
0
x f xf x f xf y f y x f x x f y x f x
f x
x y
x y
= = = + =
=
=
=
Ta chng minh
( ) ( )
.f x f x x =
Ta xét
0x
nếu
0x =
là hin nhiên.
Gi s
( )
0 0f x z
sao cho
( )
2
f x z=
. Vì
f
đơn ánh và
( )
( )
2
f zf z z=
nên
( )
x zf z=
do đó
( ) ( )
( )
( )
2
f x f zf z z f x = = =
.
Trong trường hp
( )
0f x
chứng minh tương tự.
Mt khác, ta có
( )
( )
( )
( )
( ) ( ) ( )
( )
( ) ( )
( )
( ) ( ) ( )
( )
( )
( )
2
2 2
2
2
2
2
2 2
f yf x x f xf x y
x x y x y f xf x y
y xy f x y f y
xy y f x y f y xy f yf x
= + +
= + + + + +
= + +
= + + + = +
Suy ra
( )
( )
, ,f yf x xy x y=
.
Tương tự
( )
( )
f xf y xy=
thế
( ) ( ) ( )
,xf y yf x f x cx x= =
thay vào (1) suy ra
1;1c
.
Th li hai hàm s
( )
f x x=
tha mãn yêu cu.
Bài tập 6: (Argentia TST 2008) Tìm tất cả các hàm
( ) ( )
: 0; 0;f + +
thỏa mãn
( ) ( )
( )
( ) ( )
( )
( )
2
, , 0. 1x f x f y x y f yf x x y+ = +
Li gii:
Ta chứng minh
f
là đơn ánh. Gỉa sử tồn tại
1 2
, 0x x
thỏa mãn
( ) ( )
1 2
0f x f x=
.
Lần lượt cho
1 2
,x x x x= =
thay vào
( )
1
ta được:
( ) ( )
( )
( ) ( )
( )
( ) ( )
( )
( ) ( )
( )
( ) ( )
( ) ( )
( )
2
1 1 1 1
2 2
1 2 2 1
2
2 2 2 2
1 2 1 2 1 2 1 2
0
x f x f y x y f yf x
x x y x x y
x f x f y x y f yf x
x x x x y x x x x
+ = +
+ = +
+ = +
+ + = =
Vậy
f
đơn ánh trên
( )
0;+
. Cho
thay vào
( )
1
có:
( ) ( )
( )
( ) ( )
( )
( )
2
2 2 , 0 2x f x xf xf x xf x f xf x x= =
Tài liu chuyên đề bi ng hc sinh gii
48 | C CHUYÊN ĐÊ: PHƯƠNG TRÌNH HÀM
Trong
( )
2
cho
1x =
ta thu được
( ) ( )
( )
( )
1 1 1 1f f f f= =
.
Trong
( )
1
cho
1x =
ta thu được
( ) ( ) ( ) ( )
1
1 1 , 0.f y y f y f y y
y
+ = + =
Thử lại thấy thỏa mãn.
Bài tp 7: Tìm tt c các hàm s
( ) ( )
: 0; 0;f + +
tha mãn :
( )
( )
( )
( )
( )
( )
( )
( )
( )
( )
( )
2 2 2
,xf x f f y f yf x f xy f f x f f y+ = +
vi mi
( )
, 0; .xy +
Li gii:
Thay
( ) ( )
; 1;1xy =
ta được:
( ) ( )
( )
( )
( )
( ) ( )
( )
( )
( )
( )
( )
( )
( ) ( )
( )
1 1 1 1 1 1 1 1 1f f f f f f f f f f f f f f f+ = + =
( )
( )
( )
1 0;ff +
nên
( )
1 1.f =
Thay
( ) ( )
; 1;x y x=
và s dng
( )
1 1.f =
ta được:
( )
( )
( )
( )
( )
( )
2
, 0. 2f f x f x f f x x=
Thay
( ) ( )
; ;1x y x=
và s dng
( )
1 1.f =
ta được
( )
( )
( )
( ) ( )
2 2 2
; , 0. 3xf x f x x f x xf x x= =
T (2) và (3) suy ra:
( )
( )
( )
( )
( )
( )
( )
( )
22
, 0. 4
f f x
f f x f f x x
fx
= =
Ta chng minh là một đơn ánh, thật vy gi s
( )
, 0;

+
tha mãn
( ) ( )
ff

=
. Khi
đó
( )
( )
( )
( )
( )
( )
( )
( )
( )
( )
( )
22
, 0. 5
f f f f
f f f f x
ff



= = =
Trong (1) thay
y
bi
x
ta được:
( )
( )
( )
( )
( )
( ) ( )
( )
( )
( )
( )
( )
( ) ( )
( )
( )
( )
( )
( )
( )
( )
( )
( )
( )
( )
( )
( )
2 2 2 2
2 2 2 , 0
2 , 0. 6
2 , 0. 7
f f x
fx
xf x f f x f xf x f x f f x f x x
xx
xf x f f x xf xf x f f x x
xf xf x f f x xf x x
+ = = =
+ =
=
Trong (6) thay bi ta được:
( ) ( )
( )
( )
( )
( )
( )
( )
( )
( )
( )
( )
( )
( )
2 2 2 2 2 2 2 2 2
2 , 0 2 , 0. 8x f x f f x x f x f x f f x x x f xf x f f x xf x x+ = =
T (6) và (8) suy ra:
( )
( )
( )
( )
( )
2
, 0. 9
f f x
xx
f f x
=
T (5) và (8) suy ra
( )
( )
( )
( )
( )
( )
( )
( )
22
f f f f
f f f f



= = =
. Do đó là đơn ánh. Kết hp (4) suy ra
f
x
2
x
f
Tài liu chuyên đề bi ng hc sinh gii
CHUYÊN ĐÊ 1: ĐẠI S VÀ GII TÍCH 49 |
( )
( )
( )
( ) ( )
2 2
2
1
, 0 , 0 , 0.
f x
f x f x x f x x f x x
x x
= = ⎯⎯ =
Th li thy
( )
1
, 0f x x
x
=
thỏa mãn điều kiện ban đầu.
Bài tập 8: Tìm hàm
:f
thỏa mãn một trong hai điều kiện
i)
( )
( )
( )
2
,f x f y y xf x x y+ = +
,
ii)
( )
( )
( )
(
)
( )
2
, .f f x f y y xf x x y+ = +
Hướng dn gii:
Ta tìm hàm
f
thỏa mãn ii) Đối với i) ta làm tương tự. Ngoài ra có th thấy hai điều kin này có th biến
đổi v nhau.
Ta cũng d thy
f
là đơn ánh
( ) ( )
( )
0 0, .f f f y y y= =
Trong ii) thay x bi
( )
f x
ta
( )
( )
( )
( )
( ) ( )
( )
2
.f f f x f y y f x f f x
+ = +
Mt khác
( )
( )
,f f y y y=
nên
( )
( )
( )
2
, , .f x y y xf x x y+ = +
Kết hp ii) thì
( )
( )
( )
( )
( )
(
)
2
2
f x f y f f x f y+ = +
f đơn ánh nên
( ) ( )
( )
( )
2
2
x f y f x f y+ = +
. Suy
ra
( )
( )
2
2
, .f x x x=
Ta ch ra không tn tại đồng thi
0, 0a b
tha mãn
( ) ( )
,f a a f b b= =
. Tht vy, gi s tn ti
,a b
như trên. Trong ii) ly
,x a y b= =
ta
( )
2 2
.f a b a b = +
Do
( )
( )
2
2
,f x x x=
nên
( ) ( )
2 2
2 2 2
0a b a b a b = + =
, mâu thun.
Vy
( )
f x x x=
hoc
( )
.f x x x=
Th li thy hai hàm này tha mãn.
Bài tp 9: Tìm tt c các hàm s
:f
tha mãn
( )
( )
( )
( )
2 , ,f x y f y f f x y x y+ + = +
.
Hướng dn:
Chng minh
f
là đơn ánh
Tht vy, vi mi x, y tha mãn
( ) ( )
f x f y=
ta có
Tài liu chuyên đề bi ng hc sinh gii
50 | C CHUYÊN ĐÊ: PHƯƠNG TRÌNH HÀM
( )
( )
( )
( )
( )
( )
( )
( )
2
2
f x y f y f f x y
x y
f x y f x f f y x
+ + = +
=
+ + = +
Chn
0y =
ta được
( )
( )
( )
( )
( ) ( )
0 0 ,f x f f f x f x x f x+ = ⎯⎯ = +
Vy
( )
,f x x c x= +
vi c là hng s.
Thay vào điu kiện bài toán ta được
( ) ( )
2 2 2 2x y f y c f x c y x y y c x c y+ + + = + + + + + = + +
(luôn đúng).
Bài tp 10: Tìm hàm s
:f
sao cho:
( )
( )
( )
( )
( )
2 3 ,f f x y x f f f y x x y+ = +
(1)
Li gii:
Đặt
( )
0a f=
. Trong (1), cho
( )
2
f x
y
=
ta có
( )
3 .
2
f x
a x f f f x x
= +
suy ra
f
là toàn ánh.
Ta đi chứng minh
f
là đơn ánh. Thật vây gi s
( ) ( )
f u f v=
T (1) lần lượt cho y bi
,u v
ta có
( )
( )
( )
( )
( )
2 3 (2)f f x u x f f f u x x+ = +
( )
( )
( )
( )
( )
2 3 (3)f f x v x f f f v x x+ = +
Ly (2) tr (3) ta có
( )
( )
( )
( )
2 2f f x u f f x v x+ = +
. Do
f
là toàn ánh nên suy ra được
( )
( )
( )
2f x u v f x x+ =
(4).
T (4) , s dụng phương pháp quy nạp ta có
( )
( )
( )
2 , .f x n u v f x x n+ =
(5)
T (1), cho
x
bi
( )
2x u v+
ta thu được:
( )
( )
( )
( ) ( )
( )
( )
( )
2 2 3 6 2 ,f f x u v y x u v f f f y x u v x y+ + = + +
( )
( )
( ) ( )
( )
( )
2 3 6 ,f f x y x u v f f f y x x y + = + +
(6)
T (1) và (6) ta suy ra
0u v =
. Vy
f
đơn ánh.
Trong (1) cho
0x =
ta có
( ) ( )
( )
( )
( )
( )
2 2f z y f f f y f f y y a y+ = = +
T đó (1) tr thành
( )
( )
( )
2 3 ,f f x y x f y a x x y+ = + +
Li cho tiếp
0y =
ta có :
Tài liu chuyên đề bi ng hc sinh gii
CHUYÊN ĐÊ 1: ĐẠI S VÀ GII TÍCH 51 |
( )
( )
( )
( )
( )
( )
3
2 3
f f x x f a x x
x a x f a x x
f a x a x x
f x x x
= +
+ = +
=
=
Th li ta có
( )
f x x x=
là nghim của phương trình (1).
Bài tập 11: Tìm tất cả các hàm số
:f
thỏa mãn:
2
( ) ( ( )) ( ) ( ), , ,f x y z f f x yf x f z x y z+ + = + +
(1)
Lời giải:
Giả sử tồn tại hàm số
f
thỏa đề bài ra.
Nhận thấy
( ) 0,f x x
thỏa (1)
Giả sử
( ) 0f x
tức tồn tại
u
sao cho
( ) 0f u
.
Kí hiệu
( )
,P u v
thay
x
bởi
,u y
bởi
v
vào (1). Với
x
ta có:
2
( ( )) (0) ( ( )) (0)
; ;0 ;
( ) ( )
x f f u f x f f u f
p u f u x f
f u f u
=
là toàn ánh.
( ;0;0) ( ) ( ( )) (0) ( ) (0)p x f x f f x f f x x f = + = +
(Vì
f
là toàn ánh nên
( ( ))f f x x=
) (2)
Thay (2) vào (1) ta được:
( )
2 2
2 2
(0) ( (0)) ( (0)) (0) ( (0)) ( (0))
(0) (0) ( (0)) 2 (0) ( (0)) 3
x y z f f x f y x f z f x y f x f y x f
x y x f f y x xf y f y x xf
+ + + = + + + + + + = + + +
+ = + + + + = + +
Thay y bởi vào (3) ta đưc:
(0) 0f =
khi đó từ (2) ta được:
( ) , .f x x x=
Th li, thy hàm s này không thỏa mãn(1). Do đó có duy nhất mt hàm s tha mãn yêu cu bài toán
là:
( ) 0, .f x x
Nhận xét:
Ngoài cách nhn biết tính cht toàn ánh ca
f
da vào mt vế của đẳng thc là hàm bc nht ta còn có
th chng minh
f
toàn ánh bng cách
,y R x R
tha mãn
( )f x y=
S ta có thể nhn biết được cách thay y bi
( ( )) (0)x f f u f
y
vào (1) là do
2
( , ,0) ( ) ( ( )) ( ) (0),p u y f u y f f u yf u f y R + = + +
Vi
x R
xét
( ( )) (0)
( ( )) ( ) (0)
( )
x f f u f
x f f u yf u f y
f u
= + + =
Vì vy ta thc hin
Tài liu chuyên đề bi ng hc sinh gii
52 | C CHUYÊN ĐÊ: PHƯƠNG TRÌNH HÀM
( ( )) (0)
,
( )
x f f u f
p u
f u
Để suy ra
f
là toàn ánh.
Đối với một bài toán phương trình hàm nếu ta có thể vận dụng tính chất toàn ánh của
f
điều dó
giúp ta tính được một số giá trị đặc biệt của hàm như
( ) ( )
0 ; 1f f
làm cơ sở để tìm ra đáp án của bài toán.
Bài tp 12: Tìm tt c các hàm s
:f
thỏa mãn điều kin:
( )
( )
( ) ( ) ( ) ( ) ( ) ( )
4 3 2 2 3
4 6 4 , , 1 .f y f x f x y f x y f x yf x f y x y+ = + + + +
Hướng dn:
Gi s tn ti hàm s
f
tha mãn
(1).
Trường hp 1:
( )
0f x =
. Th li ta thy
( )
0f x =
tha mãn (1).
Trường hp 2:
( ) ( )
0 0
0 , 0f x x f x
.
Ta có:
( ) ( )
( )
( ) ( ) ( )
4
4
1 , , *f y f x f y y f x y x y
+ = +
Thay
0
x x=
vào (*), ta được
( )
( )
( ) ( ) ( )
4
4
0 0
, , 2y f x f y y f x y x y
+ = +
Ta thy vế phi ca (2) là mt hàm s bc 3 nên có tp giá tr . Do đó hàm số
f
có tp giá tr
y
đều
,u v
sao cho
( ) ( )
f u f v y =
.
Thay
0y =
vào (*), ta được
( )
( )
( ) ( )
( )
4
, 0f f x f x a x a f
= + =
(3).
Thay
( )
y f y=
vào (*), ta được
( ) ( )
( )
( )
( )
( ) ( ) ( )
4 4
, ,f f x f y f f y f x f y f y x y
=
(4).
T (3) và (4) suy ra
( ) ( )
( )
( ) ( )
4
, ,f f x f y f x f y a x y
= +
hay
( ) ( )
( )
( ) ( )
4
, ,f f u f v f u f v a u v
= +
( ) ( )
4 4
, ,f y y a y f x x a x = + = +
Th li ta thy
( )
4
f x x a= +
thỏa mãn điều kin (1).
Vy
( )
0f x =
( )
4
f x x a= +
( ahng s) là các hàm s cn tìm.
Bài tập 13: Tìm tt c các hàm s
:f
thỏa mãn điều kiện:
( ( )) ( ) ( ( )) 2 ( ) ( ) 2012f x f y f x f f y xf y f y = + + +
, .x y
Li gii:
Tài liu chuyên đề bi ng hc sinh gii
CHUYÊN ĐÊ 1: ĐẠI S VÀ GII TÍCH 53 |
Cho
( )x f y=
ta thu được
2
(0) 2 ( ( )) ( ) ( ) 2012f f f y f y f y= + +
Hay
2
1 1 1
( ( )) ( ) ( ) 1006 (0)
2 2 2
f f y f y f y f= +
(2)
Và ta đoán
2
1 1
( )
2 2
f x x x c= +
(*)
Nhưng để có điều này ta cần chỉ ra
f
toàn ánh. Công việc này có vẻ khó. Ta thử thêm chút:
Thay
x
bởi
( )f x
ta có
( ( ) ( )) ( ( )) ( ( )) 2 ( ) ( ) ( ) 2012f f x f y f f x f f y f x f y f y = + + +
Và sử dụng (2) ta được
2
1 1
( ( ) ( )) ( ( ) ( )) ( ( ) ( )) (0)
2 2
f f x f y f x f y f x f y f = +
Và để thực hiện được dự đoán (*), ta cần chỉ ra với mọi
t
, tồn tại
để
0 0
( ) ( )t f x f y=
?
Hin nhiên là
f
không thể đồng nhất 0 . Do đó tồn tại
a
( ) 0f a
. Trong (1), cho
ta được
( ( )) ( ) ( ( )) 2 ( ) ( ) 2012f x f a f x f f a xf a f a = + + +
Hay
( ( )) ( ) ( ( )) 2 ( ) ( ) 2012f y f a f y f f a yf a f a = + +
Với mỗi số thực
t
, ta cần chọn
0 0
;x y
để thay vào đẳng thức này sao cho vế phải là
t
hay
0
( ( )) ( ) 2012
2 ( )
f f a f a t
y
f a
+ +
=
và vế trái là
0 0
( ) ( )f x f y
hay
0 0
( )x y f a=
Khi đó ta
0 0 0 0 0
( ) ( ) ( ( )) ( ) ( ( )) 2 ( ) ( ) 2012f x f y f y f a f y f f a y f a f a t = = + + =
Vậy
2
1 1
( ) (0)
2 2
f t t t f= +
. Thay t bởi
( )f y
và sử dụng (2) ta được
2
1 1 1
1006 (0) (0) (0) 2012 ( ) 2012,,
2 2 2
f f f f x x x x + = = = +
, thử lại thỏa.
Bài tp 14: (Hàn Quc 2003) Tìm tt c các hàm s
:f
tha mãn:
( )
( )
( ) ( ) ( )
( )
, ,f x f y f x xf y f f y x y = + +
(4)
Li gii:
Nhn thy hàm
( )
0f x
tha mãn yêu cầu bài toán. Xét trường hp
( )
0f x
Thế
( )
x f y=
vào (4) ta được
( ) ( ) ( )
( )
2
2
0
0 2
2 2
f
x
f f z z f x= + = +
.
Hay
( )
( )
( ) ( )
2
0
1 2
f x f
f f x = +
.
Thế
( )
x f z=
, vi
z
là mt s thuc thì ta được
( ) ( )
( )
( )
( )
( ) ( ) ( )
( )
f f z f y f f z f z f y f f y = + +
.
Tài liu chuyên đề bi ng hc sinh gii
54 | C CHUYÊN ĐÊ: PHƯƠNG TRÌNH HÀM
Với lưu ý là
( )
( )
( ) ( )
2
0
2 2
f y f
f f y = +
( )
( )
( ) ( )
2
0
2 2
f z f
f f z = +
Thay vào quan h hàm trên ta được
( ) ( )
( )
( ) ( )
( )
( )
2
0
2
f z f y
f f z f y f
= +
. (5)
Tiếp theo ta chng t tp
( ) ( )
| ,f x f y x y =
. Do
( )
0f x
nên tn ti mt giá tr
0
y
sao cho
( )
0
0f y a=
. Khi đó t quan h (4) ta có
( ) ( ) ( ) ( ) ( ) ( )
f x a f x xa f a f x a f x ax f a = + + = +
.
Vì vế phi là hàm bc nh c X nên
( )
xa f a+
có tp giá tr là toàn b . Do đó hiệu
( ) ( )
f x a f x
cũng
có tp giá tr là toàn b , khi
x
. Mà
( ) ( )
( ) ( )
| , |f x f y x y f x a f x x =
,
Do đó
( ) ( )
| ,f x f y x y =
. Vy t quan h (5) ta thu được
( ) ( )
2
0 ,
2
x
f x f x= +
Mt khác ta li có
( ) ( )
2
0 , .
2
x
f x f x= +
Nên
( )
0 0f =
. Th li thy hàm s
( )
2
,
2
x
f x x=
tha mãn h hàm.
Kết lun: Có hai hàm s tha mãn là
( )
2
,
2
x
f x x=
hoc
( )
0, .f x x
.
Nhn xét: Bài toán trên lấy ý tưởng t bài thi IMO 1996: Tìm tt cá các hàm s
:f
tha mãn
( )
( )
( )
( )
( ) ( )
1, ,f x f y f f y xf y f x x y = + +
.
Đáp số
( )
2
1,
2
x
f x x= +
Bài tập 15: (Việt Nam TST 2004) Tìm tất các giá trị của a sao cho tồn tại duy nhất một hàm số
:f
thỏa mãn điều kiện:
( )
( )
( )
( )
2
2
, , .f x y f y f x ay x y+ + = +
(1)
Lời giải:
Tài liu chuyên đề bi ng hc sinh gii
CHUYÊN ĐÊ 1: ĐẠI S VÀ GII TÍCH 55 |
Giả sử tồn tại hàm số
( )
f x
thỏa yêu cầu bài ra.
Khi
0a =
, từ (1) ta được
2 2
( ( )) ( ( )) , , .f x y f y f x x y+ + =
khi đó ta có hai hàm số thỏa mã là
( ) 0f x
( ) 1 0f x a =
(loại).
Khi
0a
. Vì vế phải là hàm bậc nhất nên y có tập giá trị là . Do đó
f
là toàn ánh
: ( ) 0b f b⎯⎯ =
.
Tìm
b
. Thay
y
bởi
b
vào (1) ta được :
2 2
( ) ( ( )) , .f x b f x ab x+ = +
(2)
Thay
x
bởi
x
vào (2) ta được :
2 2
( ) ( ( )) , .f x b f x ab x+ = +
Từ (2) và (3) ta được :
2 2
( ) ( )
( ( )) ( ( )) ( ) 0
( ) ( )
f x f x
f x f x f b
f x f x
=
= ⎯⎯ =
=
Thay
y
bi
b
vào (1) ta được :
2 2
( ) ( ( )) ,f x b f x ab x =
(4)
T (3), (4)
2 2
( ) ( ) 2f x b f x b ab + =
(5)
Thay
x
bi
0
vào (5) ta được:
( ) ( ) 2 2 0 0 ( 0)f b f b ab ab b a = = =
Vy
( ) 0 0.f b b= =
Thay
x
bởi 0 vào (1) ta được :
( ( ))f y f y ay+ =
(6)
Thay
y
bởi 1 vào (6) ta được:
( )
( )
1 1f f a+ =
(8)
Thay
y
bởi 0 vào (1) ta được :
( )
( )
( )
2
2
f x f x=
(7)
Thay
x
bởi 1 vào (7) ta được :
( ) ( )
( )
( )
( )
2
1 1
1 1
1 0
f
f f
f
=
=
=
( )
0 0f x x= =
nên ta nhn
(1) 1f =
Thay
( )
1 1f =
vào (8) ta được
( )
2f a=
Ta có :
2 2 2 2 2
2
( (2)) (2 ) (4) (( 2) 2) (( 2) ) (2) 2
0
a
a f f f f f a f a a
a
=
= = = = + = + = + =
=
0a
nên ta nhn
2.a =
Vy
2.a =
Khi đó (1) trở thành:
2 2
( ( )) ( ( )) 2 , ,f x y f y f x y x y+ + = +
(9)
Thay
y
bi
2
( ( ))
2
f x
vào (9) ta được:
2 2 2 2
2 2 2 2
2 2
2
( ( )) ( ( )) ( ( )) ( ( ))
( ( )) ( ( )) 0
2 2 2 2
( ( )) ( ( ))
0,
2 2
f x f x f x f x
f x f f x f x f x f
f x f x
x f x
+ = + =
+ =
Vì (
( ) 0 0f x x= =
)
Tài liu chuyên đề bi ng hc sinh gii
56 | C CHUYÊN ĐÊ: PHƯƠNG TRÌNH HÀM
2 2
2
( ( )) ( ( ))
2 2
f x f x
f x
=
(10)
Hay
2 2
( ) ( ) ,f y x y f y y x x= + =
(11)
Thay
y
bi
2
( ( ))
2
f x
( )f y y+
bi
2
y
do (11), kết hp với (7) khi đó từ (9) ta được:
2 2 2 2 2 2
( ) ( ( )) ( ( )) ( ) ( ), ,f x y f x f y f x f y x y = =
(12)
T (12) thay x bởi 0 ta được:
2 2
( ) ( )f y f y f = ⎯⎯
là hàm s l.
Khi đó t (1) ta có:
( ) ( ( )) ( ) ( ) ( ) ( ), ,f x y f x y f x f y f x f y x y f+ = = = +
cng tính.
Ta li có:
2 2
( ( )) ( )f x f x=
nên
2 2 2 2 2
2 2 2 2
( ( )) ( ) ( ( ) ( )) ( 2 )
( ( )) 2 ( ) ( ) ( ( )) ( ) ( ) (2 )
( ). ( ) ( )
f x y f x y f x f y f x xy y
f x f x f y f y f x f y f xy
f x f y f xy
+ = + + = + +
+ + = + +
=
f
nhân tính.
Hàm f va cng tính va nhân tính nên
( )f x x
th li ta thy hàm
( )f x x
tha yêu cu bài toán.
Nhận xét:
Đây là một bài toán khó, tính chất toàn ánh của
f
sử dụng khá hiệu quả; dấu hiệu nhận biết tính chất
toàn ánh ở đây chính là vế phải đẳng thức là hàm bậc nhất, khi khẳng định f là toán ánh ta luôn
: ( ) 0b R f b =
, từ đó ta đi tìm b .
Từ điều kiện
(0) 0f =
ta nhận xét được một loạt tính chất đặc biệt của
f
như
(0) 0f =
,
2 2
( ) ( ( ))f x f x=
,
tính chất f hàm số lẻ; tính nhân tính; cộng tính của hàm f làm cơ sở để tìm ra kết quả của bài toán.
Nếu ban đầu ta không tìm cách vận dụng tính chất toàn ánh của
f
thì bài này khó có thế giải quyết được.
Vì vậy khi giải một bài toán phương trình hàm thì việc nhận biết và vận dụng các tính chất của hàm
f
rất quan trọng nó sẽ giúp việc giải quyết bài toán một cách dễ dàng hơn.
Bài tp 16: Tìm hàm s
:f
thỏa mãn điều kin:
( )
( )
( ) ( )
2 , ,f x f y f x x f y x y = + +
. (6)
Li gii:
Nhn thy hàm
( )
0f x
không tha mãn yêu cu. Xét
( )
0f x
.
Thay
x
bi
( )
f y
vào (6) ta được
( )
( )
( )
( )
0
2
f
f f y f y= +
Li thay
x
bi
ta được
Tài liu chuyên đề bi ng hc sinh gii
CHUYÊN ĐÊ 1: ĐẠI S VÀ GII TÍCH 57 |
( )
( )
( ) ( )
( )
( ) ( )
( )
( )
( ) ( )
( ) ( )
( )
( )
2
0
2
2
0
f f x f y f f x f x f y
f
f x f x f y
f x f y f
= + +
= + + +
= +
Tuy nhiên vic chng minh tp
( ) ( )
| ,f x f y x y
có tp giá tr chưa thực hiện được.
T đây ta
( ) ( )
( )
( ) ( ) ( )
( )
( ) ( )
( )
( ) ( ) ( )
( ) ( )
( )
( ) ( )
( ) ( )
( )
( )
2
2
2 2 0
2 2 0 .
f f x f y f f x f y f y
f f x f y f x f y f y
f x f y f f x
f x f y f
=
= + +
= + +
= +
Ta s chng minh tp
( ) ( )
2 | ,f x f y x y
bng . Tht vy tn ti giá tr
0
y
sao cho
( )
0
0f y a=
. Khi đó thay
0
y y=
vào (6) ta có
( ) ( )
2 ,f x a f x x a x = +
Mà khi
x
thif
x a+
có tp giá tr . Chng t tp
( ) ( )
2 | ,f x f y x y =
. Mà
( ) ( )
( ) ( )
2 | , |f x f y x y f x a f x x
nên
( ) ( )
2 | ,f x f y x y =
. Do đó từ (c) ta kết
lun
( )
,f x x x=
. Thay vào (6) ta được
( )
0 0f =
Kết lun: Hàm s
( )
,f x x x=
tha mãn yêu cu bài toán.
Bài tp 17: Tìm tt c các hàm
:f
tha mãn:
( )
( )
( ) ( )
( )
( )
4
4
, , . 1f x f y f x x f y x x y+ = +
Li gii:
Nhn thy
0f
là mt nghim hàm. Xét
0f
, khi đó tn ti
a
để
( )
0f a
. Trong (1) cho
y a=
ta được:
( )
( )
( ) ( )
( )
( )
4
4
, , . 2f x f a f x x f a x x y+ = +
Chú ý VT(2) là đa thc bậc 3 đối vi
x
nên tp giá tr ca
( )
( )
( )
f x f a f x+
.
Suy ra
( )
( )
( )
f x f a f x+
là toàn ánh. Do đó với mi
t
đều tn ti
,u v
tha
( ) ( )
.t f u f v=
Trong (1) thay
x
bi
( )
f y
ta được
( ) ( )
( )
( ) ( )
( )
( ) ( ) ( )
4 4
0 , 0 , . 3f f f y f y y f f y f y f y = = +
Trong (1) thay
x
bi
( )
f x
và s dụng (3) ta được
Tài liu chuyên đề bi ng hc sinh gii
58 | C CHUYÊN ĐÊ: PHƯƠNG TRÌNH HÀM
( ) ( )
( )
( )
( )
( ) ( )
( )
( )
( ) ( )
( )
( ) ( )
( )
( ) ( )
4
4
4
, ,
0 , , . 4
f f x f y f f x f x f y f x x y
f f x f y f x f y f x y
+ = +
+ = + +
Trong (4) cho
,x u y v= =
được
( ) ( )
( )
( ) ( )
( )
( ) ( ) ( )
( ) ( )
4
4
4
0 0 ,
0 ,
f f u f v f u f v f f t t f t
f x x f x
+ = + + = +
⎯⎯ = +
Thay vào (1) được
( )
0 0.f =
Vy bài toán có 2 nghim hàm
( )
0,f x x=
hoc
( )
4
1,f x x x= +
.
Tài liu chuyên đề bi ng hc sinh gii
CHUYÊN ĐÊ 1: ĐẠI SGII TÍCH 59 |
TÀI LIỆU THAM KHẢO
[1] Nguyn Trng Tun, Bài toán hàm s qua các k thi Olympic. NXBGD, 2005.
[2] Nguyn Tài Chung, Chuyên khảo phương trình hàm. NXB Đại hc quc gia Hà Ni, 2013.
[3] Trần Nam Dũng (CB), Các k thi toán VMO Li gii và bình lun. NXB Thế gii, 2017.
[4] Titu Andreescu, Iurie Boreico, Oleg Mushkarov, Nikolai Nikolov, Topics in Functional
Equations. XYZ Press 2014.
[5] Đề thi chọn ĐT học sinh gii ca mt s tnh.
[6] Mt s tài liu trên mng internet.
| 1/59

Preview text:

Tài liệu chuyên đề bồi dưỡng học sinh giỏi
ĐẠI SỐ VÀ GIẢI TÍCH Chuyên đề 1
PHƯƠNG TRÌNH HÀM TRÊN
PHƯƠNG PHÁP THẾ VÀ PHƯƠNG PHÁP SỬ DỤNG TÍNH CHẤT ÁNH XẠ
TRONG VIỆC GIẢI BÀI TOÁN PHƯƠNG TRÌNH HÀM TRÊN
Trong chương trình chuyên toán ở các trường THPT chuyên Phương trình hàm là một chuyên đề quan
trọng. Hiện nay tài liệu về phương trình khá phong phú. Tuy vậy, việc giải được phương trình hàm vẫn là
vấn đề khó đối với nhiều học sinh. Trong chuyên đề nhỏ này, chúng tôi sẽ trình bày hai phương pháp
thông dụng và quan trọng để giải phương trình hàm trên tập . Đó là Phương pháp thếPhương pháp
sử dụng tính chất ánh xạ.

I. Phương pháp thế trong giải phương trình hàm
1. Một số lưu ý khi sử dụng phương pháp thế
Quan sát cấu trúc của phương trình xem giữa các biến có tính đối xứng không? Nếu trong phương
trình có tính đối xứng giữa x y thì ta thường hoán vị các biến này cho nhau.
Thay các biến bởi các giá trị thích hợp thoả mãn các điều kiện bài toán. Điều cần chú ý là các giá trị của
biến phải thuộc tập xác định của hàm số. Ta thường chọn giá trị cho biến sao cho sau khi thay ta thu
được một phương trình hàm đơn giản hơn hoặc có thể nhận được dạng của hàm số (tuyến tính, bậc hai,
mũ, logarit,…). Trong trường hợp f cộng tính ta có thể thay x bởi x +1 và quy giá trị hàm số về f ( ) 1 .
Một vài phép thế đặc biệt: x = y = 0, x = y = 1
 ; x = 0, y = 0, x = 1, y = 1, x =  y, x =  f (y),...
Tính các giá trị đặc biệt của hàm số f ( x) , chẳng hạn: f (0), f ( 1  .).. 2. Các ví dụ:
Ví dụ 1: Cho hàm số f thoả mãn điều kiện: f (0) = 2 và f ( x + y) = f ( y) + x, x
 , y  . Tính f (1998). Lời giải:
Giả sử tồn tại hàm f thoả mãn điều kiện đề bài. Cho y = 0 , ta có f (x) = f (0) + x = 2 + x, x  .
Thử lại thấy thoả mãn. Do đó: f (1998) = 2 +1998 = 2000
Ví dụ 2: (Australia 1995) Tìm tất cả các hàm f : (0;+) → thoả mãn các điều kiện f ( ) 1 1 = và 2
( ) = ( )  3  + ( )  3  f xy f x f f y f     , x  , y  0. (1)  y   x
CHUYÊN ĐÊ 1: ĐẠI SỐ VÀ GIẢI TÍCH 1 |
Tài liệu chuyên đề bồi dưỡng học sinh giỏi Lời giải:
Cho x = 1, y = 3 thay vào (1) ta được: 2 1  1  1 f (3) 2
= + f (3)   f (3) −  = 0  f (3) = x, y +   4  2  2 1  3  1  3 
Thay x = 1 vào (1) ta được: f ( y) = f
+ f ( y) → f = f ( y)     y +   2  y  2  y
Khi đó (1) trở thành: f ( xy) = 2 f ( x) f ( y) x, y +   (2)  1 f (x) = 3 1    2
Thay y bởi 3 vào (2) ta có: 2
f (3) = 2 f (x) f
→ = 2 f (x)     xx  2 1  f (x) = −  2 1 1 1
Thay y = x vào (2) có: f ( 2 x ) 2
= 2 f ( x) = 2. = → f (x) = , x   0 4 2 2
Thử lại thấy thoả mãn.
Ví dụ 3: (Nhật Bản 2012)Tìm tất cả các hàm số f : → sao cho: 2
f ( f (x + y) f (x y)) = x yf (y) x  , y  (1) Lời giải:
Từ (1) cho x = y = 0 ta thu được 2
f ( f (0)) = 0. Từ (1) cho 2
x = 0, y = f (0), kết hợp với kết quả f ( 2
f (0)) = 0 ta thu được f (0) = 0.
Từ (1) cho x = y có: 2
x xf (x) = 0  f (x) = x , x  0.
Hơn nữa f (0) = 0 , suy ra: f ( x) = x, x  . Thử lại ta thấy thoả mãn.
Ví dụ 4: Tìm tất cả các hàm số f : →
thoả mãn: f (xf ( y) + x) = xy + f (x) x  , y  (1) Lời giải:
Trong (1) thay x = 1 , ta được f ( f ( y) +1) = y + f (1) , y  (2)
Trong (2) thay y bởi: 1 − − f ( )
1 ta có: f ( f ( 1 − − f ( ) 1 ) + ) 1 = 1 − Đặt: 1+ f ( 1 − − f ( ) 1 ) = ; a f (0) = b
Ta có: f ( xf (a) + x) = f (0) = .
b Hay: b = f ( xf (a) + x) = ax + f ( x) ⎯
⎯ → f (x) = −ax +b
Thay biểu thức của f ( x) vào (1) ta được: −a(xf ( y) + x) + b = xy ax + b, x  , y
 −ax(−ay + b) + x+ b = xy ax + b, x  , y  2
a xy abx ax + b = xy ax + b, x  , y  a =1  2 a =1 b  = 0
Bằng cách đồng nhất các hệ số ta được:   ab a a  + = a = 1 −   b  = 0
Vậy f ( x) = x, x
f ( x) = −x, x  . Thử lại thấy thoả mãn.
Ví dụ 5: (Serbia 2014, Đồng Nai 2018) Xác định tất cả các hàm số f : → thỏa mãn
2 | C CHUYÊN ĐÊ: PHƯƠNG TRÌNH HÀM
Tài liệu chuyên đề bồi dưỡng học sinh giỏi
f ( xf ( y) − yf ( x)) = f ( xy) − xy với mọi x, y  .(1) Lời giải:
Cho y = 0 được f ( xf (0)) = f (0) . Nếu f (0)  0 → f ( x) là hàm hằng. Thử lại thấy không thỏa mãn. Do đó f (0) = 0.
Cho x = y được f ( 2 x ) 2
x = f (0) = 0 → f (x) = x, x   0 Giả sử a
 : f (a) = 0 . Cho x = a, y =1 được f (af ( )
1 ) = f (a) − a = −a f (a) = −a → 0 = −a a = 0
Với x, y  0 ta có
f ( xf ( y) − yf ( x)) = f ( xy) − xy f ( xf ( y) − yf ( x)) = xy xy = 0
xf ( y) − yf (x) = 0 → f (x) = cx, x   0.
Xét x  0  y thay vào (1) có: f ((1− c) xy) = f ( xy) − xy = (c − ) 1 x . y (2) Nếu c = 1 ⎯⎯
f (x) = x, x   .
Nếu c  1, do f ((1− c) xy) (
 1−c)xy,c(1−c)x
y nên từ (2) suy ra c = 1 − ⎯⎯
f (x) = x , x   .
Vậy có 2 hàm thỏa mãn bài toán f ( x) = x, x
  . hoặc f (x) = x , x   .
Ví dụ 6: Tìm hàm f : → thỏa mãn:
x y f ( x + y) − ( x + y) f ( x y) = xy ( 2 2 ( ) 4 x + y ) x  , y  ( ) 1 . Lời giải: Ta có: ( )
1  ( x y) f ( x + y) − ( x + y) f ( x y) = = (  
x + y) − (x y) + 
 ( x + y) + ( x y) 1  
  (x + y) + (x y) 2 1  −  
(x + y) − (x y) 2    4 4  u  = x y Đặt  ta được:
v = x + y
vf (u) − uf (v) 1
= (u + v)(u v)((u + v)2 −(u v)2) 4
vf (u) − uf (v) 3 3
= u v v u v ( f (u) 3
u ) = u ( f (u) 3 − v )
+ Với uv  0 ta có: f (u) 3 − u f (v) 3 − v f (u) 3 −u * = uv  →
= a f (u) 3 ,
= au + u , u   0 u v u
CHUYÊN ĐÊ 1: ĐẠI SỐ VÀ GIẢI TÍCH 3 |
Tài liệu chuyên đề bồi dưỡng học sinh giỏi
+ Với u = 0; v  0 suy ra: f (u) 3
u =  f (u) 3 0
= u f (0) = 0. Hàm ( ) 3
f u = au + u thỏa mãn f (0) = 0 . Vậy f (u) 3
= au + u , u   Hàm số cần tìm là: ( ) 3
f u = ax + x (a  ) . Thử lại thấy đúng.
Ví dụ 7: (Áo 1996) Tìm tất cả các hàm số f :
→ thỏa mãn điều kiện 2
x f ( x) + f ( − x) 4 1
= 2x x , x   Lời giải:
Thay x bởi 1− x ta được
( − x)2 f ( − x)+ f (x) = ( − x)−( − x)4 1 1 2 1 1 Như vậy ta có hệ 2 x f
(x)+ f (1− x) 4 = 2x − 2 (   1− x
)2 f (1− x)+ f (x) = 2(1− x)−(1− x)4 Ta có D = ( 2 x x − )( 2 1 x x + ) 1 và D = ( 2 − x )( 2 x x − )( 2 1 1 x x + ) 1 . Vậy .
D f ( x) = D , x   . Từ đó ta x x
có nghiệm của bài toán là 2 1
 − x : x a, x b
f ( x) = c  : x = a
( c là hằng số tùy ý),  4 2
2a a a : 2 = b
Với a, b là nghiệm của phương trình 2
x x −1 = 0
Nhận xét: bài toán trên được dùng một lần nữa trong kì thi VMO 2000, bảng B.
Ví dụ 8: (VMO 2005) Hãy xác định tất cả các hàm số f :
→ thỏa mãn điều kiện
f ( f ( x y)) = f ( x) f ( y) − f ( x) + f ( y) − xy, x  , y  (8) Lời giải:
Thế x = y = 0 vào (8) ta được
f ( f ( )) = ( f ( ))2 0 0
Thế x = y vào (8) và sử dụng kết quả trên thì
( f (x))2 = ( f ( ))2 2 0 + x , x   2 2
Suy ra ( f ( x)) = ( f (−x)) → f ( x) = f (−x) , x   .
4 | C CHUYÊN ĐÊ: PHƯƠNG TRÌNH HÀM
Tài liệu chuyên đề bồi dưỡng học sinh giỏi
Thế y = 0 vào (8) được
f ( f ( x)) = f (0) f ( x) − f ( x) + f (0), x   (*)
Thế x = 0, y = −x vào (8) được
f ( f ( x)) = f (0) f (−x) + f (−x) − a, x   .
Từ hai đằng thức trên ta có
f (0)( f (−x) − f ( x)) + f (−x) + f ( x) = 2 f (0), x   . (9)
Giả sử tồn tại x  0 sao cho f ( x = f x , thì thế x = x vào (9) ta có 0 ) ( 0) 0 0
f ( x = f 0 0 ) ( )
→ ( f (x ))2 = ( f (0))2 0
→ ( f (0))2 + x = ( f (0))2 2 2 + 0 0 → x = 0 0 Suy ra mâu thuẫn
Vậy f ( x) = − f ( x), x
  , từ điều này kiết hợp với (9) ta có
f (0)( f ( x) − ) 1 = 0, x  
Từ đây suy ra f (0) = 0 , vì nếu ngược lại thì f ( x) = 1, x
  0 , trái với điều kiện f là hàm lẻ. Từ đây ta nhận
được quan hệ quen thuộc x = f x = − f f x = − f x = x 0 ( 0 ) ( ( 0)) ( 0 ) 0
Vô lý. Vậy chứng tỏ f ( x) = −x, x
  . Thử lại thấy hàm này thỏa mãn bài toán.
Nhận xét: Bài toán trên cho kết quả là hàm chẵn f ( x) = −x . Nếu vẫn giữa nguyên vế phải và để nhận được
hàm lẻ f ( x) = x , ta sửa lại dữ kiện trong vế trái như trong ví dụ sau
Ví dụ 9: Tìm tất cả các hàm số f :
→ thỏa mãn điều kiện
f ( f ( x) − y) = f ( x) − f ( y) + f ( x) f ( y) − xy, x  , y Lời giải:
Thế y = 0 ta được
f ( f ( x)) = f ( x) − f (0) + f (0) f ( x), x   (10)
Thế y = f ( x) và sử dụng kết quả trên, ta được
CHUYÊN ĐÊ 1: ĐẠI SỐ VÀ GIẢI TÍCH 5 |
Tài liệu chuyên đề bồi dưỡng học sinh giỏi
f (0) = f ( x) − f ( f ( x)) + f ( x).( f ( x)) − xf ( x) (*)
= f (0) − xf (0) f (x) + ( f (x))2 + f (0).( f (x))2 − xf (x), 2 2 Hay 2
f (0). f (x) + ( f (x)) + f (0).( f (x)) − xf (x) = 0, x   .
Thế x = 0 vào đẳng thức trên ta được
( f ( ))2 −( f ( ))2 0 0
= 0 → f (0) = 0 hoặc f (0) =1.
Nếu f (0) = 0 thì thay vào (10) ta có f ( f ( x)) = f ( x), x
  , thay kết quả này vào trong (*) ta có
f ( x) = x .
Nếu f (0) = 1thay vào (10) ta có f ( f ( x)) = 2 f ( x) −1, thay vào trong (*) ta có f ( x) 1 = x +1. 2
Kết luận: thay vào ta thấy chỉ có hàm số f ( x) = x, x
  là thỏa mãn yêu cầu.
Ví dụ 10: Tìm tất cả các hàm số f : → thỏa mãn
f ((x y)2 ) = ( f (x))2 − xf ( y) 2 2 + y , x  , y  . Lời giải: 2
Thay x = y = 0 thì ( f (0)) = ( f (0)) → f (0) = 0 hoặc f (0) = 1
Trường hợp 1: Nếu f (0) = 0, thì thay x = y vào điều kiện ban đầu ta được
f ( ) = ( f ( x))2 − xf ( x) + x = ( f ( x) − x)2 2 0 2
f (x) = x, x   .
Nhận thấy hàm số này thỏa mãn.
Trường hợp 2: Nếu f (0) = 1 thì lại vẫn thay x = y = 0 ta nhận được, với mỗi x  thì hoặc là f ( x) = x +1
hoặc là f ( x) = x −1. Giả sử tồn tại giá trị a sao cho f (a) = a −1. Khi đó thay x = a, y = 0 ta được f ( 2 a ) 2 = a − 4a +1
Nhưng ta lại có hoặc là f ( 2 a ) 2
= a +1 hoặc f ( 2 a ) 2
= a −1. Do đó ta phải có hoặc là 2 2
a − 4a +1 = a +1 hoặc 1 2 2
a − 4a +1 = a −1, tức a = 0 hoặc a =
. Tuy nhiên kiểm tra đều không thỏa. 2
Vậy hàm số thỏa mãn yêu cầu là f ( x) = x, x
  hoặc là f (x) = x +1, x   .
Ví dụ 11: Tìm tất cả các hàm số f :
→ thỏa mãn điều kiện
f ( x y) + y ( ( f (x))2 3 3 2 3
+ y ) = f (x + f (y)), x  , y  . Lời giải: Thay 3 y = x ta có
6 | C CHUYÊN ĐÊ: PHƯƠNG TRÌNH HÀM
Tài liệu chuyên đề bồi dưỡng học sinh giỏi
f ( ) + x ( ( f (x))2 3 6 + x ) = f ( 3 0 2 3
x + f ( x)), x  
Thay y = − f ( x) ta được
f ( x + f ( x)) − f ( x)( ( f (x))2 + ( f (x))2 3 2 3
)= f (0), x .
Từ hai đẳng thức trên ta được
x ( ( f (x))2 + x ) = ( f (x))3 3 6 2 3 8 , x   . Do đó
0 = 4 ( f ( x))2 − x (3( f (x))2 3 6 + x )
= (4( f (x))3 − 4( f (x))2 .x )+( f (x))2 3 3 9 .x x ) = ( ( ) − )(  x   f x x
4 ( f ( x)) + x ( f ( x) + x )) = ( f (x) − x )2 f (x) 3 2 15 3 3 3 3 6 +  + x .  4  16  2 3  x  15
Chú ý rằng  2 f ( x) 6 +  +
x = 0 thì x = 0, f (0) = 0 . Bởi vậy trong mọi trường hợp ta đều có ( ) 3 f x = x  4  16
. Thử lại thấy hàm số này thỏa mãn bài toán.
3. Bài tập vận dụng
Bài tập 1: (Slovenia 1999)Tìm tất cả các hàm f : → thỏa mãn
f ( x f ( y)) = 1− x y, x  , y  ( ) 1 Lời giải:
Thay x bởi f ( y) trong ( )
1 được f (0) = 1− f ( y) − y f ( y) = − y f (0) +1 (2)
Cho y = 0 thay vào (2) được f ( ) 1 0 = . Suy ra f ( y) 1 = −y + y   2 2
Bài 2: (Khánh Hòa 2017) Tìm tất cả các hàm f : → thỏa mãn:
f ( xy) + f ( x y) + f ( x + y + )
1 = xy + 2x +1 x  , y  . Lời giải:
Đặt g ( x) + x = f (x) thì phương trình hàm đã cho trở thành:
g ( xy) + g ( x y) + g ( x + y + ) 1 = 0 x  , y  . ( ) 1
Trong (1) lần lượt cho x = 0, y = 0; x = 0, y = 1; x = 0, y = 2; x = 1, y = 1; → g (0) = 0.
Trong (1) cho y = 0 → g ( x) + g ( x + ) 1 = 0 x   . (2)
CHUYÊN ĐÊ 1: ĐẠI SỐ VÀ GIẢI TÍCH 7 |
Tài liệu chuyên đề bồi dưỡng học sinh giỏi Trong (1) cho y = 1
− → g (−x) + g (x + )
1 + g ( x) = 0 x   . (3)
Từ (2) và (3) suy ra g (−x) = 0 x
  → g (x) = 0 x
  → f (x) = x x
  . Thử lại thấy thỏa mãn.
Bài 3: Tìm tất cả các hàm số f :
→ , biết rằng f là hàm số chẵn và thỏa mãn:
f ( xy) − f ( x) f ( y) = 2014( f ( x + y) − 2xy − )
1 với mọi x, y  .. Lời giải:
Từ (1) cho y = 0 , ta có:
f (0) − f ( x). f (0) = 2014( f ( x) − )
1  ( f ( x) − )
1 ( f (0) + 2014) = 0 với mọi x  . Nếu f (0)  2
− 014 thì f (x) =1 với mọi x R . Khi đó f không thỏa mãn (1) Do đó f (0) = 2 − 014.
Từ (1) thay x bởi x y bởi x , ta được:
f ( x ) − ( f ( x))2 2 = ( f ( x) 2 2014 2 − 2x − ) 1 . (2)
Từ (1) thay x bởi x y bởi −x , ta được: f ( 2
x ) − f (x) f (−x) = ( f ( ) 2 2014 0 + 2x − ) 1 . (3)
f là hàm số chẵn nên viết (3) lại như sau:
f ( x ) − ( f ( x))2 2 = ( f ( ) 2 2014 0 + 2x − ) 1 . (4)
Lấy (4) trừ (2) vế theo vế ta được: f ( x) 2 2
= 4x + f (0) với mọi x  . Suy ra: f ( x) 2 = x + f ( ) 2
0 = x − 2014 với mọi x  . .
Bài 4: (Thanh Hóa 2017) Tìm tất cả các hàm f : → thỏa mãn:
f ( f ( x) + f ( y)) = f ( x ) + x f ( y) + ( f ( y))2 2 2 2 x  , y  . ( ) 1 Lời giải: Trong (1) lần lượt cho:
x = y = → f ( f ( ) + f ( )) = f ( ) + f ( ) 2 1, 0 1 0 1 2 0 + f (0). (2)
x = y = → f ( f ( ) + f ( )) = f ( ) 2 0, 1 0 1 0 + f ( ) 1 . (3)
8 | C CHUYÊN ĐÊ: PHƯƠNG TRÌNH HÀM
Tài liệu chuyên đề bồi dưỡng học sinh giỏi
Từ (2) và (3) suy ra f ( )
1 = − f (0) hoặc f ( ) 1 = f (0) +1. • Nếu f ( )
1 = − f (0) ta có f ( ) = f ( ) + f ( ) 2 0 1 2
0 + f (0) → f (0) = f ( ) 1 = 0.
Trong (1) cho x = 0 có: f ( f ( y)) 2
= f ( y) y   .
Trong (1) cho x = 1 có: f ( f ( y)) = f ( y) 2 2
+ f ( y) y   .
Từ đó suy ra f ( y) = y y   . • Nếu f ( )
1 = f (0) +1 , trong (1) thay x bởi y , y bởi x được
f ( x ) + x f ( y) + ( f ( y))2 = f ( y ) + y f ( x) + ( f ( x))2 2 2 2 2 2 2 x  , y  . (4)
Trong (4) lần lượt cho x = 0, x = 1 ta được:
f ( ) + ( f ( y))2 = f ( y ) + y f ( ) + ( f ( ))2 2 2 0 2 0 0 y  
f ( ) + f ( y) + ( f ( y))2 = f ( y ) + y f ( ) + ( f ( ))2 2 2 1 2 2 1 1 y   .
Từ hai phương trình trên kết hợp f ( )
1 = f (0) +1 ta được f ( x) 2
= x + f (0) x   .
Thử lại thấy f (0) = 0. Hàm ( ) 2 f x = x x
  thỏa mãn. Vậy có hai hàm thỏa mãn f (x) = 0 x   hoặc ( ) 2 f x = x x   .
Bài 5: (Chuyên ĐH Vinh 2017) Tìm tất cả các hàm f : → thỏa mãn: 1. f ( ) 1  0 2. f ( xy − )
1 + 2 f ( x) f ( y) = 3xy −1 x  , y  . ( ) 1 Lời giải:
Trong (1) cho y = 1 có: f (− )
1 + 2 f (0) f ( x) = 1 − x
  . Từ đây suy ra f (0) = 0 vì nếu ngược lại ta có
f ( x) = c x
  (c là hằng số), không thỏa mãn đề bài.
Trong (1) cho x = 1, y = 1 ta có f ( ) 2 0 + 2 f ( ) 1 = 2 → f ( ) 1 = 1 (do f ( ) 1  0 )
Trong (1) thay x bởi x +1 , cho y = 1 có f ( x) + 2 f ( x + ) 1 = 3x + 2 x   . (2)
Trong (2) cho x = −1 → f (− ) 1 = −1.
Trong (1) thay x bởi −x −1 , cho y = −1 có f ( x) − 2 f (−x − ) 1 = 3x + 2 x   . (3)
Từ (2),(3) suy ra f ( x) = − f (−x) x   .
Trong (1) cho y = −1 có f (−x − )
1 + 2 f (−x) = 3
x −1 → f (x + )
1 + 2 f ( x) = 3x +1 x   . (4)
CHUYÊN ĐÊ 1: ĐẠI SỐ VÀ GIẢI TÍCH 9 |
Tài liệu chuyên đề bồi dưỡng học sinh giỏi
Từ (3),(4) suy ra f ( x) = x x
  . Thử lại thấy thỏa mãn.
Bài 6: (Chuyên Sư phạm HN 2018) Tìm tất cả các hàm f : → thỏa mãn:
f (( x y) f ( x) − f ( y)) + ( x + )
1 f ( y x) + x = 0 x  , y  ( ) 1 Lời giải: f (0) = 0
Trong (1) cho x = y = 0 ta có: 2
f (0) = f (0) →   f  (0) = 1
Trường hợp 1: f (0) = 0.
Trong (1) cho x = 0 ta có: f ( y) = 0 y
  . Thử lại thấy không thỏa mãn.
Trường hợp 2: f (0) = 1.
Trong (1) cho y = −2x ta có: f ( x) f (−x) = 1− xf (−x) + x x   .
Trong (1) thay x bởi −x ,thay y bởi −2x ta có: f ( x) f (−x) = 1+ xf ( x) − x x   . (2)
Từ đây suy ra f (−x) = 2 − f ( x) x
  0 . Mà f (0) =1→ f ( 0
− ) = 2 − f (0) nên
f (−x) = 2 − f ( x) x   . (3)
Từ (2) và (3) được ( f ( x) − )
1 ( f ( x) + x − ) 1 = 0 x   .
Gỉa sử tồn tại a, b  0 thỏa f (a) = 1 và f (b) = 1− b . Mặt khác trong (1) cho x = b, y = a b có  = −  = f (a + b) 1 1 b b 0 =1− b →  →  , vô lý. 1−
 (a + b) =1− ba = 0
Do đó f ( x) =1 x
  hoặc f (x) = −x +1 x
  . Thử lại thấy thỏa mãn.
Bài 7: (Bắc Ninh 2018) Tìm tất cả các hàm f : → thỏa mãn:
xf ( x + xy) = xf ( x) + f ( 2
x ) f ( y), x  , y  ( ) 1 Lời giải:
Trong (1) cho x = y = 0 ta có: f (0) = 0  f ( ) 1 = 1
Trong (1) cho x = y = 1 − ta có:   f  (− ) 1 = 0
Trường hợp 1: f (− ) 1 = 0 .
10 | C CHUYÊN ĐÊ: PHƯƠNG TRÌNH HÀM
Tài liệu chuyên đề bồi dưỡng học sinh giỏi
Trong (1) cho y = −1 ta có: xf ( x) = 0 x
  . Kết hợp f (0) = 0 được f (x) = 0 x   . Thử lại thấy thỏa mãn.
Trường hợp 2: f ( ) 1 = 1.
Trong (1) cho x = 1, y = −1 ta có: f (− ) 1 = 1 −
Trong (1) cho y = −1 ta có: f ( 2
x ) = xf ( x) x
  . Kết hợp f (0) = 0 được f (x) = − f (−x) x   .
Trong (1) cho x = 1 ta có: f ( y + ) 1 = f ( y) +1 y   . Do f ( 2
x ) = xf ( x) x
  . nên (1) trở thành
f ( x + xy) = f ( x) + f ( x) f ( y) = f ( x)( f ( y) + )
1 = f ( x) f ( y + ) 1 x  , y
f (x) f ( y) = f (xy) x  , y  (2)
Suy ra f ( x + xy) = f ( x) + f ( xy) x  , y  . (3) y
Với x  0 trong (3) thay y bởi
được f ( x + y) = f ( x) + f ( y) x   0, y  . x
Kết hợp f (0) = 0 được f ( x + y) = f ( x) + f ( y) x  , y  . (4)
Từ (2) và (4) kết hợp f ( )
1 = 1 được f ( x) = x x
  . Thử lại thấy thỏa mãn. Vậy bài toán có 2 nghiệm
hàm f ( x) = 0, x
  hoặc f (x) = x, x   .
Bài 8: (VMO 2002). Hãy tìm tất cảc các hàm số f ( x) xác định trên tấp số thực và thỏa mãn hệ thức
f ( y f ( x)) = f ( 2002 x
y) − 2001. .y f (x), x  , y  (1) Lời giải:
Thế y = f ( x) vào (1) ta được
f ( ) = f ( xf (x)) − ( f (x))2 2002 0 202. , x   (2) Lại thay 2002 y = x vào (1) thì f ( 2002 x
f (x)) = f ( ) 2002 0 − 2001.x
. f ( x), x   (3)
Lấy (2) cộng với (3) ta được
f ( x)( f ( x) 2002 + x ) = 0, x  
Từ đây suy ra với mỗi giá trị x
thì ta có hoặc là f ( x) = 0 hoặc là ( ) 2002 f x = −x
. Ta sẽ chỉ ra rằng để
thỏa mãn yêu cầu bài toán thì bắt buộc phải có đồng nhất
CHUYÊN ĐÊ 1: ĐẠI SỐ VÀ GIẢI TÍCH 11 |
Tài liệu chuyên đề bồi dưỡng học sinh giỏi
f ( x)  0, x
  hoặc f (x) 2002  −x , x   .
Thật vậy, vì f (0) = 0 trong cả hai hàm số trên, nên không mất tính tổng quát ta có thể giả sử tồn tại a  0 sao
cho f (a) = 0 và tồn tại b  0 sao cho ( ) 2002 f b = b
(vì chỉ cần thay x = 0 vào quan hệ (1) ta nhận được hàm
f là hàm chẵn). Khi đó thế x = a y = −b vào (1) ta được (− ) = ( 2002 f b f a + b)
Vậy ta nhận được dãy quan hệ sau 2002 0  b − = f (b) = f ( b − ) = f ( 2002 a + b) 0 (0  0)  = −(  a + b) 002  (−(a +b)2002 2002 2002 2002  −b )
Bằng cách thử lại quan hệ hàm ban đầu ta kết luận chỉ có hàm số f ( x)  0, x
  thỏa mãn yêu cầu bài toán.
Bài 9: (Iran 1999, Trường hè toán học Mỹ 2002) Xác định các hàm số f : → thỏa mãn
f ( f ( x) + y) = f ( 2
x y ) + 4yf ( x), x  , y Lời giải: Thế 2
y = x ta được
f ( f ( x) 2 + x ) = f ( ) 2
0 + 4x f ( x), x  
Thế y = − f ( x) ta được
f ( ) = f ( f ( x) + x ) − ( f ( x))2 2 0 4 , x  
Cộng hai phương trình trên ta được
f ( x)( f ( x) 2 4 − x ) = 0, x   .
Từ đây ta thấy vỡi mỗi x
thì hoặc là f ( x)  0 hoặc là ( ) 2
f x = x . Ta chứng minh nếu f thỏa mãn yêu
cầu bài toán thì f phải đồng nhất với hai hàm số trên. Nhận thấy f (0) = 0 , từ đó thay x = 0 ta được
f ( y) = f (− y), y
  , hay f là hàm chẵn. Giả sử tồn tại a  0,b  0 sao cho f (a) = f (b) 2 0, = b − , khi đó
thay x = a, y = −b ta được f ( b − ) = f ( 2
a + b) → f (b) = f ( 2 a + b).
Từ đó ta có quan hệ sau
12 | C CHUYÊN ĐÊ: PHƯƠNG TRÌNH HÀM
Tài liệu chuyên đề bồi dưỡng học sinh giỏi 2 0  b − = f (b) = f ( b − ) = f ( 2 a + b) 0 (0  0)
= −(a +b)2 
(−a(a +b)2 2 2 2  b − )
Do đó xảy ra điều mâu thuẫn. Thử lại thấy hàm số f ( x)  0 , ( ) 2
f x = x thỏa mãn yêu cầu. Nhận xét:
1. Rõ ràng bài toán VMO 2002 có ý tưởng giống bài toán này.
2. Ngoài phép thế như trên thì bài toán này ta cũng có thể thực hiện những phép thế khác nhau như: 1 a. Thế y =
( 2x f (x)). 2 b. Thế y = 0 để ( ( )) = ( 2 f f x f x ) , sau đó thế 2
y = x f ( x) .
c. Thế y = x f ( x) và sau đó là 2
y = x x .
Bài 10: Tìm tất cả các hàm số f : \   0 →
sao cho với mọi x, y khác 0 và x y ta có  x
f ( y) − f (x) = f ( y). f   . x y Lời giải:     Đặ 1 x
t g(x) = f   ta được: g(y) − g(x) = g(y)g 1−   (1)  x   y   x   x
+ Cho y = 1: g(1) − g(x) = g(1)g (1− x) . Suy ra g(1) − g = g(1)g 1−     (2)  y   y   x
g(1) − g    y   x
+ Từ (1) và (2) suy ra g( y) − g(x) = g( y).  g(y)g
= g(x).g(1)   (3), với mọi g(1)  y
x, y  0; x y .  x
+ Trong (3) thay x bởi y x , ta được: g( y)g 1−
= g(y x).g(1)   (4).  y
+ Từ (1), (4) suy ra g( y) − g(x) = g( y x).g(1) . Từ đây suy ra g(u + v) = g(u) + g(v).g(1) (5), với mọi
u, v  0;u + v  0 .
+ Từ (3) suy ra g(xy)g(1) = g(x).g( y) với mọi x, y  0 (6).
+ Hoán đổi vai trò của u,v trong (5) suy ra nếu g(1)  1 thì g(x)  0 (mâu thuẫn). Do đó g(1) = 1 và ta
được: g(u + v) = g(u) + g(v); g(uv) = g(u).g(v) với mọi u, v  0 .
CHUYÊN ĐÊ 1: ĐẠI SỐ VÀ GIẢI TÍCH 13 |
Tài liệu chuyên đề bồi dưỡng học sinh giỏi 1
Theo kết quả cơ bản ta được g(x) = x . Vậy f (x) =
là hàm duy nhất cần tìm. x
4. Bài tập củng cố
Bài tập 1: Tìm tất cả các hàm số f : →
thoả mãn: f ( f ( x + y)) = x + f ( y), x  , y  (1) Lời giải:
Giả sử hàm số f thoả mãn đề bài. Trong (1) thay x bởi y và thay y bởi x ta được:
f ( f ( x + y)) = y + f ( x), x  , y  (2)
Từ (1) và (2) suy ra: x + f ( y) = y + f ( x), x  , y  (3)
Từ (3) cho y = 0 ta được f (x) = x + c, x  (c = f (0))
Thay (1) vào ta được c = 0 . Vậy hàm số cần tìm là: f ( x) = x, x
Bài tập 2: Tìm tất cả các hàm số f : (0, +) → (0, +) thoả mãn: ( ) ( )  y   x
f x f y = xf   + yf   , x  , y (0, +) (1)  2   2  Lời giải:
Giả sử f là hàm số thoả mãn yêu cầu của đề bài.  x
Trong (1) cho x = y , ta được: 2
f ( x) = 2xf   , x  (0,+)  2  2  x f ( x) Suy ra: f   = , x   (0, +).  2  2x 2 2 f y f x
Thay vào (1) ta có: f ( x) f ( y) ( ) ( ) = x + y , x  , y (0, +) 2 y 2xx f ( y) y f ( x) 2  x f ( y) y f ( x)   −  = 0 , x
 , y (0, +)  = , x  , y (0, +)  2 y 2x   2 y 2x f ( x) f ( y) → = f x ; xy ( +) ( ) , 0, → = a x
 (0,+) ( a là hằng số) thay f (x) = ax, x y x x  (0,+).
Thay vào (1), đồng nhất ta được: a = 0 hoặc a = 1.
Vậy có 2 hàm số thoả mãn đề bài: f ( x) = 0, x
 (0,+) và f ( x) = x , x  (0,+).
Bài tập 3: Tìm tất cả các hàm số f :
→ thoả mãn điều kiện:
f ( x + y) − f ( x y) 2 2
= 2y(x + y ), x  , y  . Hướng dẫn:
Đặt u = x + y,v = x − . y Đáp số: ( ) 3
f x = x + c , x  .
Bài tập 4: Tìm hàm f :
→ thỏa mãn: f (x) + xf (−x) = x +1, x   ( ) 1 .
14 | C CHUYÊN ĐÊ: PHƯƠNG TRÌNH HÀM
Tài liệu chuyên đề bồi dưỡng học sinh giỏi Lời giải:
Đặt t = −x , ta được: f ( t − ) −tf ( t − ) = t − +1, t   ( ) 1 . Ta có hệ:  f
 ( x) + xf (−x) = x +1 
f (x) = . Thử lại hàm số cần tìm là: f ( x) =1. −xf
(x)+ f (−x) 1 = −x +1  x −1
Bài tập 5: Tìm hàm số f : \ 0,  1 →
Thỏa mãn: f ( x) * + f =1+ x, x     (2) .  x Lời giải: − Đặ x 1 t x =
, 2  f x + f x = 1+ . x 1 ( ) ( ) ( 1) x − Đặ x 1 1 t 1 x = =
, 2  f x + f x =1+ x . 2 ( ) ( 1) ( 2) 1 x x −1 1 − Đặ x 1 t 2 x =
= x, 2  f x + f x =1+ x . 3 ( ) ( 2) ( ) 2 x2
f (x + f x =1+ x 1 ) ( ) 
1+ x x + x 1  1 1 
Ta có hệ  f ( x ) + f ( x ) = 1+ x f ( x) 1 2 = = x + +
. Thử lại thấy đúng. Vậy 2 1 1     −  ( 2 2 x 1 x f x + f x =1+ x 3 ) ( 2) 2  
hàm số cần tìm có dạng: f ( x) 1 1 1 = x + +   . 2  x 1− x   x −1
Bài tập 6: Tìm hàm số f : \  1 − ;0;  1 →
thỏa mãn: xf ( x) + xf =1, x   1 −   (3).  x +1 Lời giải: − Đặ x 1 t x =
, 3 → xf x + 2 f x = 1. 1 ( ) ( ) ( 1) x +1 − Đặ x 1 1 t 1 x =
= − , 3 → x f x + 2 f x =1. 2 ( ) 1 ( 1) ( 2) x +1 x 1 − + Đặ x 1 x 1 t 2 x = = , 3 → x f x + 2 f x =1. 3 ( ) 2 ( 2) ( 3) x +1 x −1 2 − Đặ x 1 t 3 x =
= x, 3 → x f x + 2 f x =1. 4 ( ) 3 ( 3) ( ) x +1 3
CHUYÊN ĐÊ 1: ĐẠI SỐ VÀ GIẢI TÍCH 15 |
Tài liệu chuyên đề bồi dưỡng học sinh giỏi
xf (x) + 2 f (x =1 1 ) 
x f ( x ) + 2 f ( x ) 2 = 1 − + 1 1 2 4x x 1 Ta có hệ  → f (x) = . Thử lại thấy đúng. x f x + 2 f x = 1 5x x −1  2 ( 2 ) ( 3) ( )
x f x + 2 f x =1  3 ( 3 ) ( ) 2 4x x +1
Vậy hàm số cần tìm là: f ( x) = x ( x − ) . 5 1
Bài tập 7: Tìm tất các các hàm số f :
→ thỏa mãn điều kiện
f ( x + y) + f ( x y) = 2 f ( x) cos y, x  , y Hướng dẫn:  1. Thế y → 2 
2. Thế y y + 2 3. Thế x → 0
Đáp số: f ( x) = a cos x + bsin x(a,b  ), x   .
Bài tập 8: Cho hàm số f :
→ thỏa mãn điều kiện f (xy + x + y) = f (xy) + f (x) + f ( y), x, y  . Chứng minh rằng:
f ( x + y) = f ( x) + f ( y), x  , y Hướng dẫn: 1. Tính f (0)
2. Thế y = −1. Chứng minh f là hàm số
3. Thế y = 1 → f (2x + ) 1 = 2 f ( x) +1
4. Tính f (2(u + v + uv) + )
1 theo (3) và theo giả thiết để suy ra f (2uv + u) = 2 f (uv) + f (u ) 1 y 5. Cho v = − ,
x u y, 2uv x để suy ra điều phải chứng minh 2 2
Bài tập 9: Tìm tất cả các hàm số f :
→ đồng thời thỏa mãn các điều kiện sau:
f ( x) = xf ( ) 1 x, x   0
f ( x) + f ( y) = 1+ f ( x + y), x
 , y  ,(x, y)  (0,0); x + y  0 Hướng dẫn:
1. Tính f (0), f (− ) 1
16 | C CHUYÊN ĐÊ: PHƯƠNG TRÌNH HÀM
Tài liệu chuyên đề bồi dưỡng học sinh giỏi x +   
2. Tính a +1 với a = f ( ) 1 1 1 = f = f x +1   
 theo cả hai điều kiện.  x +1  x +1 
Đáp số: f ( x) = x +1, x   .
Bài tập 10: Tìm tất các các hàm số * f :
→ thỏa mãn điều kiện:   f ( z) 1 * + 2 f = 3x, x      y Hướng dẫn: 1 Thế x x Đáp số 2 : f ( x) * = − x, x   . x
Bài tập 11: Tìm tất cả các hàm số f : \ 0,  1 → thỏa mãn điều kiện:  −  f ( x) x 1 + f = 2x, x   \   0,  1  x Hướng dẫn: x −1 1 − Thế x → , x x x −1 − Đáp số 1 x 1
: f ( x) = x + − , x   \0;  1 1− x x
Bài tập 12: (Belarus 1995) Tìm tất cả các hàm số f : → thỏa mãn:
f ( f ( x + y)) = f ( x + y) + f ( x) f ( y) Lời giải:
Rõ ràng f khác hằng số.
y = 0 vào điều kiện bài toán ta được
f ( f ( x)) = (1+ f (0)) f ( x), x  
Trong đẳng thức trên thay x bởi x + y thì
(1+ f (0)) f (x+ y) = f ( f (x + y)) = f (x + y)+ f (x) f (y)− xy, Đơn giản ta được
f (0). f ( x + y) = f ( x) f ( y) − xy (7)
Thay y = 1 vào (7) thì
CHUYÊN ĐÊ 1: ĐẠI SỐ VÀ GIẢI TÍCH 17 |
Tài liệu chuyên đề bồi dưỡng học sinh giỏi f (0)( x + )
1 = f ( x) f ( ) 1 − x .
Lại thay y = −1vào x bởi x +1 vào (7) ta có
f (0). f ( x) = f ( x + ) 1 . f (− ) 1 + x +1.
Kết hợp hai đẳng thức trên ta được ( f ( ))2 0 − f ( ) 1 f (− )
1 f ( x) − = f ( f (0) − f (− ) 1 ) x + f (0) . 2
Nếu ( f (0)) − f ( ) 1 f (− )
1 = 0 , thì thay x = 0 vào phương trình cuối cùng ta được f (0) = 0 , nên theo (7) thì 2
f ( x) f ( y) = xy . Khi đó f ( x) f ( ) 1 = x, x
  , điều này dẫn đến ( f (0)) − f ( ) 1 f (− ) 1 = 1 − , mâu thuẫn. 2
Vậy ( f (0)) − f ( ) 1 f (− )
1  0 , suy ra f ( x) là một đa thức bậc nhất nên có dạng f ( x) = ax + b . Thay vào
quan hệ hàm ban đầu suy ra a = 1, b = 0 . Vậy hàm số thỏa mãn yêu cầu bài toán là f ( x) = x, x   .
Nhận xét: Nếu chịu khó tính ta sẽ tính được f (0) = 0 bằng cách thế các biến x, y bởi hai số 0 và 1.
Bài tập 13: Cho hàm số f :
→ thỏa mãn điều kiện: 2 2
(x y) f (x + y) − (x + y) f (x y) = xy(x y ) với x, y  . i)
Tính f (0) và chứng minh f là hàm số lẻ. ii)
Tìm tất cả các hàm số f . Lời giải: i)
Tính f (0) và chứng minh f là hàm số lẻ.
Với x = y = 1 thì 2
f (0) = 0 hay f (0) = 0 . (1) y    − y
Với y = 0 thì f (0) = f ( 0 − ) = 0 (do (1)) x = 0 Với 
thì − yf ( y) − yf (− y) = 0 hay f (− y) = − f ( y) y  0
Vậy f là hàm số lẻ. ii)
Tìm tất cả các hàm số f .  u + v x = u  = x + y  Đặ 2 t  ta suy ra 
v = x y u vy =  2 2 2 − Khi đó u v
v f (u) − u f (v) = uv (2) 4
18 | C CHUYÊN ĐÊ: PHƯƠNG TRÌNH HÀM
Tài liệu chuyên đề bồi dưỡng học sinh giỏi u   0 Với  v  0 2 2 f (u) f (v) u v 2 2 f (u) u f (v) v Từ (2) ta được − = hay − = − . u v 4 u 4 v 4 2 f (u) u 1 Chọn v = 1 , ta có − = f (1) − . u 4 4 Đặ 1 t a = f (1) − 4 2 f (u) u Ta có − = a, u   0 . (3) u 4 3 x Suy ra f (x) = + ax, x   0 4 3 x
Từ (1), (3) ta được f (x) = + ax, x   . 4 Thử lại 3 x 1 1 Với f (x) =
+ ax , ta có: f (0) = 0; f (1) = + a a = f (1) − 4 4 4 và 2 2
(x y) f (x + y) − (x + y) f (x y) = xy(x y ) 3 x Vậy f (x) = + ax, x   . 4
Bài tập 14: Tìm tất cả các hàm số f : → thoả mãn: 1 f (xy) 1
+ f (xz) − f (x) f ( yz) 1  , x
 , y, z  . 3 3 9 Lời giải: 2 1 1 1  1  1
Cho x = y = z = 0 thì f (0) + f (0) 2
f (0)   f (0) −  0  f (0) =   3 3 9  3  3 2 1 1 1  1  1
Cho x = y = z = 1 thì f ( ) 1 + f ( ) 2 1 − f ( ) 1   f ( ) 1 −  0  f ( ) 1 =   3 3 9  3  3 2 1
Cho y = z = 0 thì
f (0) − f ( x) f (0)  . 3 9 Do f ( ) 1 0 = nên f ( x) 1  , x   . (1) 3 3
CHUYÊN ĐÊ 1: ĐẠI SỐ VÀ GIẢI TÍCH 19 |
Tài liệu chuyên đề bồi dưỡng học sinh giỏi 1 1 1
Cho y = z = 1, ta có f ( x) +
f ( x) − f ( x) f ( ) 1  . 3 3 9 Do f ( ) 1 1 = nên f ( x) 1  , x   . (2) 3 3
Từ (1) và (2) ta được f ( x) 1 = , x   . 3
Bài tập 15: Tìm tất cả các hàm số f : → thoả mãn f ( 2 2
x + y ) = xf ( x) + yf ( y), x  , y  (1). Lời giải: Cho x = 0 , từ ( ) 1 suy ra f ( 2
y ) = yf ( y), y   Cho y = 0 , từ ( ) 1 suy ra f ( 2
x ) = xf ( x), x   . Do đó (1) trở thành: f ( 2 2
x + y ) = f ( 2 x ) + f ( 2 y ), x
 , y  → f (x + y) = f (x) + f ( y), x  , y  0 (*)
thay y bởi − y từ ( ) 1 ta được: f ( 2 2
x + y ) = xf ( x) − yf (− y) → − yf (− y) = yf ( y), y
  → f (−x) = − f (x), x  
yf (−y) = yf ( y), y
  → f (−x) = − f (x), x
  , chứng tỏ f là hàm số lẻ.
Do đó với mọi x  0, y  0 ta có
f ( x y) = f ( x + (− y)) = f ( x) + f (− y) = f ( x) − f ( y)
f (x) = f (x y) + f ( y)
f ((x y) + y) = f (x y) + f ( y)
f (x + y) = f (x) + f ( y), x   0, y   0 (**)
Với mọi x  0, y  0 ta có
f ( x + y) = − f (−x y) = − ( f (−x) + f (− y)) = − (− f ( x) − f ( y)) = f ( x) + f ( y) (***)
Kết hợp (*),(**), (***) và ta được f ( x + y) = f ( x) + f ( y), x  , y  . 2 tính f ((x + ) 1 ) theo hai cách. Ta có
20 | C CHUYÊN ĐÊ: PHƯƠNG TRÌNH HÀM
Tài liệu chuyên đề bồi dưỡng học sinh giỏi f ((x + )2 1 ) = f ( 2 x + 2x + ) 1  (x + ) 1 f ( x + ) 1 = f ( 2
x ) + f (2x) + f ( ) 1  (x + )
1 ( f ( x) + f ( )
1 ) = xf ( x) + 2 f ( x) + f ( ) 1
f (x) = xf ( ) 1 , x  
f (x) = ax, x   ,a
Bài tập 16: (Đồng Nai 2015) Tìm tất cả các hàm g : → thỏa mãn:
g g ( x) 2
x + yz = g (x) g (x) 2 2 2
− 2x + 2yz + z y g ( y) 2 2
 + y z g (z)         2
− 2x yz + x + g ( y) g (z) 4 + x , x  , y, z  . Lời giải: Đặt ( ) = ( ) 2 f x
g x x ta được:
f ( f ( x) + yz) = x + f ( y) f ( z), x
 , y, z  . ( ) 1
Trong (1) cho x = y = 0 được: f ( f (0)) = f (0) f ( z), z
  . Từ đây ta có f (0) = 0 vì nếu ngược lại
f (0)  0 thì f là hàm hằng không thỏa mãn.
Trong (1) cho y = 0 ta được f ( f ( x)) = x, x
  .(2)Suy ra f là toàn ánh.
Trong (1) cho x = 0 ta được f ( yz) = f ( y) f ( z ), y  , z  .(3)
Từ (2) và (3) phương trình đã cho đươc viết lại f ( f ( x) + yz) = f ( f ( x)) + f ( yz), x
 , y, z  . (4)
Do f toàn ánh nên từ (4) suy ra f ( x + y) = f ( x) + f ( y), x  , y  .
Vì f vừa cộng tính vừa nhân tính nên f ( x) = ax, x   . Thử lại (1) có a = ⎯⎯
f (x) = x x   ⎯⎯ → g (x) 2 1 ,
= x + x, x   .
Bài tập 17: Tìm tất cả các hàm số f : → thỏa mãn:  1  1 
f (x + xy + f ( y)) = f (x) + f ( y) + , x  , y     .  2  2  Lời giải:
Dễ thấy hàm f hằng không thỏa mãn. Ta xét f không hằng.  1  1 
f (x + xy + f ( y)) = f (x) + f ( y) + , x  , y  (1)     2  2   1  1 
Trong (1) cho y = −1 ta được: f ( f ( 1 − )) = f (x) + f ( 1 − ) + , x   (2)     2  2 
CHUYÊN ĐÊ 1: ĐẠI SỐ VÀ GIẢI TÍCH 21 |
Tài liệu chuyên đề bồi dưỡng học sinh giỏi 1 Rõ ràng nếu f ( 1
− ) +  0 thì f là hàm hằng. Do đó: 1 1 f ( 1 − ) + = 0  f ( 1 − ) = − 2 2 2
Ta sẽ chứng minh: f ( x) 1 + = 0  x = 1 − . 2 1 −
Thật vậy, giả sử tồn tại a  −1 sao cho f (a) = . 2  1 
Trong (1) chọn y = a ta có: f ax + x − = 0, x     .  2 
Mâu thuẫn vì f không là hàm hằng. Do đó ta có: a = −1 .  1 −  Chú ý là f (− ) 1 1 = −
nên từ (2) ta có : f = 0   . 2  2  1 − − f (y) Trong (1) chọn 2 x = , ( y  1 − ) 1+ ta được: y  1      − − f  ( y) 1 − − f ( y) 1 − − f    ( y)    f  +
y + f ( y) 1  =  f   +  f  ( y) 1 2 2 2 . +  1+ y 1+ y 1+ y 2       2            1   − − f   ( y)  1       f   +  f  ( y) 1 1 2 + = f − = 0, y  −1    1+ y 2     2   2         1  − − f  ( y) 1 − − f  ( y) 1 2 2 → f   = − ,  y  1 −  = 1 − , y  1 − 1+ y 2 1+ y     Suy ra f ( y) 1
= y + , y  1 − 2 Do f (− ) 1 1 = − nên f ( x) 1 = x + , x   . 2 2
Thử lại ta có hàm số cần tìm là f ( x) 1 = x + , x   . 2
Bài tập 18: Tìm tất cả các hàm số f : → thoả mãn
f (xf ( y) + y) + f (xy + x) = f (x + y) + 2013xy (1). Lời giải:
22 | C CHUYÊN ĐÊ: PHƯƠNG TRÌNH HÀM
Tài liệu chuyên đề bồi dưỡng học sinh giỏi 1 Thay x = 1, y =
vào (1) ta được: f (a) = 1 2013   Trong đó 1 1 a = f +    2013  2013
Tiếp tục thay y = a vào (1), ta thu được:
f (x + a) + f (xa + x) = f (x + a) + 2013ax, x   .
hay f (ax + x) = 2013ax, x   . (2) t
Từ (2) suy ra a  −1. Thay x = vào (2), ta được a +1 2013at f (t) = , t
  . hay f (t) = ct,(c R) a +1
Tiếp theo, thay biểu thức của f (t ) vào (1), ta thu được đẳng thức 2
c xy + cy + cxy + cx = cx + cy + 2013xy; x  , y  . 2
 (c + c − 2013)xy = 0, x  , y R  1 − − 8053 c = 2 2
c + c − 2013 = 0   1 − + 8053 c =  2 1 − + 8053 1 − − 8053
Vậy ta nhận được hai hàm số thoả mãn đề bài là f (x) =
x f (x) = x 2 2
Bài tập 19: Tìm tất cả các hàm f :
→ thỏa mãn: f xf (y) + 2015x = 2015xy + f (x) với mọi x, y. Lời giải:
Cho x = 1 thì f f ( y) + 201 
5 = 2015y + f (1)
Chọn y thỏa mãn 2015 y + f (1) = −2015 , và đặt t = f (y) + 2015 thì f (t) = −2015 .
Chọn y = t , và thay vào giả thiết thì: f xf (t) + 2015 x = 2015xt + f (x)
Hay: f (0) = 2015xt + f (x)  f (x) = 2
− 015tx + f (0), x   .
Vậy f (x) là hàm bậc nhất.
Giả sử f (x) = mx + n . Thay vào giả thiết ta có:
mxf ( y) + 2015x + n = 2015xy + mx + n mx(my + n) + 2015mx + n = 2015xy + mx + n 2
m xy + (mn + 2015m) x = 2015xy + mx
CHUYÊN ĐÊ 1: ĐẠI SỐ VÀ GIẢI TÍCH 23 |
Tài liệu chuyên đề bồi dưỡng học sinh giỏi
Đẳng thức trên đúng với mọi x, y nên: 2 m = 2015 m =  2015   
mn + 2015m = m n = 2 − 014
Vậy có 2 hàm thỏa mãn yêu cầu, là f (x) =  2015x − 2014 .
Bài tập 20: Tìm hàm số f : → thỏa mãn
f ((x y)2 ) 2
= x − 2yf (x) + ( f ( y))2 ; x  , y  . Lời giải: f 0 = 0 2
Cho x = y = 0 → f (0) = ( f (0)) ( )    f  (0) = 1
+) Nếu f (0) = 0. Cho y = 0, x  ta được: f ( 2 x ) 2
= x f (t) = t, t   0 2 2 Cho x = y  ta được f ( ) 2
0 = x − 2xf ( x) + ( f ( x)) ( f ( x) − x) = 0  f ( x) = x . Thử lại thấy đúng
+) Nếu f (0) = 1 cho y = 0, x  ta được f ( 2 x ) 2
= x +1→ f (t) = t +1, t   0 . Cho x = 0, y  ta được  = + f ( f y y 1 2 y ) = 2 − y + ( f ( 2 y ) → ( 2 2 ( )
f ( y)) = f ( 2 y ) 2
+ 2y = y + 2y +1 = ( y + ) 1    f
 ( y) = − y −1
Giả sử tồn tại y  sao cho f ( y = −y −1 0 ) 0 0
Chọn x = y = y ta được: 0  f y = y +
1 = y − 2 y f ( y ) + ( f ( y )) 1 2 ( 0 ) 0 2   0 0 0 0  f
 ( y = − y −1 0 ) 0 y = 0  Nếu f ( y ) 0
= −y −1→ −y −1 = y −1→  (loại) 0 0 0 0  f  (0) = 1 −
Nếu f ( y = y +1→ −y −1 = y +1→ y = 1 − → f −1 = 0 0 ) 0 0 0 0 ( )
Vậy f ( y) = y +1, y   . .
Bài tập 21: Tìm tất cả các hàm số f : → + + thỏa mãn:
f (x + f (x) + 2 y) = 2x + f (2 f ( y)) ,  ; x y  . + Lời giải:
Giả sử tồn tại hàm số f ( x) thỏa mãn yêu cầu bài ra.
Đặt f (0) = a với a  +
Chọn x = 0 ; y = x , thay vào (14) ta được
f ( f (0) + 2x) = f (2 f (x)) x  
f (2x + a) = f (2 f (x)) x   + + ,
Nên f (x + f (x) + 2 y) = 2x + f (2 y + a) ,  ; x y  + (i)
24 | C CHUYÊN ĐÊ: PHƯƠNG TRÌNH HÀM
Tài liệu chuyên đề bồi dưỡng học sinh giỏi
Thay 2 y bởi y ta được
f (x + f (x) + y) = 2x + f ( y + a) ,  ; x y  + (ii)
Với x; y a thỏa mãn f (x) = f ( y) = t
Thay y bởi y a vào (ii) ta được: f (x + t + y a) = 2x + t ,  ; x y  +
Thay x bởi y, y bởi x a vào (ii) ta được: f ( y + t + x a) = 2 y + t ,  ; x y  + Do đó x = y
Chọn x = 0; y = 0 thay vào (i) ta có: f (a) = f (2a)
Theo kết quả phần trên suy ra a = 2a Suy ra a = 0
Chọn x = 0; y = x , thay vào (i) ta được f (2x) = f (2 f (x)) , x   +
Suy ra: 2 f (x) = 2x , x   ⎯⎯ → =   + f (x) x , x +
Thử lại thấy hàm số vừa tìm thỏa mãn yêu cầu bài toán.
Vậy f (x) = x , x
  + là hàm số cần tìm.
Bài tập 22: Tìm tất cả các hàm số f :
→ thỏa mãn điều kiện 2 2
f (x ) = f (x + y). f (x y) + y , x  , y  . Lời giải: f =
Cho x = y = 0 ta được f =  f 2 (0) 0 (0) (0)    f (0) =1
Cho x = y = 2 ta được f (4) = f (4). f (0) + 4 → f (0)  1
Vậy f (0) = 0. Cho x = y ta được 2 2 2
f (x ) = f (2x). f (0) + x = x f (t) = t, t   0.
Cho x = 0, y = t  0 , ta được 2 2
f (0) = f (t) f ( t
− ) + t  0 = tf ( t
− ) + t f ( t − ) = t − , t   0
Vậy f ( x) = x, x   .
Thử lại ta thấy hàm số f ( x) = x, x
  . thỏa mãn bài toán.
Bài tập 23: Tìm f ( x) xác định x  thỏa mãn     f  (0) = 2013; f = 2014     2   f
 ( x + y) + f ( x y) = 2 f (x)cos y x  , y  . ( ) 1
CHUYÊN ĐÊ 1: ĐẠI SỐ VÀ GIẢI TÍCH 25 |
Tài liệu chuyên đề bồi dưỡng học sinh giỏi Lời giải: Trong (1) cho  −      −    −     x = 0; y = → f + f = 0  f = − f = 2 − 014         2  2   2   2   2  Trong (1) cho              y = → f x + + f x − = 0  f x − = − f x +         2  2   2   2   2  Trong (1) cho  −      −    −  x = → f y − + fy = 2 f cos y = 4 − 028cos y       2  2   2   2       
Trong (1) cho y = x f x
+ f − − x = 4 − 028cos . x     (2)  2   2 
Trong (1) cho x = 0 → f ( y) + f (− y) = 2 f (0) cos y = 4026 cos y Trong (1) cho           y = x + → f x + + f x − = 4026cos x + = 4026sin . x       (3) 2  2   2   2 
Trừ từng vế hai phương trình (2) và (3) được:       2 f x − = 4
− 028cos x − 4026sin x f x − = 2
− 014cos x − 2013sin x      2   2  
Trong (1) thay x bởi x + có: 2       f ( x) = 2 − 014cos x + − 2013sin x +
= 2014sin x − 2013cos x      2   2 
Bài tập 24: Tìm các hàm số f : (1; +) → thoả mãn điều kiện:
f ( x) − f ( y) = ( y x) f ( xy) với mọi x, y  1 Lời giải:
Với mọi t  1 , thay ( ;
x y ) = (t; 2),(t; 4) và (2t; 2) vào (1) ta được:
f (t ) − f (2) = (2 − t ) f (2t )
f (t ) − f (4) = (4 − t ) f (4t )
f (2t ) − f (2) = (2 − 2t ) f (4t )
26 | C CHUYÊN ĐÊ: PHƯƠNG TRÌNH HÀM
Tài liệu chuyên đề bồi dưỡng học sinh giỏi
f (4) + (t −3) f (2) = t (2t −5) f (4t) (2), với mọi t 1. Lấy 5 t = → f ( ) 1 4 = f (2) 2 2   Thay vào (2) ta đượ 5 c: t f
 (2) = t (2t − 5) f (4t)  2  5 f 2
Do đó với mọi t  1,t  → f (4t) ( ) = 2 2t 2 f 2
Từ (1) ta có: f (t ) = f (4) + (4 − t ) f (4t ) ( ) = vơi 5 t  1, t  . t 2 5 5 Với t = , từ (1) thay x = , y = 2 ta có: 2 2  5  =   ( ) 1 f f f f f 2 − f (5) 4 (2) 2 (2) = = ⎯⎯ → f (t) 2 (2) = , t   1.  2  2 5 5 t 2 Đặ c
t c = 2 f (2) → f ( x) = với x  1 . x
Thử lại thỏa mãn điều kiện (1).
Vậy hàm số cần tìm là: ( ) c f x = , x  1. . x
Bài tập 25: Tìm tất cả các hàm số f : → thoả mãn f ( 2 2
x + y ) = xf ( x) + yf ( y), x  , y  (1). Lời giải: Cho x = 0 , từ ( ) 1 suy ra f ( 2
y ) = yf ( y), y   Cho y = 0 , từ ( ) 1 suy ra f ( 2
x ) = xf ( x), x   . Do đó (1) trở thành: f ( 2 2
x + y ) = f ( 2 x ) + f ( 2 y ), x
 , y  → f (x + y) = f (x) + f ( y), x  , y  0 (*)
thay y bởi − y từ ( ) 1 ta được: f ( 2 2
x + y ) = xf ( x) − yf (− y) → − yf (− y) = yf ( y), y
  → f (−x) = − f (x), x  
yf (−y) = yf ( y), y
  → f (−x) = − f (x), x
  , chứng tỏ f là hàm số lẻ.
Do đó với mọi x  0, y  0 ta có
CHUYÊN ĐÊ 1: ĐẠI SỐ VÀ GIẢI TÍCH 27 |
Tài liệu chuyên đề bồi dưỡng học sinh giỏi
f ( x y) = f ( x + (− y)) = f ( x) + f (− y) = f ( x) − f ( y)
f (x) = f (x y) + f ( y)
f ((x y) + y) = f (x y) + f ( y)
f (x + y) = f (x) + f ( y), x   0, y   0 (**)
Với mọi x  0, y  0 ta có
f ( x + y) = − f (−x y) = − ( f (−x) + f (− y)) = − (− f ( x) − f ( y)) = f ( x) + f ( y) (***)
Kết hợp (*),(**), (***) và ta được f ( x + y) = f ( x) + f ( y), x  , y  . 2 tính f ((x + ) 1 ) theo hai cách. Ta có f ((x + )2 1 ) = f ( 2 x + 2x + ) 1  (x + ) 1 f ( x + ) 1 = f ( 2
x ) + f (2x) + f ( ) 1  (x + )
1 ( f ( x) + f ( )
1 ) = xf ( x) + 2 f ( x) + f ( ) 1
f (x) = xf ( ) 1 , x  
f (x) = ax, x   ,a
Bài tập 26: Tìm tất cả các hàm số f : → sao cho: x  , y  , 2015 2015 f (x
+ 2014y) = f (2x + y) + f (3x + 2013y) + x − 5x − 2015. Lời giải:
Đặt f (x) − x = g(x), x
  .Thay vào giả thiết ta có 2015 g(x
+ 2014y) = g(2x + y) + g(3x + 2013y) + 2015  , x y  (1). Thay 2015
y = 3x x vào (1) ta có 2015
g(4x x ), x   (2). Xét hàm số 2015
h(x) = 3x x x
  ta có h(x) liên tục trên và lim h(x) = − ;
 lim h(x) = + , suy ra x→+ x→−
tập giá trị của h ( x) là
. Từ (2) suy ra g(x) = −2015 x   .
Do đó f (x) = x − 2015, x   . .
Thay vào thử lại ta thấy f (x) = x − 2015 x   . thỏa mãn.
Vậy f (x) = x − 2015 x   . .
Bài tập 27: Xác định hàm số f :
→ thỏa mãn đồng thời 3 điều kiện:
1. f (−x) = − f ( x) ; 2. f ( x + )
1 = 1+ f ( x), x    1  f ( x) 3. f = , x   0   . 2  x x
28 | C CHUYÊN ĐÊ: PHƯƠNG TRÌNH HÀM
Tài liệu chuyên đề bồi dưỡng học sinh giỏi Lời giải: x +1  1   1  f ( x) f = f 1+ =1+ f =1+ 1       2 ( ) Với mọi x ta có:  x   x   x x
Mặt khác, với mọi x khác 0; −1 ta có:    x f 2 2    x +1  1   x +1
x   x +1  x +1  1  f = f     = = f . = f 1−         2  x x     x
x +1  x   x   x +1   x +1     x +1   +  +  x 1 f ( x ) 1
x +1 ( x + )2 2 2 1 − f ( x + ) 1 = . 1  −  =    
x   (x + ) . 2 1   x   (x + )2 1 1 = (x + )2 1 −1− f ( x) 1 2  =
x + 2x f x  2 2 2 ( ) ( )     x x f ( x) 1 Từ ( ) 1 và (2) ta có: 2 1+ =
x + 2x f x  ⎯⎯
f x = x, x   \ 0; 1 − . 2 2 ( ) ( )     x x
Từ (1) có f (0) = − f (0) suy ra f (0) = 0 Ta có f (− ) 1 = − f ( ) 1 = − 1  + f (0) = 1 −  
. Vậy f (x) = x, x   ..
Bài tập 28: Tìm tất cả các hàm số f : → thỏa mãn:
xf ( x + xy) = xf ( x) + f ( 2
x ) f ( y), x  , y  . ( ) 1 . Lời giải:
Trong (1) lấy x = y = 0 được f (0) = 0.
Trong (1) lấy y = −1 ta có
xf ( x) + f ( 2 x ) f (− ) 1 = xf (0) = 0, x   (2)
Trong (2) lấy x = −1 ta được:  f (− ) =
f ( ) f (− ) − f (− ) 1 0 1 1 1 = 0    f  ( ) 1 = 1 + Nếu f (− )
1 = 0 thì từ (2) suy ra f đồng nhất 0 và hàm này thỏa mãn bài toán. + Nếu f ( )
1 = 1 thì trong (2) lại lấy x = 1 ta thu được f (− ) 1 = 1 − .
Từ đó (2) trở thành: f ( 2
x ) = xf ( x), x   (3)
Trong (1) ta cho y = 1:
xf ( x) = xf ( x) + f ( 2 2 x ) f ( ) 1 , x
   f (2x) = 2 f (x), x   0
Kết hợp (1) và (3) ta được:
f ( x + xy) = f ( x) + f ( x) f ( y), y   , x  0 (4)
Từ (4) lần lượt lấy x = 1, x = −1 ta có:
f (1+ y) = 1+ f ( y), y   f ( 1 − − y) = 1
− − f ( y), y  
CHUYÊN ĐÊ 1: ĐẠI SỐ VÀ GIẢI TÍCH 29 |
Tài liệu chuyên đề bồi dưỡng học sinh giỏi
Như vậy hàm f là một hàm số lẻ.
Trong (4) thay y bởi − y và sử dụng tính lẻ của hàm f :
f ( x xy) = f ( x) + f ( x) f (− y) = f ( x) − f ( x) f ( y), y   , x  0 (5)
Cộng vế theo vế (4) và (5) :
f ( x + xy) + f ( x xy) = 2 f ( x) = f (2x), y   , x  0
f (0) = 0 nên ta có f ( x + xy) + f ( x xy) = 2 f ( x) = f (2x), x  , y
f ( x + y) = f ( x) + f ( y), x  , y
Và bây giờ ta sẽ tính biểu thức f ((x + )2 1 ) theo hai cách:
f ((x + )2 ) = f ( 2
x + x + ) = f ( 2 1 2 1
x ) + f (2x) + f ( )
1 = xf ( x) + 2 f ( x) +1, x   f ((x + )2 1 ) = (x + ) 1 f ( x + ) 1 = ( x + ) 1 ( f ( x) + ) 1 , x  
Từ hai điều trên thu được:
xf ( x) + 2 f ( x) +1 = ( x + ) 1 ( f ( x) + ) 1 , x
   f (x) = x, x  
Thử lại thỏa. Kết luận của bài toán là: f ( x) = 0, x
  ; f (x) = x, x   .
II. Sử dụng tính chất ánh xạ để giải phương trình hàm
1. Nhắc lại một số khái niệm và tính chất của ánh xạ 1.1. Ánh xạ
Định nghĩa 1. Một ánh xạ f từ tập X đến tập Y là một quy tắc đặt tương ứng mỗi phần tử x của X
với một và chỉ một phần tử của Y . Phần tử này được gọi là ảnh của x qua ánh xạ f và được ký hiệu là f ( x) .
• Tập X gọi là tập xác định của f . Tập Y gọi là tập giá trị của f
• Ánh xạ từ X đến Y được ký hiệu
f : X Y x y = f ( x)
• Khi X Y là các tập số thực, ánh xạ f được gọi là một hàm số xác định trên X
• Cho a X , y Y . Nếu f (a) = y thì ta nói y là ảnh của a a là nghịch ảnh của y qua ánh xạ f .
• Tập hợp Y = yY x
  X , y = f (x) gọi là tập ảnh của f . Nói cách khác, tập ảnh f ( X ) là tập
hợp tất cả các phẩn tử của Y mà có nghịch ảnh.
1.2. Đơn ánh, toàn ánh, song ánh
Định nghĩa 2. Ánh xạ : f : X Y được gọi là đơn ánh nếu với, a X ,b Y a b thì
f (a)  f (b) , tức là hai phần tử phân biệt sẽ có hai ảnh phân biệt.
30 | C CHUYÊN ĐÊ: PHƯƠNG TRÌNH HÀM
Tài liệu chuyên đề bồi dưỡng học sinh giỏi
Từ định nghĩa ta suy ra ánh xạ f là đơn ánh khi và chỉ khi với , a X ,b Y f (a) = f (b) , ta phải có a = b .
Định nghĩa 3. Ánh xạ f : X Y được gọi là toàn ánh nếu với mỗi phần tử y Y đều tồn tại một phần
tử x X sao cho y = f ( x) . Như vậy f là toàn ánh nếu và chỉ nếu Y = f ( X ) .
Định nghĩa 4. Ánh xạ f : X Y được gọi là song ánh nếu nó vừa là đơn ánh vừa là toàn ánh. Như vậy
ánh xạ f : X Y là song ánh nếu và chỉ nếu với mỗi y Y , tồn tại và duy nhất một phần tử x X để
y = f ( x) .
1.3. Ánh xạ ngược của một song ánh
Định nghĩa 4. Ánh xạ ngược của f , được kí hiệu bởi 1
f − , là ánh xạ từ Y đến X gán cho mỗi phần tử −
y Y phần tử duy nhất x X sao cho y = f ( x) . Như vậy 1 f
(x) = y f (x) = y
Chú ý. Nếu f không phải là song ánh thì ta không thể định nghĩa được ánh xạ ngược của f . Do đó chỉ
nói đến ánh xạ ngược khi f là song ánh. 1.4. Ánh xạ hợp
Định nghĩa 5. Nếu g : A B f : B C g ( A)  B thì ánh xạ hợp f g : A C được xác định
bởi ( f g )(a) = f ( g (a)) Kí hiệu n
p = p p ... p n
Để có thể khai thác tính chất của ánh xạ cần nắm vững các khái niệm về đơn ánh, toàn ánh, song ánh. 2. Các ví dụ:
2.1. Sử dụng tính đơn ánh giải phương trình hàm

➢ Trong phương trình một vế có chứa f ( x) , vế còn lại chứa biến x bên ngoài thông thường f là đơn ánh.
➢ Nếu trong phương trình chứa f ( f ( x) + ( ;
x y )) hoặc f ( f ( y) + ( ;
x y )) , trong đó  ( ; x y ) là
một biểu thức đối xứng của x y thì ta thường chứng minh f là đơn ánh.
f đơn ánh trên D a,b D sao cho f (a) = f (b) thì a = . b
➢ Nếu hàm f đơn điệu thực sự trên D thì f đơn ánh trên . D
Ví dụ 1: Tìm hàm số f ( x) xác định trên thoả mãn: f ( f (x) + 2y) = 4x + 4y + 3, x  , y  . Lời giải:
Nhận xét f ( x) là đơn ánh.
Thật vậy, giả sử f (x ) = f (x ) thì: f ( f (x ) + 2y) = f ( f (x ) + 2 y) 1 2 1 2
→ 4x + 4y + 3 = 4x + 4y + 3  x = x . Vậy f là đơn ánh. 1 2 1 2
Ta có: f ( f (x) + 2 y) = 4x + 4 y + 3 = f ( f ( y) + 2x).
CHUYÊN ĐÊ 1: ĐẠI SỐ VÀ GIẢI TÍCH 31 |
Tài liệu chuyên đề bồi dưỡng học sinh giỏi
Vì f là đơn ánh nên: f (x) + 2x = f ( y) + 2x hay f (x) − 2x = f ( y) − 2 y , x  , y  .
Do đó: f ( x) − 2x = c , c  . Thay f ( x) = 2x + c vào điều kiện ta có c = 1.
Vậy hàm số cần tìm là: f ( x) = 2x +1, x  .
Ví dụ 2: Tìm tất cả các hàm số f : + + → thoả mãn điều kiện: y
f ( x + f ( y)) 2 3 4 = , x, y +   (1) 9xy + 8 Lời giải:
Ta sẽ chứng minh f là đơn ánh. Giả sử: f ( x) = f ( y), x, y +   , khi đó: 2 y
= ( x + f ( y)) = f ( x + f (x)) 2x f 3 4 3 4 = 2 9xy + 8 9x + 8
Vậy f là đơn ánh. x
 , y (5a,+) − Với 2 y 2 y 8a a  0 xét = a x = . Do đó, từ (1) suy ra: 9xy + 8 9ay  2y −8a   2x −8af
+ 4 f ( y) = a = f   
+ 4 f (x) , x
 , y (5a,+) (2)  3ay   3ax  2 y − 8a 2x − 8a
Do f đơn ánh nên từ (2) ta có: + 4 f ( y) = + 4 f ( x) , 3ay 3axf ( x) 2 − = f ( y) 2 − , x
 , y (5a,+) 3x 3yf ( x) 2 = + c , x
 , y (5a,+) ( c là hằng số) (3) 3x
Với x  0 , luôn tồn tại a  0 sao cho x  5a theo (3) ta có: f ( x) 2 = + . c 3x Vậy f ( x) 2 = + ; c x
  (0, +). Thử lại vào (1) dễ dàng suy ra: c = 0 hoặc 265 c = − . 3x 204
Dễ thấy: f ( x) 2 =
thoả mãn yêu cầu đề bài. Còn f ( x) 2 265 = −
không thế lớn hơn 0 với mọi x  0 . 3x 3x 204
Vậy có duy nhất 1 hàm số cần tìm f ( x) 2 = , x   0. 3x
Ví dụ 3: (Sóc Trăng 2018) Tìm tất cả các hàm f : → thỏa mãn:
f (2x + 2y + f ( x)) = f ( f ( y)) + 8x x  , y  ( ) 1 Lời giải:
Ta chứng minh f là đơn ánh. Từ (1) hoán vị x, y được
f (2x + 2y + f ( y)) = f ( f ( x)) + 8y x  , y  . (2)
Gỉa sử tồn tại x , x
thỏa mãn f ( x = f x . Khi đó từ (1) và (2) được 8x = 8x x = x nên f 1 ) ( 2) 1 2 1 2 1 2
đơn ánh. Trong (2) cho y = 0 được f (2x + f (0)) = f ( f (x)) x
  . Vì f đơn ánh nên
f ( x) = 2x + f (0) x
  hay f (x) = 2x + a x
  . Thử lại thấy thỏa mãn.
32 | C CHUYÊN ĐÊ: PHƯƠNG TRÌNH HÀM
Tài liệu chuyên đề bồi dưỡng học sinh giỏi
Ví dụ 4: (Italy TST 2007) Tìm tất cả các hàm f : → thỏa mãn:
f ( xy + f ( x)) = xf ( y) + f ( x) x  , y  ( ) 1 Lời giải:
Nhận thấy f  0 là một nghiệm hàm. Xét f  0 . Ta chứng minh f là “tựa đơn ánh”, tức là nếu tồn tại
a, b thỏa f (a) = f (b)  0 thì a = .
b Trong (1) lần lượt cho x = a, y = ; b x = ,
b y = a được:
f (ab + f (a)) = af (b) + f (a)
f (ab + f (b)) = bf (a) + f (b)
Từ 2 điều trên, do f (a) = f (b)  0 suy ra a = b ⎯⎯
f “tựa đơn ánh”.
Trong (1) cho x = y = 0 được: f ( f (0)) = f (0) ⎯⎯
f (0) = 0 (vì nếu ngược lại, f (0)  0 do f “tựa
đơn ánh” nên f (0)  0 , mâu thuẫn).
Trong (1) cho y = 0 có: f ( f ( x)) = f ( x), x
  . (2) . Do f “tựa đơn ánh” nên từ (2) suy ra
f ( x) 0;  x , x   . (3)
Dễ thấy các hàm: f ( x) = 0, x
  hoặc f (x) = x, x
  thỏa mãn (1). Ta sẽ chứng minh ngoài 2 hàm
này không còn hàm nào khác. Gỉa sử tồn tại hàm f thỏa (1) và  f  (a) = a
a : f (a)  0, b  : f (b) (3)  b ⎯⎯→   f  (b) = 0
Trong (1) cho x = a, y = b được: f (ab + a) = f (a) . Vì f (a)  0 và f “tựa đơn ánh” nên
ab + a = a ab = 0 , mâu thuẫn. Vậy có 2 hàm thỏa mãn bài toán là f ( x) = 0, x   hoặc
f ( x) = x, x   .
2.2. Sử dụng tính toàn ánh giải phương trình hàm
➢ Nếu f là toàn ánh thì ta hay dùng tồn tại một số b sao cho f (b) = 0 ( f (b) = 1
− , f (b) =1,...) sau đó tìm b. ➢ Nếu f : →
là toàn ánh thì với mọi y
luôn tồn tại x
sao cho f ( x) = . y
➢ Khi giải phương trình hàm dựa vào giá trị của đối số và giá trị của hàm số ta cũng có thể vận dụng
tính toàn ánh của f .
➢ Mọi đa thức bậc lẻ có tập giá trị là . Tức là các hàm đa thức bậc lẻ đều là toàn ánh trên . Do đó
ta thường biến đổi để xuất hiện một đa thức bậc lẻ để có thể sử dụng tính toàn ánh.
➢ Đối với phương trình hàm có chứa f (x f ( y)), f (x), f ( y),... thì thường lấy x = 0 và thường
thay x bởi  f ( x),  f ( y),...
Ví dụ 1: Tìm tất cả các hàm số f : → thỏa mãn
CHUYÊN ĐÊ 1: ĐẠI SỐ VÀ GIẢI TÍCH 33 |
Tài liệu chuyên đề bồi dưỡng học sinh giỏi
f (x + f ( y)) = x + f ( y) + xf ( y), x  , y  . Lời giải:
Giả sử tồn tại hàm số f ( x) thỏa yêu cầu bài ra.
Ta có thể viết lại quan hệ hàm dưới dạng :
f ( x + f ( y)) = x( f ( y) + ) 1 + f ( y) (2)
Trường hợp 1: f ( y) = 1 − , y
  , thử lại thỏa yêu cầu bài toán.
Trường hợp 2: f ( y) không đồng nhất với 1
− , khi đó vế phải của (2) là hàm số bậc nhất theo x nên tập
giái trị của f (x + f ( y)) là . Suy ra f là toàn ánh.
Thay x bởi 0 vào (1) ta được: f ( f ( y)) = f ( y) (3)
f là toán ánh nên x
  R,y  thỏa f ( y) = x nên từ (3) ta được, thử lại ta thấy f (x) = x không
thỏa mãn. Vậy f (x) = 1
− là hàm số thỏa mãn yêu cầu bài toán. Nhận xét:
Trong các bài toán này từ điều kiện
f ( f ( y)) = f ( y) giúp ta tìm ra nghiệm của bài toán nhưng với điều kiện f phải là toán ánh.
Như vậy, tính toán ánh của f trong bài toán này tỏ ra khá hiệu quả giúp bài toán trở nên dễ dàng hơn
trong quá trình tìm bời giải.
Ví dụ 2: Tìm tất cả các hàm f : → thỏa mãn: f ( 3
x + y + f ( y)) 2
= 2y + x f (x), x  , y  . ( ) 1 Lời giải:
Cho x = 1 ta có f (1+ y + f ( y)) = 2y + f ( ) 1 , y   ⎯⎯
f là toàn ánh. Đặt f (0) = a . Trong (1) cho
x = y = 0 được f (a) = 0 . Trong (1) cho x = 0, y = a được f (a) = 2a . Do đó a = 0 ⎯⎯ → f (0) = 0.
Trong (1) cho x = 0 được f ( y + f ( y)) = 2y, y   .
Trong (1) cho y = 0 được f ( 3 x ) 2
= x f (x), x   .
Từ đó ta viết (1) dưới dạng f ( 3
x + y + f ( y)) = f ( y + f ( y)) + f ( 3 x ), x  , y  .
f toàn ánh nên từ đây có f ( x + y) = f ( x) + f ( y), x  , y  ⎯⎯
f (nx) = nf (x), x   ,n  . Lại có:
f ((x + )3 + (x − )3 ) = f ( 2
x + x) = f ( 3
x ) + f ( x) 2 1 1 2 6 2 6
= 2x f (x) + 6 f (x), x   .(2)
34 | C CHUYÊN ĐÊ: PHƯƠNG TRÌNH HÀM
Tài liệu chuyên đề bồi dưỡng học sinh giỏi f ((x + )3 1 + ( x − )3 1 ) = f ((x + )3
1 ) + f ((x − )3 1 ) = (x + )2 1 f (( x + ) 1 ) + ( x − )2 1 f (( x − ) 1 ) =( 2 x + 2x + )
1 ( f ( x) + f ( ) 1 ) + ( 2 x − 2x + )
1 ( f ( x) − f ( ) 1 ) =( 2
2x + 2) f ( x) + 4xf ( ) 1 , x   .(3)
Từ (2) và (3) suy ra f ( x) = xf ( ) 1 , x
  . Thử lại thấy f ( ) 1 = 1 ⎯⎯
f (x) = x, x   .
Ví dụ 3: (Chuyên KHTN 2018) Tìm tất cả các hàm f : → thỏa mãn:
f (( x y) f ( x) − f ( y)) + ( x + )
1 f ( y x) + x = 0 x  , y  ( ) 1 Lời giải:
Trong (1) cho x = y ta có: f (− f ( x)) = −( f (0) + )
1 x f (0) x   . (2)
Trường hợp 1: f (0) = 1 −
Trong (2) cho x = 0 ta có: f (− f (0)) = − f (0) = 1 → f ( ) 1 = 1
Từ đó trong (1) cho x = y = 1 ta có: f (− )
1 = 1, cho x = −1, y = 0 được f (0) = 1, mâu thuẫn.
Trường hợp 2: f (0)  1
− . Từ (2) suy ra f là toàn ánh trên nên tồn tại c  sao cho f (c) = 0 .
Trong (1) cho x = 0, y = c ta có: f (cf (0)) = 0
Trong (1) cho x = y = c ta có: f (0) + (c + ) 1 f (0) + c = 0
Trong (1) cho x = y = −cf (0) ta có: f (0) + (−cf (0) + )
1 f (0) − cf (0) = 0 Từ đó ( f ( ) + )2 0
1 c = 0 → c = 0 hay f (0) = 0 . Do đó từ (2) suy ra f (− f ( x)) = −x x   . (3)
Trong (1) cho x = 0 ta có: f (− f ( y)) = − f ( y) y
  → f (− f (x)) = − f (x) x   . (4)
Từ (3) và (4) suy ra f ( x) = x x
  . Thử lại thấy thỏa mãn.
Ví dụ 4: (Shortlist IMO 2012) Tìm tất cả các hàm f : → thỏa mãn:
f ( f ( x) + y) = 2x + f ( f ( y) − x), x  , y  . ( ) 1 Lời giải:
Trong (1) thay y bởi − f ( x) ta có f (0) = 2x + f ( f (− f ( x)) − x), x   . ⎯⎯ → f là toàn ánh. Suy ra a
sao cho f (a) = 0 .
Trong (1) thay x = a được f ( y) = 2a + f ( f ( y) − a)  f ( f ( y) − a) = f ( y) − a − . a (2)
CHUYÊN ĐÊ 1: ĐẠI SỐ VÀ GIẢI TÍCH 35 |
Tài liệu chuyên đề bồi dưỡng học sinh giỏi
Do f toàn ánh nên với mỗi x
đều tồn tại y
sao cho x = f ( y) − a . Từ (2) suy ra
f ( x) = x a, x
  . Thử lại thấy thỏa mãn.
Ví dụ 5: Tìm tất cả f : → , thỏa mãn
f ( x f ( y)) = f ( x) − 2xf ( y) + f ( f ( y)) x  , y  (1) Lời giải:
Xét f (x) = 0 x  . thỏa mãn.
Giả sử tồn tại a sao cho f (a)  0
Thay y bới a vào (1) ta có 2 .
x f (a) − f ( f (a) = f (x) − f (x f (a))
Khi đó ta có x  thì tồn tại a,b sao cho x = f (a) − f (b)
Do đó f (x) − f ( y) toàn ánh trên
Thay x bới f ( y) vào (1) ta được 2
f (0) = 2 f ( f ( y)) − 2 f ( y)
Thay x bới f (x) vào (1) : f (0) 2 2
f ( f (x) − f ( y)) = f ( f (x)) − 2 f (x). f ( y) + f ( f ( y)) = f (x) − 2 f (x). f ( y) + f ( y) + 2 f (0) 2
= ( f (x) − f (y)) +  , x y  2 ⎯⎯
f (x) = x + b x   . 2
Thử lại thấy b = 0
Vậy có 2 hàm thỏa mãn là f (x) = 0, x   và 2
f (x) = x , x  
2.3. Sử dụng tính song ánh giải phương trình hàm
Khi f là một song ánh ta có thể chú ý đến tính chất đơn ánh và toán ánh mà ta đã vận dụng trong phần 1
và phần 2. Ngoài ra ta có thể chú ý thêm. Nếu f :
→ thoả mãn: f ( f (x)) = ax + , b x
  ,(a  0) thì f là song ánh.
f song ánh liên tục (đơn điệu) và cộng tính trên tập
khi đó f ( x) = ax
( f song ánh và cộng tính trên tập rời rạc khi đó f ( x) = ax )
Ví dụ 1: (Olympic 30.4.2011)
Tìm tất cả các hàm số f :
→ thoả mãn điều kiện:
f ((1+ f (x)) f ( y)) = y + xf ( y), x  , y  (1)
36 | C CHUYÊN ĐÊ: PHƯƠNG TRÌNH HÀM
Tài liệu chuyên đề bồi dưỡng học sinh giỏi Lời giải:
Từ (1) cho x = 0 ta được f ((1+ f (0)) f ( y)) = y , y   (1)
Giả sử f ( y = f y , từ (2) dễ dàng suy ra y = y . Do vậy f là đơn ánh. Với y   khi đó tồn tại 1 ) ( 2 ) 1 2
x = (1+ f (0)) f ( y) sao cho f ( x) = y, suy ra f là toàn ánh, dẫn tới f là song ánh.
Vì thế tồn tại c  sao cho f (c) = 0. Từ (1) cho y = c được f (c) = 0.
Từ (1) cho x = y = 0 ta được f ((1+ c)c) = 0.
Vậy f ((1+ c)c) = f (c) , mà f là đơn ánh nên: (1+ c) c = c c = 0  f (0) = 0.
Từ (1) cho x = 0 ta được f ( f ( y)) = y , y   (3)
Từ (1) thay x bởi f ( x) , thay y bởi f ( y) và sử dụng (3) ta có:
f ( y (1+ x)) = f ( y) + yf ( x) , x  , y  (4)
Từ (4), cho x = −1 , sử dụng f (0) = 0 và đặt a = − f (− )
1 ta được: f ( y) = ay , y  
Thay vào (3) đồng nhất được a  1 − :  1
Vậy: f ( x)  x, f ( x)  − .
x Thử lại ta thấy cả 2 hàm đều thoả mãn.
Ví dụ 2: Tìm tất cả các hàm số f : → thỏa mãn: 2 2
f (x + 2 f ( y)) = 2 y + f (x); x  , y Lời giải:
Giả sử tồn tại hàm số f (x) thỏa mãn 2 2
f (x + 2 f ( y)) = 2 y + f (x); x  , y  (1)
Trong (1), thay x = 0 suy ra 2
f (2 f ( y)) = 2 y + f (0) (2)
Suy ra f là song ánh
Cho y = 0 , thay vào (1) ta được 2 2
f (x + 2 f (0)) = f (x); x, y (3)
Thay x bởi –x vào (3) ta được 2 2
f (x + 2 f (0)) = f (−x) Suy ra 2 2
f (−x) = f (x); x
Nếu f (x) = f (−x)  x = −x x = 0
Do đó f (−x) = − f (x), x  0
f là song ánh nên tồn tại duy nhất b  : f (b) = 0.
Nếu b  0 thì f ( b
− ) = − f (b) = 0 , hay f ( b
− ) = f (b) hay –b = b ta được b = 0. Suy ra f (0) = 0 hay
f (−x) = − f ( x)  x  hay f là hàm lẻ trên
f (x) = 0  x = 0
CHUYÊN ĐÊ 1: ĐẠI SỐ VÀ GIẢI TÍCH 37 |
Tài liệu chuyên đề bồi dưỡng học sinh giỏi
Khi đó (2) thành f (2 f ( y)) = 2y, y  (3) thành 2 2
f (x ) = f (x)  0 ,  x
, hay f (x)  0 ,  x  0 Khi đó (1) thành 2 2
f (x + 2 f ( y)) = f (2 f ( y)) + f (x ) (4)
Hay f (x) + f ( y) = f (x + y)  x  0, y
Mặt khác nếu x  0 , y
: f (x) + f ( y) = −( f (−x) + f (− y)) = − f (−x y) = f (x + y)
Hay f (x) + f ( y) = f (x + y)  x, y
Với x y ta có: f ( x) = f ( x y + y) = f ( x y ) + f ( y )  f ( y )
hay f đồng biến trên
Ta đi chứng minh f (x) = x , x
Thật vậy nếu  x
: f (x )  x hay 2 f (x )  2x f ( f (x ))  f (2x ) o o o o o o o
hay 2x  2 f (x )  x f (x ) ( vô lý) o o o o
Tương tự nếu  x  : f (x )  x , ta cũng dẫn tới vô lý. o 0 o
hay f (x) = x x
Thử lại ta thấy f (x) = x x
thỏa mãn phương trình ( 1)
Vậy f (x) = x x
Ví dụ 3: (VMO 2017) Tìm tất cả các hàm f : → thỏa mãn:
f ( xf ( y) − f ( x)) = 2 f ( x) + xy x  , y  ( ) 1 Lời giải:
Trong (1) cho x = 1 đươc: f ( f ( y) − f ( ) 1 ) = 2 f ( ) 1 + y y   (2)
Từ đây dễ thấy f là song ánh. Do đó tồn tại duy nhất số thực a để f (a) = 0 . Trong (1) cho x = a được
f (af ( y)) = ay y   . (3)
Trong (3) cho y = 0 được f (af (0)) = 0 = f (a).Từ đó vì f đơn ánh nên a = 0 hoặc f (0) = 1.
Nếu a = 0 → f (0) = 0 . Thay y = 0 vào (1) được f (− f ( x)) = 2 f ( x) . Do f là toàn ánh nên f ( x) = 2 − x, x
  . Thử lại thấy không thỏa mãn. Vậy a  0 → f (0) =1.
Trong (1) thay x = 0 ta được f (− )
1 = 2 . Thay y = a vào (3) được 2
a = f (0) = 1 → a = 1 (do f (− ) 1 = 2) tức f ( ) 1 = 0.
38 | C CHUYÊN ĐÊ: PHƯƠNG TRÌNH HÀM
Tài liệu chuyên đề bồi dưỡng học sinh giỏi Do f ( ) (2)
1 = 1 ⎯⎯→ f ( f ( y)) = y y   . (2)
Thay y bởi f ( y) vào (1) và sử dụng (2’) được f ( xy f ( x)) = 2 f ( x) + xf ( y) x  , y  . (4) f ( x)
Trong (4), xét x  0 và thay y = được xf x   f x  − f x 1 = 2 f ( x) ( ) ( ) 1 2 ( ) + xf   ⎯⎯ → f   = , x   0. x x x     f ( x) Thay y =
vào (1) và sử dụng kết quả trên được f (1− 3 f ( x)) = 3 f ( x), x   0. x
Do f là song ánh và f (0) = 1 nên với x  0 thì 1− 3 f ( x) có thể nhận mọi giá trị thực khác −2 . Do đó
từ kết quả trên suy ra f ( x) = −x +1, x   2 − . Nói riêng f (3) = 2
− . Thay y = 3 vào (2’) ta được f ( 2
− ) = 3 . Vậy f (x) = −x +1, x   . Thử lại thỏa mãn.
3. Bài tập vận dụng
Bài 1: (Olympic 30.4.2006)
Tìm tất cả các hàm số f :
→ thoả mãn điều kiện:
f ( x + f ( y) + xf ( y)) = x + xy + y , x  , y  (1) Lời giải:
Ta sẽ chứng minh f là đơn ánh. Giả sử f ( y = f y 1 ) ( 2 )
f (x + f (y ) + xf (y )) = f x + f y + xf y 1 1 ( ( 2 ) ( 2 ))
x + xy + y = x + xy + y y = y ⎯⎯ → f là đơn ánh. 1 1 2 2 1 2
Cho x = 0 , thay vào (1) ta có: f ( f ( y)) = y, y   .
Cho y = 0 , thay vào (1) ta có: f ( x + f (0) + xf (0)) = x = f ( f ( x)) → x + f (0) + xf (0) = f (x).
Đặt f (0) = a, từ đó ta có: f (x) = (a +1)x + a, x  (2)
Thay (2) vào (1), dễ dàng suy ra a = 0 hoặc a = −2.
Khi a = 0, f ( x) = . x
Khi a = −2, f (x) = −x − 2. Thử lại ta thấy thoả mãn.
Vậy có 2 hàm số toả mãn đề bài: f (x) = x, x
f (x) = −x − 2, x  .
Bài 2: Tìm tất cả các hàm số f xác định và đồng biến trên thỏa mãn:  1  f f
( y)+ 2x = 4x + y +1, x  , y  .  ( ) 1  4 
CHUYÊN ĐÊ 1: ĐẠI SỐ VÀ GIẢI TÍCH 39 |
Tài liệu chuyên đề bồi dưỡng học sinh giỏi Lời giải: 1  1 1  Thay x = −
y vào (1) được: f f
( y)− y =1, y   .  (2) 4  4 2   1 
Trong (1) cho x = y = 0 được f f  (0) =1.  Kết hợp (2) suy ra  4   1    f f  ( y) 1 1 − y = f f   (0) y   .  (3)  4 2   4 
Do f đồng biến nên f là đơn ánh trên , từ (3) suy ra 1 f ( y) 1 1
y = f (0), y   ⎯⎯
f (x) = 2x + a, x   . 4 2 4 Thay vào (1) đượ 2 2 c a = ⎯⎯
f (x) = 2x + , x   . 3 3
Bài 3: (VMO 2016) Tìm tất cả các số thực a để tồn tại hàm f : → thỏa mãn: i) f ( ) 1 = 2016 ii)
f ( x + y + f ( y)) = f ( x) + ay, x  , y  . Lời giải:
Với a = 0 có thể tìm được 1 hàm thỏa mãn bài toán là f ( x) = 2016. Do đó chỉ xét trường hợp a  0 là
đủ. Thay x = − f ( y) vào ii) ta được: f ( y) = f (− f ( y)) + ay, y   .
Từ đó dễ thấy f là đơn ánh.
Thay y = 0 vào ii) được f ( x + f (0)) = f ( x), x   ⎯⎯ → f (0) = 0. f ( x) f ( x)  f (x)  Thay y = −
vào ii) và kết hợp tính đơn ánh của f được − + f −  = −x, x   . a a a   f ( y) Thay y bởi −
vào ii) và sử dụng kết quả trên được f ( x y) = f ( x) − f ( y), x
 , y  . Suy ra f a
cộng tính. Từ đó dễ tính được f ( ) = f ( ) 2 2016 2016 1 = 2016 .
Do f cộng tính nên từ ii) rút ra f ( y) + f ( f ( y)) = ay y
  . Cho y = 1⎯⎯ → a = 2016.2017.
Thử lại với giá trị trên của a ta tìm được f ( x) = 2016x thỏa mãn bài toán.
Vậy có 2 giá trị cần tìm của a a = 0 hoặc a = 2016.2017.
Bài 4: (Bà Rịa – Vũng Tàu 2014) Tìm tất cả các hàm f : → thỏa mãn:
f ( xy + f ( x)) + f ( x yf ( x)) = 2x x  , y  ( ) 1
40 | C CHUYÊN ĐÊ: PHƯƠNG TRÌNH HÀM
Tài liệu chuyên đề bồi dưỡng học sinh giỏi Lời giải:
Trong (1) cho y = 0 được f ( f ( x)) + f ( x) = 2x, x   ⎯⎯ → f là đơn ánh.
Trong (1) lần lượt cho y = −1, y = 1 được
f ( x + f ( x)) + f ( x f ( x)) = 2x, x   . (2)
f (−x + f ( x)) + f ( x + f ( x)) = 2x, x   . (3)
Từ (2) và (3) suy ra f ( x f ( x)) = f (−x + f ( x)), x
  . (4) . Vì f đơn ánh nên từ
(4) ⎯⎯→x f (x) = −x + f (x), x   ⎯⎯
f (x) = x, x
  . Thử lại thấy thỏa mãn.
Bài 5: (Quãng Ninh 2018) Tìm tất cả các hàm f : → thỏa mãn:
f (x −( f ( y))2 2 )= xf (x) 2 + y x  , y  ( ) 1 Lời giải: Đặt 2
a = − f (0) . Trong (1) cho:
x = y = 0 được f (a) = 0
x = 0, y = a được f ( ) 2 0 = −a a = 0 Do đó 4 a = −a   a = 1 − Nếu a = 1
− → f (0) = −1. Trong (1) cho x = 1 − , y = 0 → 1 − = − f (− ) 1 → 1 = f (− )
1 = f (a) = 0 , vô lý.
Suy ra a = 0 → f (0) = 0 . Trong (1) cho y = → f ( 2 0
x ) = xf ( x) x   . Nếu f ( y) (1) = ⎯⎯→ f ( 2
x ) = xf ( x) 2 0
y y = 0.
Trong (1) cho thay x = 0, y = x được f ( 2 − f (x)) 2 = −x x   . (2)
Trong (1) cho thay x bởi −x , y = 0 được ( 2
f x ) = −xf (−x) x  
Suy ra xf ( x) = −xf (−x) x
  → f (−x)= − f (x) x   0.Kết hợp
f (0) = 0 → f (−x) = − f ( x) x   . (3)
Ta sẽ chứng minh f là toàn ánh trên . Từ (2) ta có với mỗi y  0 luôn tồn tại x sao cho f ( x) = . y
Còn với y  0 theo (2), (3) thì tồn tại x sao cho 2 = = − ( 2 − ( )) = ( 2 y x f f x f f ( x)) .
CHUYÊN ĐÊ 1: ĐẠI SỐ VÀ GIẢI TÍCH 41 |
Tài liệu chuyên đề bồi dưỡng học sinh giỏi
Vậy f là toàn ánh. Tiếp theo chứng minh f ( x) = x, x
  . Từ (1) sử dụng các kết quả trên ta có f ( 2 2
x f ( y)) = f ( 2 x ) + f ( 2
f ( y)) x  , y  . (4)
Từ (4) do f toàn ánh nên f ( x + y) = f ( x) + f ( y), x   0, y   0.
Kết hợp (3) suy ra f ( x + y) = f ( x) + f ( y), x   0, y   0.
Mặt khác f ( x) = f ( x + y y) = f ( x + y) + f (− y) = f ( x + y ) − f ( y ), x   0, y   0.
Kết hợp (3) suy ra f ( x) = f ( x + y) − f ( y), x   0, y   0.
Vậy f ( x + y) = f ( x) + f ( y), x  , y  . (5) Sử dụng f ( 2
x ) = xf ( x) x
  . và (5) ta được: f ((x + )2 1 ) = (x + )
1 ( f ( x) + f ( ) 1 ) f ((x + )2 1 ) = f ( 2 x + 2x + )
1 = xf ( x) + 2 f ( x) + f ( ) 1
f (x) = xf ( ) 1 = cx x   .
Thay vào (1) được c = 1 → f ( x) = x, x
  . Thử lại thỏa mãn
Bài 6: (Hà Nam 2018) Cho số thực a khác 0 và khác 1
− . Tìm tất cả các hàm f : → thỏa mãn:
f ( f ( x) + ay) = ( 2
a + a) x + f ( f ( y) − x) x  , y  ( ) 1 Lời giải: f (x)
  − f (x)  
Trong (1) thay y bởi
ta có: f f   − x = −( 2
a + a) x + f (0) x   . aa     
Suy ra f là toàn ánh trên .
Bây giờ giả sử tồn tại x , x
sao cho f ( x = f x , do f là toàn ánh nên tồn tại t sao cho 1 ) ( 2) 1 2
f (t ) = x + x . Khi đó, trong (1) cho x = x , y = t có: f ( f ( x ) + at ) = ( 2
a + a x + f x 1 ) 1 ( 2) 1 2 1
Trong (1) cho x = x , y = t có: f ( f ( x ) + at ) = ( 2
a + a x + f x 2 ) 2 ( 1) 2
Do đó x = x , tức f là đơn ánh trên . Mặt khác trong (1) cho x = 0 có: 1 2
f (ay + f (0)) = f ( y) y
  → f ( y) = y + f (0) y
  . Thử lại thấy thỏa mãn.
Bài 7: (Bà Rịa – Vũng Tàu 2017) Tìm tất cả các hàm f : → thỏa mãn:
f ( x + yf ( x)) = xf ( y) + f ( x) x  , y  . ( ) 1
42 | C CHUYÊN ĐÊ: PHƯƠNG TRÌNH HÀM
Tài liệu chuyên đề bồi dưỡng học sinh giỏi Lời giải:
Trong (1) cho y = 0 có: f ( x) = xf (0) + f ( x) x   → f (0) = 0.
• Ta nhận thấy f (x)  0 là một nghiệm hàm.
• Xét f (x)  0 . Khi đó tồn tại a  0 sao cho f (a)  0
Trong (1) cho x = a ta có af ( y) = 0 → f ( y) = 0 y
  vô lý.Vậy f (x) = 0  x = 0. Trong (1) cho x = 1 − , y = 1 − → f (− ) 1 = 1 − .
Trong (1) cho x = 1, y = 1
− → f (1− f ( ) 1 ) = f ( ) 1 −1. 2
Trong (1) cho x = 1− f ( )
1 , y = 1 → ( f ( ) 1 − ) 1 = 0 → f ( ) 1 = 1.
Trong (1) cho x = 1 → f ( y + )
1 = f ( y) +1 y   .
Trong (1) thay y bởi y +1 → f ( x + yf ( x) + f ( x)) = xf ( y) + f ( x) + x = f (x + yf (x)) + x x  , y  .
Mặt khác do x + yf ( x) toàn ánh nên thay x + yf ( x) bởi y được f ( y + f ( x)) = f ( y) + x x  , y  . (2)
Trong (2) cho y = 0 được f ( f ( x)) = x x   .
Trong (2) thay x bởi f ( x) được f ( x + y) = f ( x) + f ( y) x  , y  (3)
Trong (1) thay x bởi f ( x) và kết hợp (3) được f ( xy) = f ( x) f ( y) x  , y  . (4)
Từ (3), (4) ta có nghiệm hàm là f ( x)  x hoặc f ( x)  0.
Bài 8: (Chọn đổi tuyển Indonesia 2010)
Xác định tất cả các số thực a sao cho có một hàm số thoả mãn:
x + f ( y) = af ( y + f (x)) , x  , y  (1) Lời giải:
Dễ thấy a = 0 không thoả mãn. Giả sử: a  0. x + f
Thay y = 0 vào (1) ta được: (0)
f ( f (x)) = ; x  (2) a
Từ (2) suy ra f là toàn ánh nên tồn tại   sao cho f() = 0.
Khi đó từ (1) lấy x =  ta được: + f ( y) = af ( y), y
  hay:  = (a −1) f (y), y   (3)
Từ (3) thì sẽ xảy ra hoặc a = 1 hoặc f là hàm hằng.
+) Nếu f là hàm hằng thì không thoả mãn (1).
+) Nếu a = 1 chọn f ( x) = x thoả mãn (1).
CHUYÊN ĐÊ 1: ĐẠI SỐ VÀ GIẢI TÍCH 43 |
Tài liệu chuyên đề bồi dưỡng học sinh giỏi Vậy a = 1.
4. Bài tập củng cố
Bài tập 1: Tìm tất cả các hàm f : → thỏa mãn
f ( x + f ( x) + 2 f ( y)) = x + f ( x) + y + f ( y) ( ) 1 với x  , y  . Lời giải:
Giả sử tồn tại hàm f ( x) thỏa mãn.
Ta chứng minh f là đơn ánh. Thật vậy, giả sử tồn tại y , y thỏa mãn f ( y = f y . Cố định x , lần 1 ) ( 2 ) 1 2
lượt cho y = y , y = y từ ( ) 1 ta có: 1 2
f ( x + f ( x) + 2 f ( y
= f x + f x + 2 f y
x + f x + y + f y = x + f x + y + f y 1 )) ( ( ) ( 1)) ( ) 1 ( 1) ( ) 2 ( 2 )
Suy ra y = y . Vậy f là đơn ánh. 1 2
Bài tập 2: (Thụy Sĩ 2010) Tìm tất cả các hàm f : → thỏa mãn:
f ( f ( x) + f ( y)) = 2y + f ( x y) x  , y  . ( ) 1 Lời giải:
Trong (1) cho x = y được: f (2 f ( x)) = 2x + f (0) x
  . Từ đây dễ dàng suy f đơn ánh.
Trong (1) cho x = y = 0 được: f (2 f (0)) = f (0) ⎯⎯ → f (0) = 0.
Trong (1) cho y = 0 được:
f ( f ( x) + f (0)) = f ( x) x   ⎯⎯
f (x) + f (0) = x, x   ⎯⎯
f (x) = x, x   .
Thử lại thấy thỏa mãn.
Bài tập 3: (Tây Ban Nha 2012) Tìm tất cả các hàm f : → thỏa mãn:
(x − 2) f ( y)+ f ( y + 2 f (x)) = f (x + yf (x)), x  , y  . ( ) 1 Lời giải:
Nếu f (0) = 0 trong (1) cho x = 0 được f ( y) = 0, y   .
Nếu f (0)  0 . Ta chứng minh f đơn ánh. Thật vậy trong (1) cho y = 0 được:
(x − 2) f (0)+ f (2 f (x)) = f (x), x
  . Từ đây dễ dàng suy ra f đơn ánh.
Trong (1) cho x = 2, y = 1 được: f (1+ 2 f (2)) = f (2 + f (2)) ⎯⎯
→1+ 2 f (2) = 2 + f (2) ⎯⎯ → f (2) =1
Do f đơn ánh nên f ( x)  1, x   2.
44 | C CHUYÊN ĐÊ: PHƯƠNG TRÌNH HÀM
Tài liệu chuyên đề bồi dưỡng học sinh giỏi
2 f ( x) − x Trong (1) thay y = ta có: f ( x) −1
( − )  2 f (x)− x
 2 f (x) − x   2 f x xx 2 f   +  +  =  +     f (x) f   f (x) 2 f ( x) ( ) f x   f ( x)
f ( x) , x 2 1 1 1  − − −        (
f x x x − 2) 2 ( ) f   =    f ( x) 0, x 2 1  −    ( )
2 f ( x) − x f a = 0, a =   f ( x) , x 2 −1
Trong (1) thay x = a, y = 2 ta có:
(a − 2) f (2)+ f (2+ 2 f (a)) = f (a + 2 f (a)) = 0  a = f (2) =1
2 f ( x) − x ⎯⎯ → =   ⎯⎯ → = −   f ( x) 1, x 2
f ( x) x 1, x 1. −1
Ta thấy với x = 2 cũng thỏa mãn hàm trên. Vậy bài toán có 2 nghiệm hàm f ( x) = 0, x   hoặc
f ( x) = x −1, x   .
Bài tập 4: (Thổ Nhĩ Kỳ 2012) Tìm tất cả các hàm f : → thỏa mãn: i) f ( f ( 2
x ) + y + f ( y)) 2
= x + 2 f ( y). ii)
x y thì f ( x)  f ( y). Lời giải:
• Ta chứng minh f đơn ánh.
Cho y = 0 thay vào i) được f ( f ( 2 x ) + f ( )) 2 0
= x + 2 f (0), x  ⎯⎯
f ( f (x) + f (0)) = x + 2 f (0), x  0. ( ) 1
Do đó f đơn ánh trên 0;+)
Giả sử tồn tại y , y thỏa mãn f ( y ) = f ( y ) i) ⎯⎯ → f ( f ( 2
x ) + y + f ( y )) = f ( f ( 2 x + y + f y . Vì 1 2 1 1 ) 2 ( 2)) 1 2
f không bị chặn nên cho x đủ lớn thì f ( 2
x ) + y + f ( y ), f ( 2 x
+ y + f y đều dương nên từ (1) kết 1 1 ) 2 ( 2)
hợp f đơn ánh trên 0; +) ta có y = y ⎯⎯
f đơn ánh trên . 1 2
• Ta chứng minh f (0) = 0.
+ Nếu f (0)  0 cho x = 2
f (0) thay vào (1) được f ( f ( 2
f (0)) + f (0)) = 0 hay tồn tại c thỏa f (c) = 0.
Cho x = 0, y = c thay vào i) được f ( f (0) + c) = 0. Vì f đơn ánh nên f (0) + c = c f (0) = 0.
CHUYÊN ĐÊ 1: ĐẠI SỐ VÀ GIẢI TÍCH 45 |
Tài liệu chuyên đề bồi dưỡng học sinh giỏi
Nếu f ( x)  0 cho x = y = 0 thay vào i) được f (2 f (0)) = 2 f (0).
Trong (1) cho x = x = 3 f 0 = f 0 + f 2 f 0 có: 0 ( ) ( ) ( ( ))
f ( x = f f 0 + f 2 f 0
= 2 f 0 + 2 f 0 = 4 f 0 . 0 ) ( ( ) ( ( ))) ( ) ( ) ( )
Ta có f ( f ( x + f 0 = f 5 f 0 = 3 f 0 + 2 f 0 = 5 f 0 . 2 0 ) ( )) ( ( )) ( ) ( ) ( ) ( )
Cho x = 0, y = 2 f (0) thay vào (1) được f (5 f (0)) = 4 f (0). (3)
Từ (2) và (3) suy ra f (0) = 0.
• Khi đó (1) viết lại f ( f (x)) = x, x  0. (2)
Thay x = 0 vào i) ta được f ( y + f ( y)) = 2 f ( y),  y  .
Cho y = f ( x) ta được f ( f ( x) + x) = 2x, x   .
Từ (2) và (3) suy ra f ( x) = x, x   0. (3)
• Với mỗi y  luôn tồn tại x sao cho 2
x + y + f y
 0 . Do đó từ i) và (3) ta có 0 0 ( 0) 0 0 f ( 2
x + y + f ( y )) 2
= x + y + f ( y ) 2
= x + 2 f y ⎯⎯
f y = y ⎯⎯
f x = x, x   . 0 0 0 0 0 0 0 ( 0 ) ( 0 ) 0 ( )
Thử lại thấy thỏa mãn.
Bài tập 5: (Olympic Duyên hải Bắc bộ 2016) Tìm tất cả các hàm số f : → sao cho:
f ( xf ( x + y)) = f ( yf ( x)) 2 + x , x  , y  (1) Lời giải:
Cho x = 0 vào (1) ta có f (0) = f ( yf (0)), y
  , suy ra f (0) = 0 vì nếu ngược lại cho t y =
thì f (0) = f (t ), t
  ta thu được f là hàm hằng, thay vào (1) thấy vô lý. f (0)
Cho y = 0 và y = − x vào (1) thì có ( ( )) 2 f xf x = x và = ( ( − )) = (− ( )) 2 2 0 f xf x x f xf x + x x
  → −x = f (−xf (x)) .
Nếu tồn tại t  0 sao cho f (t = 0 thì 0 = f (t f t = t vô lý. 0 ( 0 )) ( 0)2 0 ) 0
Chứng minh hàm số cần tìm là đơn ánh.
Giả sử f ( x) = f ( y) ta có:
46 | C CHUYÊN ĐÊ: PHƯƠNG TRÌNH HÀM
Tài liệu chuyên đề bồi dưỡng học sinh giỏi 2
x = f ( xf ( x)) = f ( xf ( y)) = f (( y x) f ( x)) 2
+ x f (( y x) f (x)) = 0  f (x) = 0 →  → x = yx y = 0
Ta chứng minh f (−x) = − f ( x)  x  .
Ta xét x  0 vì nếu x = 0 là hiển nhiên.
Giả sử f ( x)  0 ⎯⎯ → z   0 sao cho ( ) 2
f x = z . Vì f đơn ánh và ( ( )) 2 f zf z
= z nên x = zf (z) do đó (− ) = (− ( )) 2 f x f zf z
= −z = − f (x) .
Trong trường hợp f ( x)  0 chứng minh tương tự. Mặt khác, ta có
f ( yf ( x)) 2
= −x + f (xf (x + y))
= −x + (x + y)2 − (x + y)2 2
+ f (−xf (x + y))   2
= y + 2xy f ((x + y) f ( y))
= 2xy + (−y)2 + f ((x + y) f (−y)) = 2xy + f (−yf (x))  
Suy ra f ( yf ( x)) = xy ,  x, y  .
Tương tự f (xf ( y)) = xy vì thế xf ( y) = yf (x) → f (x) = cx,  x  thay vào (1) suy ra c  1 − ;  1 .
Thử lại hai hàm số f ( x) = x thỏa mãn yêu cầu.
Bài tập 6: (Argentia TST 2008) Tìm tất cả các hàm f : (0;+) → (0;+) thỏa mãn 2
x ( f ( x) + f ( y)) = ( x + y) f ( yf ( x)), x  , y  0. ( ) 1 Lời giải:
Ta chứng minh f là đơn ánh. Gỉa sử tồn tại x , x  0 thỏa mãn f ( x = f x  0 . 1 ) ( 2) 1 2
Lần lượt cho x = x , x = x thay vào ( ) 1 ta được: 1 2 2
x f x + f y = x + y f yf x  1 ( ( 1 ) ( )) ( 1 ) ( ( 1)) 2 2 
x x + y = x x + y 2
x f x + f y = x + y f yf x 2  ( ( 2 ) ( )) ( 2 ) ( ( 2)) 1 ( 2 ) 2 ( 1 )
 (x x x x + y x + x = 0  x = x 1 2 ) ( 1 2 ( 1 2 )) 1 2
Vậy f đơn ánh trên (0;+). Cho x = y thay vào ( ) 1 có: 2
2x f ( x) = 2xf ( xf ( x)) → xf ( x) = f ( xf ( x)), x   0 (2)
CHUYÊN ĐÊ 1: ĐẠI SỐ VÀ GIẢI TÍCH 47 |
Tài liệu chuyên đề bồi dưỡng học sinh giỏi
Trong (2) cho x = 1 ta thu được f ( ) 1 = f ( f ( ) 1 ) → f ( ) 1 = 1. Trong ( )
1 cho x = 1 ta thu được + f ( y) = ( y + ) f ( y) → f ( y) 1 1 1 = , y
  0. Thử lại thấy thỏa mãn. y
Bài tập 7: Tìm tất cả các hàm số f : (0; +) → (0; +) thỏa mãn : xf ( 2
x ) f ( f ( y)) + f ( yf ( x)) = f (xy)( f ( f ( 2
x ) + f ( f ( 2
y ) ), với mọi x, y (0;+). Lời giải: Thay ( ; x y ) = (1; ) 1 ta được: f ( ) 1 f ( f ( ) 1 ) + f ( f ( ) 1 ) = f ( ) 1 ( f ( f ( ) 1 ) + f ( f ( )
1 )) → f ( f ( ) 1 ) = f ( ) 1 f ( f ( ) 1 ) Vì f ( f ( )
1 )(0;+) nên f ( ) 1 = 1. Thay ( ;
x y ) = (1; x) và sử dụng f ( )
1 = 1. ta được: f ( f ( x)) = f ( x) f ( f ( 2 x ) , x   0. (2) Thay ( ; x y ) = ( ; x ) 1 và sử dụng f ( )
1 = 1. ta được xf ( 2
x ) = f ( x) 2 x f ( 2 ;
x ) = xf ( x), x   0. (3) f f x 2 ( )
Từ (2) và (3) suy ra: f ( f ( x ) ( ) = = f ( f ( 2 x )   f ( x) , x 0. (4)
Ta chứng minh f là một đơn ánh, thật vậy giả sử  ,   (0; +) thỏa mãn f ( ) = f ( ) . Khi f f f f  2 ( ) ( ( ))
đó f ( f ( ) ( ) = = = f ( f ( 2  )   f ( ) f ( ) , x 0. (5)
Trong (1) thay y bởi x ta được: xf ( f x f f x 2 x ) f ( 2 2 2 ( )
f ( x)) + f ( xf ( x)) = 2 f ( x ) f ( f (x ) ( ) = 2 f ( x ) ( ) = 2 , x   0 x x
xf (x) f ( f (x)) + xf (xf (x)) = 2 f ( f (x)), x   0. (6)
xf (xf (x)) = f ( f (x))(2 − xf (x)), x   0. (7)
Trong (6) thay x bởi 2 x ta được: 2 x f ( 2
x ) f ( f ( 2 x ) 2 + x f ( 2 x f ( 2
x ) = f ( f ( 2 x ) 2 x
  → x f (xf (x)) = f ( f ( 2 2 , 0
x ) (2 − xf (x)), x   0. (8) f ( f ( 2 x )
Từ (6) và (8) suy ra: x =  
f ( f ( x)) , x 0. (9) ( ( 2) ( ( 2 f f f f  ) Từ (5) và (8) suy ra  = =
=  . Do đó f là đơn ánh. Kết hợp (4) suy ra f ( f ( )) f ( f ( ))
48 | C CHUYÊN ĐÊ: PHƯƠNG TRÌNH HÀM
Tài liệu chuyên đề bồi dưỡng học sinh giỏi
f ( x ) =  f  (x) 2 f ( x)  , x   0 → =  f   (x) 2 1 2  , x   0 ⎯⎯ → f  (x) = , x   0. x x
Thử lại thấy f ( x) 1 = , x
  0 thỏa mãn điều kiện ban đầu. x
Bài tập 8: Tìm hàm f : →
thỏa mãn một trong hai điều kiện i) f ( 2
x + f ( y)) = y + xf ( x) x  , y  , 2 ii)
f ( f (x)) + f ( y)) = y + xf (x) x  , y  . Hướng dẫn giải:
Ta tìm hàm f thỏa mãn ii) Đối với i) ta làm tương tự. Ngoài ra có thể thấy hai điều kiện này có thể biến đổi về nhau.
Ta cũng dễ thấy f là đơn ánh và f (0) = 0, f ( f ( y)) = y y   .
Trong ii) thay x bởi f ( x) ta có 2 f ( f
 ( f (x)) + f ( y) 
)= y+ f (x) f (f (x)).
Mặt khác f ( f ( y)) = y, y   nên f ( 2
x + ( y)) = y + xf ( x), x  , y  . 2 2 Kết hợp ii) thì ( 2
f x + f ( y)) = f ( f (x)) + f ( y)) mà f đơn ánh nên 2
x + f ( y) = ( f ( x)) + f ( y) . Suy ra ( f ( x))2 2 = x , x   .
Ta chỉ ra không tồn tại đồng thời a  0, b  0 thỏa mãn f (a) = a, f (b) = b
− . Thật vậy, giả sử tồn tại
a, b như trên. Trong ii) lấy x = a, y = b ta có f ( 2 a b) 2 = a + . b 2 2 Do ( f ( x))2 2 = x , x   nên ( 2 a b) = ( 2 a + b) 2
a b = 0 , mâu thuẫn.
Vậy f ( x) = x x
  hoặc f (x) = −x x   .
Thử lại thấy hai hàm này thỏa mãn.
Bài tập 9: Tìm tất cả các hàm số f : → thỏa mãn
f ( x + y + f ( y)) = f ( f ( x)) + 2y, x  , y  . Hướng dẫn:
Chứng minh f là đơn ánh
Thật vậy, với mọi x, y thỏa mãn f ( x) = f ( y) ta có
CHUYÊN ĐÊ 1: ĐẠI SỐ VÀ GIẢI TÍCH 49 |
Tài liệu chuyên đề bồi dưỡng học sinh giỏi f
 ( x + y + f ( y)) = f ( f ( x)) + 2y  ( → = + + ( )) = ( ( )) x y f x y f x f f y + 2x
Chọn y = 0 ta được f ( x + f (0)) = f ( f ( x)) ⎯⎯
f (x) = x + f (0), x  
Vậy f ( x) = x + c, x
  với c là hằng số.
Thay vào điều kiện bài toán ta được
x + y + f ( y) + c = f ( x) + c + 2y x + y + y + 2c = x + 2c + 2y (luôn đúng).
Bài tập 10: Tìm hàm số f : → sao cho:
f ( f ( x) + 2y) = 3x + f ( f ( f ( y)) − x) x  , y  (1) Lời giải: f (x)
   − f (x)  
Đặt a = f (0) . Trong (1), cho y =
ta có a = 3x + f f f   − x x   . 2   2       
suy ra f là toàn ánh.
Ta đi chứng minh f là đơn ánh. Thật vây giả sử f (u) = f (v)
Từ (1) lần lượt cho y bởi u, v ta có
f ( f ( x) + 2u) = 3x + f ( f ( f (u)) − x) x   (2)
f ( f ( x) + 2v) = 3x + f ( f ( f (v)) − x) x   (3)
Lấy (2) trừ (3) ta có f ( f ( x) + 2u) = f ( f ( x) + 2v) x
  . Do f là toàn ánh nên suy ra được
f ( x + 2(u v)) = f ( x) x   (4).
Từ (4) , sử dụng phương pháp quy nạp ta có f ( x + 2n(u v)) = f ( x) x   , n   . (5)
Từ (1), cho x bởi x + 2(u v) ta thu được:
f ( f ( x + 2(u v)) + 2y) = 3x + 6(u v) + f ( f ( f ( y)) − x − 2(u v)) x  , y
f ( f (x) + 2y) = 3x + 6(u v) + f ( f ( f ( y))− x) x  , y  (6)
Từ (1) và (6) ta suy ra u v = 0 . Vậy f đơn ánh.
Trong (1) cho x = 0 ta có f ( z + 2y) = f ( f ( f ( y))) → f ( f ( y)) = 2y + a y  
Từ đó (1) trở thành f ( f ( x) + 2y) = 3x + f ( y + a x) x  , y
Lại cho tiếp y = 0 ta có :
50 | C CHUYÊN ĐÊ: PHƯƠNG TRÌNH HÀM
Tài liệu chuyên đề bồi dưỡng học sinh giỏi
f ( f ( x)) = 3x + f (a x) x  
→ 2x + a = 3x + f (a x) x  
f (a x) = a x x  
f (x) = x x  
Thử lại ta có f ( x) = x x
  là nghiệm của phương trình (1). Bài tập 11:
Tìm tất cả các hàm số f : thỏa mãn: 2
f (x + y + z) = f ( f (x)) + yf (x) + f (z), x
 , y, z  (1) Lời giải:
Giả sử tồn tại hàm số f thỏa đề bài ra.
• Nhận thấy f (x)  0, x   thỏa (1)
• Giả sử f (x)  0 tức tồn tại u  sao cho f (u)  0 .
Kí hiệu P (u,v) thay x bởi u, y bởi v vào (1). Với x  ta có: 2  
x f ( f (u)) − f (0) 
x f ( f (u)) − f (0)  p u; ; 0 → f u;  = x ⎯⎯ → f     là toàn ánh. f (u)  f (u)        p( ;
x 0; 0) → f (x) = f ( f (x)) + f (0) → f (x) = x + f (0)
(Vì f là toàn ánh nên f ( f (x)) = x ) (2)
Thay (2) vào (1) ta được: 2 2
x + y + z + f (0) = f (x + f (0)) + y(x + f (0)) + z + f (0) → x + y = f (x + f (0)) + y(x + f (0)) 2 2
x + y = x + f (0) + f (0) + y(x + xf (0)) → y = 2 f (0) + y(x + xf (0)) (3)
Thay y bởi vào (3) ta được: f (0) = 0 khi đó từ (2) ta được: f (x) = x, x   .
Thử lại, thấy hàm số này không thỏa mãn(1). Do đó có duy nhất một hàm số thỏa mãn yêu cầu bài toán
là: f (x)  0, x   . Nhận xét:
Ngoài cách nhận biết tính chất toàn ánh của f dựa vào một vế của đẳng thức là hàm bậc nhất ta còn có
thể chứng minh f là toàn ánh bằng cách y
  R,x R thỏa mãn f (x) = y
x f ( f (u)) − f (0)
Sỏ dĩ ta có thể nhận biết được cách thay y bởi vào (1) là do y 2
p(u, y, 0) → f (u + y ) = f ( f (u)) + yf (u) + f (0), y   R
Với x R xét
x f ( f (u)) − f (0)
x = f ( f (u)) + yf (u) + f (0) → y = f (u) Vì vậy ta thực hiện
CHUYÊN ĐÊ 1: ĐẠI SỐ VÀ GIẢI TÍCH 51 |
Tài liệu chuyên đề bồi dưỡng học sinh giỏi
x f ( f (u)) − f (0)  p u,    f (u) 
Để suy ra f là toàn ánh.
Đối với một bài toán phương trình hàm nếu ta có thể vận dụng tính chất toàn ánh của f điều dó
giúp ta tính được một số giá trị đặc biệt của hàm như f (0); f ( )
1 làm cơ sở để tìm ra đáp án của bài toán.
Bài tập 12: Tìm tất cả các hàm số f :
→ thỏa mãn điều kiện:
f ( y + f ( x)) 4 = f (x) 3 + y f (x) 2 2 + y f (x) 3 4 6
+ 4yf (x) + f (−y), x  , y  ( ) 1 . Hướng dẫn:
Giả sử tồn tại hàm số f thỏa mãn (1).
Trường hợp 1: f ( x) = 0 . Thử lại ta thấy f ( x) = 0 thỏa mãn (1).
Trường hợp 2: f ( x)  0 → x   , f x  0 . 0 ( 0) 4
Ta có: ( )  f ( y + f ( x)) − f (−y) =  y + f ( x) 4 1  − y , x  , y    (*)
Thay x = x vào (*), ta được 0
(y + f (x ))− f (−y) = y + f  (x ) 4 4  − y , x  , y  2 0 0  ( )
Ta thấy vế phải của (2) là một hàm số bậc 3 nên có tập giá trị là
. Do đó hàm số f có tập giá trị là → y   đều u
 ,v  sao cho f (u) − f (v) = y . 4
Thay y = 0 vào (*), ta được f ( f ( x)) =  f
 ( x) + a, x   
(a = f (0)) (3).
Thay y = − f ( y) vào (*), ta được
f ( f ( x) − f ( y)) − f ( f ( y)) =  f ( x) − f ( y) 4
 −  f ( y) 4  , x  , y      (4).
Từ (3) và (4) suy ra f ( f ( x) − f ( y)) =  f ( x) − f ( y) 4  + a, x  , y    hay
f ( f (u) − f (v)) =  f (u) − f (v) 4  + a, u  ,v    → f ( y) 4 = y + a y   ⎯⎯ → f (x) 4 ,
= x + a, x   Thử lại ta thấy ( ) 4
f x = x + a thỏa mãn điều kiện (1).
Vậy f ( x) = 0 và ( ) 4
f x = x + a ( a là hằng số) là các hàm số cần tìm.
Bài tập 13: Tìm tất cả các hàm số f : → thỏa mãn điều kiện:
f (x f ( y)) = f (x) + f ( f ( y)) − 2xf ( y) + f ( y) + 2012 x  , y  . Lời giải:
52 | C CHUYÊN ĐÊ: PHƯƠNG TRÌNH HÀM
Tài liệu chuyên đề bồi dưỡng học sinh giỏi
Cho x = f ( y) ta thu được 2
f (0) = 2 f ( f ( y)) − f ( y) + f ( y) + 2012 1 1 1 Hay 2
f ( f ( y)) = f ( y) − f ( y) −1006 + f (0) (2) 2 2 2 Và ta đoán 1 1 2 f (x) = x x + c (*) 2 2
Nhưng để có điều này ta cần chỉ ra f toàn ánh. Công việc này có vẻ khó. Ta thử thêm chút:
Thay x bởi f (x) ta có f ( f (x) − f ( y)) = f ( f (x)) + f ( f ( y)) − 2 f (x) f ( y) + f ( y) + 2012
Và sử dụng (2) ta được 1 1 2
f ( f (x) − f ( y)) =
( f (x) − f ( y)) −
( f (x) − f ( y)) + f (0) 2 2
Và để thực hiện được dự đoán (*), ta cần chỉ ra với mọi t , tồn tại x , y để t = f (x ) − f ( y ) ? 0 0 0 0
Hiển nhiên là f không thể đồng nhất 0 . Do đó tồn tại a f (a)  0 . Trong (1), cho y = a ta được
f (x f (a)) = f (x) + f ( f (a)) − 2xf (a) + f (a) + 2012
Hay f ( y f (a)) − f ( y) = f ( f (a)) − 2 yf (a) + f (a) + 2012
Với mỗi số thực t , ta cần chọn x ; y để thay vào đẳng thức này sao cho vế phải là t hay 0 0
f ( f (a)) + f (a) + 2012 − t y =
và vế trái là f (x ) − f ( y ) hay x = y f (a) 0 2 f (a) 0 0 0 0
Khi đó ta có f (x ) − f ( y ) = f ( y f (a)) − f ( y ) = f ( f (a)) − 2 y f (a) + f (a) + 2012 = t 0 0 0 0 0 Vậy 1 1 2 f (t) = t
t + f (0) . Thay t bởi f ( y) và sử dụng (2) ta được 2 2 1 1 1 2 1
− 006 + f (0) = f (0) → f (0) = 2012 → f (x) = x x + 2012,, x
  , thử lại thỏa. 2 2 2
Bài tập 14: (Hàn Quốc 2003) Tìm tất cả các hàm số f : → thỏa mãn:
f ( x f ( y)) = f ( x) + xf ( y) + f ( f ( y)), x  , y  (4) Lời giải:
Nhận thấy hàm f ( x)  0 thỏa mãn yêu cầu bài toán. Xét trường hợp f ( x)  0
Thế x = f ( y) vào (4) ta được f ( x f
0) = 2 f ( z) + z f ( x) 2 0 2 ( ) = − + . 2 2 Hay f ( 2 f ( x)) f ( x) f (0) = − + . 1 2
Thế x = f ( z) , với z là một số thuộc thì ta được
f ( f ( z) − f ( y)) = f ( f ( z)) + f ( z) f ( y) + f ( f ( y)) .
CHUYÊN ĐÊ 1: ĐẠI SỐ VÀ GIẢI TÍCH 53 |
Tài liệu chuyên đề bồi dưỡng học sinh giỏi Với lưu ý là 2 f z f 0 f ( 2 f ( y)) f ( y) f (0) = − +
f ( f ( z)) ( ) ( ) = − + 2 2 2 2
Thay vào quan hệ hàm ở trên ta được ( f z f y )2
f ( f ( z) − f ( y)) ( ) ( ) = − + f (0) . (5) 2
Tiếp theo ta chứng tỏ tập  f ( x) − f ( y) | x, y   = . Do f ( x)  0 nên tồn tại một giá trị y sao cho 0
f ( y = a  0 . Khi đó từ quan hệ (4) ta có 0 )
f ( x a) = f ( x) + xa + f (a) → f ( x a) − f ( x) = ax + f (a) .
Vì vế phải là hàm bậc nhấ cả X nên xa + f (a) có tập giá trị là toàn bộ
. Do đó hiệu f ( x a) − f (x) cũng
có tập giá trị là toàn bộ , khi x  . Mà
f (x)− f (y)| x, y  f (xa)− f (x)| x  = ,
Do đó  f (x) − f ( y) | x, y   = . Vậy từ quan hệ (5) ta thu được ( ) 2 x f x = − + f (0), x   2 Mặt khác ta lại có ( ) 2 x f x = − + f (0), x   . 2 x
Nên f (0) = 0 . Thử lại thấy hàm số f ( x) 2 = − , x
  thỏa mãn hệ hàm. 2 x
Kết luận: Có hai hàm số thỏa mãn là f ( x) 2 = − , x
  hoặc f (x)  0, x   .. 2
Nhận xét: Bài toán trên lấy ý tưởng từ bài thi IMO 1996: Tìm tất cá các hàm số f : → thỏa mãn
f ( x f ( y)) = f ( f ( y)) + xf ( y) + f ( x) −1, x  , y  . Đáp số xf ( x) 2 = − +1, x   2
Bài tập 15: (Việt Nam TST 2004) Tìm tất các giá trị của a sao cho tồn tại duy nhất một hàm số f :
→ thỏa mãn điều kiện:
f ( x + y + f ( y)) = ( f ( x))2 2 + ay, x  , y  . (1) Lời giải:
54 | C CHUYÊN ĐÊ: PHƯƠNG TRÌNH HÀM
Tài liệu chuyên đề bồi dưỡng học sinh giỏi
Giả sử tồn tại hàm số f ( x) thỏa yêu cầu bài ra.
Khi a = 0 , từ (1) ta được 2 2
f (x + y + f ( y)) = ( f (x)) , x
 , y  . khi đó ta có hai hàm số thỏa mã là
f (x)  0 và f (x)  1 → a = 0 (loại).
Khi a  0 . Vì vế phải là hàm bậc nhất nên y có tập giá trị là
. Do đó f là toàn ánh ⎯⎯ → b
  : f (b) = 0 .
Tìm b . Thay y bởi b vào (1) ta được : 2 2
f (x + b) = ( f (x)) + a , b x   . (2)
Thay x bởi –x vào (2) ta được : 2 2
f (x + b) = ( f (−x)) + ab, x   .
f (x) = f (−x)
Từ (2) và (3) ta được : 2 2
( f (x)) = ( f (−x)) → ⎯⎯ → f ( b − ) = 0 
f (x) = − f (−x)
Thay y bởi –b vào (1) ta được : 2 2
f (x b) = ( f (x)) − a , b x   (4) Từ (3), (4) 2 2 ⎯⎯
f (x + b) − f (x b) = 2ab (5)
Thay x bởi 0 vào (5) ta được: f (b) − f ( b
− ) = 2ab → 2ab = 0  b = 0 (a  0)
Vậy f (b) = 0  b = 0.
Thay x bởi 0 vào (1) ta được : f ( y + f ( y)) = ay (6)
Thay y bởi 1 vào (6) ta được: f (1+ f ( ) 1 ) = a (8)
Thay y bởi 0 vào (1) ta được : ( ) = ( ( ))2 2 f x f x (7)  f 1 = 1 2
Thay x bởi 1 vào (7) ta được : f ( ) 1 = ( f ( ) 1 ) ( ) →   f  ( ) 1 = 0
f ( x) = 0 → x = 0 nên ta nhận f (1) = 1 Thay f ( )
1 = 1 vào (8) ta được f (2) = a a = 2 Ta có : 2 2 2 2 2
a = ( f (2)) = f (2 ) = f (4) = f (( 2) + 2) = f (( 2) ) + a = f (2) + a = 2a →  a = 0
a  0 nên ta nhận a = 2. Vậy a = 2. Khi đó (1) trở thành: 2 2
f (x + y + f ( y)) = ( f (x)) + 2 y, x  , y  (9) 2 ( f (x)) Thay y bởi − vào (9) ta được: 2 2 2 2 2  ( f (x))
 ( f (x))   ( f (x))
 ( f (x))  2 2 2 2 f x − + f −
 = ( f (x)) − ( f (x)) → f x − + f −  = 0 2   2  2    2  2 2 ( f (x))  ( f (x))  2 → x − + f −  = 0, x   2  2 
Vì ( f (x) = 0  x = 0 )
CHUYÊN ĐÊ 1: ĐẠI SỐ VÀ GIẢI TÍCH 55 |
Tài liệu chuyên đề bồi dưỡng học sinh giỏi 2 2  ( f (x))  ( f (x)) 2 → f −  = −x − (10)  2  2 Hay 2 2
f ( y) = −x y f ( y) + y = −x , x   (11) 2 ( f (x)) Thay y bởi −
f ( y) + y bởi 2
y do (11), kết hợp với (7) khi đó từ (9) ta được: 2 2 2 2 2 2 2
f (x y ) = ( f (x)) − ( f ( y)) = f (x ) − f ( y ), x  , y  (12)
Từ (12) thay x bởi 0 ta được: 2 2
f (− y ) = − f ( y ) ⎯⎯
f là hàm số lẻ. Khi đó từ (1) ta có:
f (x + y) = f (x − (− y)) = f (x) − f (− y) = f (x) + f ( y), x  , y  ⎯⎯ → f cộng tính. Ta lại có: 2 2
( f (x)) = f (x ) nên 2 2 2 2 2
( f (x + y)) = f (x + y)  ( f (x) + f ( y)) = f (x + 2xy + y ) 2 2 2 2
 ( f (x)) + 2 f (x) f (y) + ( f ( y)) = f (x) + f ( y ) + f (2xy)
f (x). f (y) = f (xy) → f nhân tính.
Hàm f vừa cộng tính vừa nhân tính nên f (x)  x thử lại ta thấy hàm f (x)  x thỏa yêu cầu bài toán. Nhận xét:
Đây là một bài toán khó, tính chất toàn ánh của f sử dụng khá hiệu quả; dấu hiệu nhận biết tính chất
toàn ánh ở đây chính là vế phải đẳng thức là hàm bậc nhất, khi khẳng định f là toán ánh ta luôn b
  R : f (b) = 0 , từ đó ta đi tìm b .
Từ điều kiện f (0) = 0 ta nhận xét được một loạt tính chất đặc biệt của f như f (0) = 0 , 2 2
f (x ) = ( f (x)) ,
tính chất f là hàm số lẻ; tính nhân tính; cộng tính của hàm f làm cơ sở để tìm ra kết quả của bài toán.
Nếu ban đầu ta không tìm cách vận dụng tính chất toàn ánh của f thì bài này khó có thế giải quyết được.
Vì vậy khi giải một bài toán phương trình hàm thì việc nhận biết và vận dụng các tính chất của hàm f
rất quan trọng nó sẽ giúp việc giải quyết bài toán một cách dễ dàng hơn.
Bài tập 16: Tìm hàm số f :
→ thỏa mãn điều kiện:
f ( x f ( y)) = 2 f ( x) + x + f ( y), x  , y  . (6) Lời giải:
Nhận thấy hàm f ( x)  0 không thỏa mãn yêu cầu. Xét f ( x)  0 .
Thay x bởi f ( y) vào (6) ta được
f ( f ( y)) = − f ( y) f (0) + 2
Lại thay x bởi f ( x) ta được
56 | C CHUYÊN ĐÊ: PHƯƠNG TRÌNH HÀM
Tài liệu chuyên đề bồi dưỡng học sinh giỏi
f ( f ( x)) − f ( y) = 2 f ( f ( x)) + f ( x) + f ( y)  f  = 2− f (x) (0) +
 + f ( x) + f ( y) 2  
= −( f (x) − f ( y)) + f (0)
Tuy nhiên việc chứng minh tập  f ( x) − f ( y) | x, y   có tập giá trị là chưa thực hiện được. Từ đây ta có
f ( f ( x) − 2 f ( y)) = f ( f ( x) − f ( y) − f ( y))
= 2 f ( f (x) − f ( y)) + f (x) − f ( y) + f ( y) = 2
− ( f (x) − f ( y)) + 2 f (0) + f (x)
= −( f (x) − 2 f ( y)) + 2 f (0).
Ta sẽ chứng minh tập  f ( x) − 2 f ( y) | x, y   bằng . Thật vạy tồn tại giá trị y  sao cho 0
f ( y = a  0 . Khi đó thay y = y vào (6) ta có 0 ) 0
f ( x a) − 2 f ( x) = x + a, x   Mà khi x
thif x + a có tập giá trị là
. Chứng tỏ tập  f ( x) − 2 f ( y) | x, y   = . Mà
f (x)−2 f (y)| x, y  f (x a)− f (x)| x  nên  f (x)−2 f (y)| x, y  = . Do đó từ (c) ta kết
luận f ( x) = −x, x
  . Thay vào (6) ta được f (0) = 0
Kết luận: Hàm số f ( x) = −x, x
  thỏa mãn yêu cầu bài toán.
Bài tập 17: Tìm tất cả các hàm f : → thỏa mãn:
f ( x + f ( y)) − f ( x) = ( x + f ( y))4 4 − x , x  , y  . ( ) 1 Lời giải:
Nhận thấy f  0 là một nghiệm hàm. Xét f  0 , khi đó tồn tại a
để f (a)  0 . Trong (1) cho 4
y = a ta được: f ( x + f (a)) − f ( x) = ( x + f (a)) 4 − x , x  , y  . (2)
Chú ý VT(2) là đa thức bậc 3 đối với x nên tập giá trị của f (x + f (a)) − f (x) là . Suy ra
f ( x + f (a)) − f ( x) là toàn ánh. Do đó với mỗi t  đều tồn tại u, v  thỏa t = f (u) − f (v).
Trong (1) thay x bởi − f ( y) ta được
f ( ) − f (− f ( y)) 4
= − f ( y) y
   f (− f ( y)) 4 0 ,
= f ( y) + f (0), y   . (3)
Trong (1) thay x bởi − f ( x) và sử dụng (3) ta được
CHUYÊN ĐÊ 1: ĐẠI SỐ VÀ GIẢI TÍCH 57 |
Tài liệu chuyên đề bồi dưỡng học sinh giỏi
f (− f ( x) + f ( y)) − f (− f ( x)) = (− f ( x) + f ( y))4 4
f (x), x  , y
f (− f (x) + f ( y)) = (− f (x) + f ( y))4 + f (0), x  , y  . (4)
Trong (4) cho x = u, y = v được
f (− f (u) + f (v)) = (− f (u) + f (v))4 + f (0)  f (t ) 4
= t + f (0), t   ⎯⎯ → f (x) 4
= x + f (0), x  
Thay vào (1) được f (0) = 0. Vậy bài toán có 2 nghiệm hàm f ( x) = 0, x   hoặc f ( x) 4 = x +1, x   .
58 | C CHUYÊN ĐÊ: PHƯƠNG TRÌNH HÀM
Tài liệu chuyên đề bồi dưỡng học sinh giỏi
TÀI LIỆU THAM KHẢO [1]
Nguyễn Trọng Tuấn, Bài toán hàm số qua các kỳ thi Olympic. NXBGD, 2005. [2]
Nguyễn Tài Chung, Chuyên khảo phương trình hàm. NXB Đại học quốc gia Hà Nội, 2013. [3]
Trần Nam Dũng (CB), Các kỳ thi toán VMO – Lời giải và bình luận. NXB Thế giới, 2017.
[4] Titu Andreescu, Iurie Boreico, Oleg Mushkarov, Nikolai Nikolov, Topics in Functional
Equations. XYZ Press 2014.
[5] Đề thi chọn ĐT học sinh giỏi của một số tỉnh.
[6] Một số tài liệu trên mạng internet.
CHUYÊN ĐÊ 1: ĐẠI SỐ VÀ GIẢI TÍCH 59 |